You are on page 1of 260

CATapult Courseware

Module 3
Quantitative Ability
Published by IMS Learning Resources Pvt. Ltd. in the Year 2020

Registered Office: 6th Floor, NCL Building, ‘E’ Block, Near Bandra Family Court,
Bandra Kurla Complex (BKC), Bandra (E), Mumbai - 400051
Tel.: +91 22 66170000
Toll Free: 1800-1234-467
CIN : U80220MH1999PTC121823
E-mail : support@imsindia.com Website: www.imsindia.com

Copyright © IMS Learning Resources Pvt. Ltd.

All copyrights to this material vests with IMS Learning Resources Pvt. Ltd. No part of this material
either in part or as a whole shall be copied, reprinted, reproduced, sold, distributed or transmitted
in any form or by any means, electronic, mechanical, photocopying, recording or otherwise, or
stored in any retrieval system of any nature without the permission of IMS Learning Resources
Pvt. Ltd., and any such violation would entail initiation of suitable legal proceedings.

The views of and opinions expressed in this book are not necessarily those of the publishers. While
every effort has been made to ensure that all facts are stated correctly, the publishers regret their
inability to accept responsibility for any inadvertent errors or inaccuracies. Readers are advised in
their own interest to reconfirm facts before acting upon them.

The publishers shall endeavour, wherever possible to remedy all errors of commission and omission
which are brought to their attention in subsequent editions.

This book is sold subject to the condition that it shall not, but way of trader or otherwise, be
lent, resold, hired out, or otherwise circulated without the publisher’s prior written consent in any
form of binding or cover other than that in which it is published and without a similar condition
including this condition being imposed on the subsequent purchaser and without limiting the rights
under copyright reserved above.
CATapult
GEOMETRY

QA-3.1 TRIANGLES - I THEORY

Lines and Planes


Definition of Terms
Line: A line has length. It has neither width nor thickness. It can be extended indefinitely in both
directions.

Ray: A line with one end point is called a ray. The end point is called the origin. Two rays, which
lie on the same line and have only the origin as a common point are called opposite rays. Rays
OA and OB are opposite rays.

B O A

Line Segment: A line with two end points is called a segment.

Plane: A plane is a flat surface. It has length and width but no thickness.

Parallel lines: Two lines, which lie in a plane and do not intersect, are called parallel lines. The
distance between two parallel lines is constant. Parallel is indicated by the symbol ||.
Perpendicular lines: Two lines, which lie in a plane and intersect each other at right angles are
called perpendicular lines. Perpendicular is denoted by the symbol .

Note: The concept of right angles will be covered in Angles later in this chapter.

Properties of Lines and Planes


1. There are an infinite number of lines passing through a given point. These lines are called
concurrent lines.
2. There is one and only one line passing through two distinct points.
3. The intersection of two distinct lines is a point.
4. Three or more points are said to be collinear if they lie on a line, otherwise they are said
to be non-collinear.

1
CATapult
GEOMETRY

5. There are an infinite number of planes passing through any given line.
6. There is exactly one plane passing through three non-collinear points.
7. A line and a point not on the line lies in one plane.
8. Two distinct intersecting lines lie in a plane.
9. Two or more lines are said to be coplanar if they lie in the same plane, otherwise they
are said to be non-coplanar.
10. Four or more points are said to be coplanar if they lie in the same plane, otherwise they
are said to be non-coplanar.
11. If two points of a line lie in a plane, the whole line lies in the plane.
12. A-P-B implies that the points A, P and B are collinear and P lies in between A and B.
13. The intersection of two planes is a line.
14. A line, which intersects two or more given coplanar lines in distinct points, is called a trans-
versal of the given lines.
Example
In fig (i) below, line is a transversal of lines a, b, c and line a is a transversal of lines and m
15. A line which is perpendicular to a line segment i.e., intersects at 90° and passes through
the midpoint of the segment is called the perpendicular bisector of the segment.
16. Every point on the perpendicular bisector of a segment is equidistant from the two endpoints
of the segment.
P

A O B
Conversely, if any point is equidistant from the two endpoints of the segment, then it must
lie on the perpendicular bisector of the segment.
If PO is the perpendicular bisector of segment AB, then, AP = PB.
Also, if AP = PB, then P lies on the perpendicular bisector of , m
segment AB.
17. If two lines are perpendicular to the same line, they are parallel Q
a
P
to each other.
18. The ratio of intercepts made by three parallel lines on a trans-
versal is equal to the ratio of the corresponding intercepts made b
R S
on any other transversal by the same parallel lines.
If line a || line b || line c and line , and line m are two trans-
QS c
T U
versals, then, PR = .
RT SU
fig (i)

2
Chapter QA
TRIANGLES-I 3.1

Angles

THEORY
An angle is the union of two non-collinear rays with a common origin. A
The common origin is called the vertex and the two rays are the sides
of the angle.

PRACTICE EXERCISE CLASS EXERCISE


The angle AOB denoted by AOB, is formed by rays OA and OB and
point O is the common origin. Angles are measured in degrees or
radians.
mAOB denotes the measure of AOB. O B

Congruent Angles: Two angles are said to be congruent, (denoted by ), if their measures are equal.

Bisector of an angle: A ray is said to be the bisector of an angle if it A


divides the interior of the angle into two angles of equal measure.

Ray OX is the bisector of mAOB if mAOX = mXOB = 1 mAOB X


2

Every point on the angle bisector is equidistant from the sides of the
angle. Conversely, if any point in the plane of an angle is equidistant from
O B
the sides of the angle, then it lies on the angle bisector of the angle.
If OX is the angle bisector of AOB then AX = XB. Also, if AX = XB, then X lies on the angle
bisector of AOB.

Types of angles
A right angle is an angle of 90° as shown in fig.(i). Ray OB and Ray OA are perpendicular to each
other.
An angle less than 90° is called an acute angle.
An angle greater than 90° but less than 180° is called an obtuse angle.
An angle of 180° is a straight line.
An angle greater than 180° but less than 360° is called a reflex angle.

fig (ii)

3
CATapult
GEOMETRY

Pairs of Angles

Adjacent Angles: Two angles are called adjacent angles if they have a common
side and their interiors are disjoint i.e., separate.
AOB and BOC are adjacent angles.

Linear Pair: Two angles are said to form a linear pair if they have a common
side and their other two sides are opposite rays. The sum
of the measures of the angles is 180°. The angles that C
form a linear pair are always adjacent.
AOC and COB form a linear pair.
mAOC + mCOB = 180°

A O B

Complementary Angles: Two angles whose sum is 90°, are complementary,


each one is the complement of the other. If mAOC + mCOB = 90°, then
AOC and COB are complementary angles.

Supplementary Angles: Two angles whose sum is 180°, are supplementary,


each one is the supplement of the other. mAOB + mCED = 120°
+ 60° = 180° therefore these angles are supplementary.

Also, angles of a linear pair are supplementary. (Refer diagram of linear pair
as shown above).

Vertically Opposite Angles: Two angles are called vertically


A C
opposite angles if their sides form two pairs of opposite rays.
In other words, when the two lines intersect, two pairs of
vertically opposite angles are formed. O

Vertically opposite angles are congruent.


AOC and DOB are vertically opposite.
Also, AOD and COB are vertically opposite. D B
AOC  DOB and AOD  COB
Also, sum of all angles at a point is 360°.  mAOC + mDOB + mAOD + mCOB = 360°

4
Chapter QA
TRIANGLES-I 3.1

Corresponding Angles: When two lines are intersected by a transversal, they form A

THEORY
F
four pairs of corresponding angles. B
E
EF and GH are intersected at points B and C by the transversal AD. The four
pairs of corresponding angles are:

PRACTICE EXERCISE CLASS EXERCISE


G C H
(i) ABE, BCG (ii) EBC, GCD
D
(iii) ABF, BCH (iv) FBC, HCD
Fig.A

When the two parallel lines are intersected by a transversal, the pairs of corresponding angles
so formed are congruent. A
If EF is parallel to GH and AD is the transversal then: E FB
(i) ABE  BCG (ii) EBC  GCD
(iii) ABF  BCH (iv) FBC  HCD
G C H

Fig.B
Conversely, if the transversal intersects two lines and if one pair of corresponding angles is congruent
then the two lines are parallel. Hence, when one pair of corresponding angles is congruent, then
all the pairs of corresponding angles are congruent.

Alternate Angles: When two lines are intersected by a transversal, they form two pairs of alternate
angles.
In fig. A, the pairs of alternate angles are:
(i) EBC, BCH (ii) FBC, GCB
When two parallel lines are intersected by a transversal, the pairs of alternate angles so formed
are congruent.
In fig. B:
(i) EBC  BCH (ii) FBC  GCB
Conversely, if the transversal intersects two lines and if one pair of alternate angles is congruent,
then the two lines are parallel. Hence, when one pair of alternate angles is congruent then the
other pair of alternate and all pairs of corresponding angles are congruent.

Interior Angles: When two lines are intersected by a transversal, they form two pairs of interior
angles.
In fig.A, the pairs of interior angles are:
(i) EBC, GCB (ii) FBC, BCH
When two parallel lines are intersected by a transversal, the pairs of interior angles so formed
are supplementary.
In fig.B:
(i) mEBC + mGCB = 180° (ii) mFBC + mBCH = 180°

5
CATapult
GEOMETRY

Conversely, if the transversal intersects two lines and if one pair of interior angles is supplementary
then the two lines are parallel. Hence, when one pair of interior angles is supplementary, the other
pair is also supplementary and all pairs of alternate and corresponding angles are congruent.

SOLVED EXAMPLES

Q : AB and CD are two parallel lines. Find x, y and z.

A : x = 75° (vertically opposite angles)


y = 75° (alternate angles)
y + z = 180° (supplementary angles)
 z = 105°.

Q : From the given figure, calculate a and b. (Given lines l and m are parallel)

A : Draw the construction, as shown alongside.


OAD = AOC(alternate angle)
Thus, AOC = 45°
EBO = BOC = 30° (alternate angles)
Thus, b = 75°
a = 360° – 75° = 285°

6
Chapter QA
TRIANGLES-I 3.1

Q : Calculate x.

THEORY
PRACTICE EXERCISE CLASS EXERCISE
A : The sum of all angles = 180°  x + 3x + 8x = 180°
 12x = 180° x = 15

Q : Find x.

The sum of all the angles is 360°.


 3x + 37 + 68 = 360°
 3x = 360 – 105 = 255  x = 85°

7
CATapult
GEOMETRY

Q : Find x.

A : 133 + 90 + 79 + x = 360° x = 360 – 302 = 58

Q : In the adjoining figure mCBA is 70° determine x°.

A : AB || CD and BC || AD
mCBA + mBAD = 180° (interior angles on same side of the transversal) mBAD = 110°
mBAD = x° (corresponding angles)  x° = 110°

8
Chapter QA
TRIANGLES-I 3.1

Concept Builder 1

THEORY
1. In the given figure, lines l, m and n are parallel. If PR = 10 and PQ = 4. Find ST : TV.
a b

PRACTICE EXERCISE CLASS EXERCISE


l
P S

m
Q T

R V n

2.
x

25° o
z

y
Ray OZ is the bisector of XOY. Find mXOY
3. Measure of an angle is 55°. Find: a) complement of the angle b) supplement of the angle.
4. Line l || line m || line n. Find angles a, b, c, d, e
y P
l
30°

b a
60°
m
c

e
d
Q n

Answer key

4. a = 30°, b = 150°, c = 120°, d = 60°, e = 120°


3. a) 35° b) 125°
2. 50° 1. 2 : 3

9
CATapult
GEOMETRY

Triangles

The plane figure bounded by the union of three lines, which join three non- A
collinear points, is called a triangle. A triangle is denoted by the symbol .
The three non-collinear points are called the vertices of the triangle. In ABC,
A, B and C are the vertices of the triangle. Segment AB, segment BC and
segment AC are the three sides of the triangle.

ABC, BAC and ACB are the three interior angles of the triangle. Sides B C
of ABC can be denoted by a, b, c; where a = side opposite to vertex A, b = side opposite to
vertex B, c = side opposite to vertex C.

The angle formed by extending one side of a triangle with another side is called an exterior angle
of the triangle. A triangle has six exterior angles.

The exterior angles of ABC are FAC, ACD, ECB, CBI, HBA and
BAG.
Here, HBA = CBI, BAG = FAC and ACD = ECB ... [Vertically
opposite angles]

Properties of Triangles

Fig.C

1. The sum of the three interior angles of a triangle is 180°.


2. The sum of an interior angle and the adjacent exterior angle is 180°.
In fig.C, mABC + mABH = 180° & mABC + mCBI = 180°
3. Two exterior angles having the same vertex are congruent.
In fig.C, GAB  FAC
4. The measure of an exterior angle is equal to the sum of the measures of the two interior
angles (called remote interior angles) of the triangle, not adjacent to it.
In fig.C, mABH = mBAC + mBCA
Hence, an exterior angle of a triangle is greater than each of the interior angles not adjacent
to it.
e.g., mABH > mBAC & mABH > mBCA

10
Chapter QA
TRIANGLES-I 3.1

5. If two sides of a triangle are not congruent, then the angle opposite to the greater side is

THEORY
greater.
In ABC, If AB > AC, then mACB > mABC.
Conversely, if two angles of a triangle are not congruent, then the sides opposite to the

PRACTICE EXERCISE CLASS EXERCISE


greater angle is greater.
In ABC, if mABC > mACB then AC > AB i.e., if the sides are arranged in the ascending
order, then the angles opposite the sides will also be in ascending order
6. The sum of lengths of any two sides of a triangle is always greater than the third side. In
ABC, ,(AB) + ,(BC) > ,(AC), also ,(AB) + ,(AC) > ,(BC) and ,(AC) + ,(BC) > ,(AB).
7. The difference of any two sides is always less than the thirdside. From last result,
,(AB) + ,(BC) > ,(AC)
 ,(AB) > ,(AC) – ,(BC)
 ,(AC) – ,(BC) < ,(AB)
Similarly,  ,(BC) – ,(AB) < ,(AC) and ,(AC) – ,(AB) < ,(BC)
8. A triangle will have at least two acute angles.

Types of Triangles
1. With regard to their sides, triangles are of three types:
Scalene Triangle: A triangle in which none of the three sides is equal is A
called a scalene triangle.
Isosceles Triangle: A triangle in which at least two sides are equal is
called an isosceles triangle. In an isosceles triangle, the angles opposite to
the congruent sides are congruent. Conversely, if two angles of a triangle
are congruent the sides opposite to them are congruent.
In ABC, AB = AC, mABC = mACB B C

A
Equilateral Triangle: A triangle in which all the three sides are equal
is called an equilateral triangle. In an equilateral triangle, all the angles
are congruent and equal to 60°.
In ABC, AB = BC = AC
mABC = mBCA = mCAB = 60° B C

A
2. With regard to their angles, triangles are of three types.
Right Triangle: If any one angle of a triangle is a right angle, i.e. 90°,
then the triangle is a right angled triangle. The other two angles of the
right-angled triangle will be acute and complementary. The side opposite
to the right angle is called the hypotenuse.
B C
d
In ABC, AC is the
hypotenuse

11
CATapult
GEOMETRY

Acute triangle: If all the three angles of a triangle are acute i.e., less than 90°, then the triangle
is an acute-angled triangle.
Obtuse triangle: If any one angle of a triangle is obtuse i.e., greater than 90°,then the triangle is
an obtuse- angled triangle. The other two angles of the obtuse triangle will be acute.

SOLVED EXAMPLES

Q : Calculate ABC and BAC.

A : ABC + 110° = 180° (straight line)  ABC = 70°


BAC = 180° – (60° + 70°) = 180° – 130° = 50° (sum of angles of a trianlge = 180°)

Q : Calculate ACD and CDA.

A : ABC is isoceles (AC = BC)  ABC = 35°


ACD = ABC + BAC = 70° (exterior angle)
 CDA = 180° – (ACD + CAD) = 180° – (70° + 40°) = 180° – 110° = 70°

Q : AB = AD, BC = CD, BAD = 100° and DBC = 60°. Calculate ADC and BDC.

BCD is an isoceles triangle (BC = CD)


 BDC = 60°

12
Chapter QA
TRIANGLES-I 3.1

a AD = AB  ABD = ADB

THEORY
In ABD, 2ABD + 100° = 180°
 2ABD = 80°

PRACTICE EXERCISE CLASS EXERCISE


 ABD = 40° = ADB
 ADC = ADB + BDC = 40° + 60° = 100°

Concept Builder 2

1. In the figure, AB = AC. Find ACB, BAC, MCA and PAB P


A

70°
B C M
2. In ABC, B = 95°, ACB can be which of the following? A
a) 90° b) 85°
c) 75° d) 95°

95°
B C

3. A triangle having sides (5, 5, 10) can be formed (True/False)

A
4. State whether the following are True or False:
a. ACD = ACB + ABC
b. ACD > BAC
c. ACD < CBA
B C D

Answer key

a) False b) True c) False 4.


3. False 75° 2.
ACB = 70°, BAC = 40°, MCA = 110°, PAB = 140° 1.

13
CATapult
GEOMETRY

Pythagoras Theorem

Theorem of Pythagoras: In a right-angled triangle, the square A


of the hypotenuse is equal to the sum of the squares of
the other two sides.
(AC)2 = (AB)2 + (BC)2
Conversely, if the square of one side of a triangle is equal
to the sum of the squares of its remaining two sides, then B C
the angle opposite to the first side is a right angle.
e.g.,52 = 32 + 42; 3, 4 and 5 are
The numbers, which satisfy this relation, are called called Pythagorean triplets.
Pythagorean triplets. e.g., 3 × 0.7, 4 × 0.7, 5 × 0.7
Few of the Pythagoras triplets are: i.e. 2.1, 2.8, 3.5 is a Pythagoras
triplet too.
3, 4, 5; 5, 12, 13; 7, 24, 25; 8, 15, 17; 9, 40, 41; 11, 60,
61; 12, 35, 37; 16, 63, 65; 20, 21, 29; 28, 45,53
Any multiple of a Pythagoras triplet will also form a Pythagoras triplet.

A
In an obtuse angled triangle,
AB2 + BC2 < AC2

B C
In an acute angled triangle, A
AB2 + BC2 > AC2

B C

14
Chapter QA
TRIANGLES-I 3.1

Congruency of Triangles

THEORY
One-to-one correspondence between triangles: Pairing of vertices of two triangles is called one-
to-one correspondence between vertices, denoted by ‘’ :

PRACTICE EXERCISE CLASS EXERCISE


If ABC and PQR are any two triangles then there exist six one-to-one correspondences between
their vertices.
A P
(i) ABC  PQR (ii) ABC  QPR
(iii) ABC  PRQ (iv) ABC  QRP
(v) ABC  RPQ (vi) ABC  RQP

B C Q R

For a given correspondence ABC  PQR A P

Side AB  Side PQ ABC  PQR


Side BC  Side QR BAC  QPR
Side AC  Side PR ACB  PRQ
B C Q R

Congruence of triangles
For a given correspondence between two triangles, if the sides and angles of one triangle are
congruent to the corresponding sides and angles of the other triangle, then the two triangles are
said to be congruent.
If ABC  PQR, then
(i) Side AB  Side PQ (iv) ABC  PQR
(ii) Side BC  Side QR (v) BAC  QPR
(iii) Side AC  Side PR (vi) ACB  PRQ

Tests of Congruency
It is not necessary to list all the six conditions i.e., congruence of the three sides and three angles
to prove that the two triangles are congruent. If certain selected conditions are satisfied, then the
others will necessarily follow. These selected conditions are called the tests of congruence.

SSS Test: For a given correspondence between two triangles, A P


if the three sides of one triangle are congruent to the
corresponding three sides of the other triangle, then the two
triangles are congruent.
AB  PQ, AC  PR, BC  QR
 ABC  PQR by SSS test of congruence. B C Q R

15
CATapult
GEOMETRY

SAS Test: For a given correspondence between two triangles, A P


if two sides and the angle included between them are
congruent to the corresponding sides and the included angle
of the other triangle, then the two triangles are congruent.
AB  PQ, BC  QR, ABC  PQR C Q R
B
 ABC  PQR by SAS test of congruence.

ASA Test: For a given correspondence between two triangles, P


A
if two angles and the included side of a triangle are congruent
to the corresponding angles and the included side of the
other triangle, then the two triangles are congruent.
ABC  PQR, BC  QR, ACB  PRQ
B C Q R
 ABC  PQR by ASA test of congruence.

AAS Test: For a given correspondence between two triangles, A P


if two angles and a side other than the included side of a
triangle are congruent to the corresponding angles and a
corresponding side other than the included side of the other
triangle, then the two triangles are congruent.
B C Q R
CAB  RPQ, ABC  PQR, BC  QR
 ABC  PQR by AAS test of congruence.

RHS Test: For a given correspondence between two A P


right-angled triangles, if the hypotenuse and one side of
the right triangle are congruent to the hypotenuse and a
corresponding side of the other right triangle, then the two
right triangles are congruent.
B C Q R
AB  PQ, AC  PR, ABC  PQR
 ABC  PQR by RHS test of congruence.

Similarity of Triangles
For a given correspondence between two triangles, if the corresponding angles are congruent and
their corresponding sides are in proportion, then the two triangles are said to be similar. Similarity
is denoted by the symbol ‘~’. A
If ABC ~ PQR, then, P
ABC  PQR
BAC  QPR
BCA  QRP
AB = BC = AC B C Q R
PQ QR PR

16
Chapter QA
TRIANGLES-I 3.1

Tests for Similarity

THEORY
It is not necessary to list all the conditions for similarity i.e., proportionality of sides and congruence
of angles to prove that two triangles are similar. If certain selected conditions are satisfied, then

PRACTICE EXERCISE CLASS EXERCISE


the others will necessarily follow. These selected conditions are called the tests for similarity.

A
AA Test: For a given correspondence between two triangles, P
if the two angles of one triangle are congruent to the
corresponding two angles of the other triangle, then the
two triangles are similar.
ABC  PQR C B R Q

ACB  PRQ
 ABC ~ PQR by AA test for similarity.

P
SSS Test: For a given correspondence between two triangles,
if the three sides of one triangle are proportional to the 2y A
2x
corresponding three sides of the other triangle, then the y
x
two triangles are similar.
PQ PR RQ Q R B z C
 PQR ~ ABC by SSS test for similarity. 2z
AB = AC = CB

A
SAS Test: For a given correspondence between two
triangles, if the two sides of one triangle are proportional 2x P
to the corresponding two sides of the other triangle and x
the angle included by them are congruent, then the two
triangles are similar. B C Q y R
2y
AB = BC
PQ QR

ABC  PQR
 ABC ~ PQR by SAS test for similarity.

17
CATapult
GEOMETRY

Concept Builder 3

1. Two congruent triangles have all corresponding sides and angles equal. (True/False)

2.

RQ
In the figure, if AB = AC = then, ABC ~ PQR (True/False)
PR PQ BC

3. Which of the following are Pythagorean triplets?


a) 24, 10, 26 b) 333, 444, 555 c) .07, 2.4, 2.5

Answer key

(a) and (b) 3.


True 2. False 1.

18
Chapter QA
TRIANGLES-I 3.1

CLASS EXERCISE

Teaser

4 straight lines (without lifting pen from paper) so that they pass through all the 9 dots below:

19
CATapult
GEOMETRY

Lines and Angles

1. In the adjacent figure, AB // CD and WX // YZ


a) If a = 40°, then how much is h?
b) If e = 55°, then how much is b?
c) If f = 145°, then how much is s?
d) If p = 42°, then how much is q?
e) If k = 70°, then how much is d + v?
f) If r = 130°, then how much is m + t?

2. In the adjacent figure, PQ // RS, PQT = 20° and TSR = 25°.


Find QTS.

3. Find the value of x in the adjoining figure:

4. *In the adjacent figure, PQ // RS and MON = 25°. Find:

a)  LOM

b)  OLN

c)  POM

d)  NOL

20
Chapter QA
TRIANGLES-I 3.1

Triangles: Properties, Classification, Basic Theorems

THEORY
5.

PRACTICE EXERCISE CLASS EXERCISE


Consider a triangle ABC (where a, b and c are the lengths of the sides opposite to A, B
and C respectively)

a) If A = 70° and C = 50°, then find B


1) 50° 2) 60°
3) 70° 4) Cannot be determined

b) If A = 95° and C = 45°, then find the exterior angle at B


1) 40° 2) 70° 3) 140° 4) Cannot be determined

c) If a = 8 and b = 3.5, how many integer values could c take?


1) 5 2) 7 3) 11 4) Cannot be determined

d) If A is 50° and Δ ABC is isosceles, find the exterior angle at B


1) 80° 2) 100° 3) 115° 4) Cannot be determined

e) If A is 100° and Δ ABC is isosceles, find the exterior angle at C


1) 40° 2) 80° 3) 140° 4) Cannot be determined

f) If a = 8 inches, b = 15 inches and C = 90°, then what is the value of c?


1) 13 inches 2) 17 inches
3) 23 inches 4) Cannot be determined

g) If the sides of ∆ ABC are 6, 8 and 10, what kind of triangle will ∆ ABC be?
1) Acute 2) Right 3) Obtuse 4) Cannot be determined

h) If the sides of Δ ABC are 5, 7 and 9, what kind of triangle will ∆ ABC be?
1) Acute 2) Right 3) Obtuse 4) Cannot be determined

i) If A = 30°, B = 80°, and C = 70°, what kind of triangle will ∆ ABC be?
1) Equilateral 2) Isosceles
3) Scalene 4) Cannot be determined

6. Find the hypotenuse of a right angled triangle with perpendicular sides as:
a) 6 and 8 b) 5 and 12 c) 10 and 24
d) 14 and 48 e) *39 and 52 f) * 25 and 60

21
CATapult
GEOMETRY

In any triangle ABC (where a, b and c are the lengths of the sides opposite to A, B and C
respectively) the following properties hold:
• Sum of Angles: The sum of all angles of the triangle is 180° i.e. A + B + C = 180°
• Triangle Inequality: The sum of any two sides exceeds the third (a + b > c, b + c > a,
c + a > b)
• Conversely, the difference of any two sides is less than the third (|a – b| < c, |b – c| <
a, |c – a| < b)
• Exterior Angle Theorem: The exterior angle equals the sum of the remote interior angles
• Theorem of Pythagoras: In a right-angled triangle, the square of the hypotenuse equals
the sum of squares on the other two sides. Hence if A = 90° then a2 = b2 + c2
• Conversely, if a2 = b2 + c2 then A is a right angle (Note: if a2 > b2 + c2 then A is an
obtuse angle while if a2 < b2 + c2 then A is an acute angle)

7. *Consider a triangle ABC (where a, b and c are the lengths of the sides opposite to A, B
and C respectively)

a) If the exterior angles at A and B add up to 240°, find C


1) 40° 2) 60°
3) 120° 4] Cannot be determined

b) If the sides of  ABC are 6, 8 and 9, what kind of triangle will  ABC be?
1) Acute 2) Right
3) Obtuse 4] Cannot be determined

c) If A = 30° and B = 75°, what kind of triangle will  ABC be?


1) Equilateral 2) Isosceles
3) Scalene 4] Cannot be determined

d) If Δ ABC is right-angled and a = 12 and c = 5, how much is b?


1) 13 2) 14
3) 15 4] Cannot be determined

e) If A = 60° and B = C what kind of triangle will  ABC be?


1) Equilateral 2) Isosceles
3) Scalene 4] Cannot be determined

8. * Find the smallest side of a right triangle whose two larger sides are:
1) 40 and 41 2) 50 and 48
3) 36 and 39 4] 45 and 51

22
Chapter QA
TRIANGLES-I 3.1

Triangles: Similarity and Congruence

THEORY
9. In each of the following cases, some
information is given about the sides

PRACTICE EXERCISE CLASS EXERCISE


and/or angles of two triangles PQR
and XYZ. From the given information,
identify whether the two triangles
PQR and XYZ are congruent, similar,
or unrelated.

I P Q R l(PQ) l(PR) l(QR) X Y Z l(XY) l(XZ) l(YZ)


65° 45° 12 65° 45° 12
Relationship:

II P Q R l(PQ) l(PR) l(QR) X Y Z l(XY) l(XZ) l(YZ)


32° 7 6 32° 14 12
Relationship:

III P Q R l(PQ) l(PR) l(QR) X Y Z l(XY) l(XZ) l(YZ)


81° 55° 12.7 81° 55° 12.7
Relationship:

IV P Q R l(PQ) l(PR) l(QR) X Y Z l(XY) l(XZ) l(YZ)


125° 17° 125° 17°
Relationship:

V P Q R l(PQ) l(PR) l(QR) X Y Z l(XY) l(XZ) l(YZ)


87° 12 8 87° 12 18
Relationship:

VI P Q R l(PQ) l(PR) l(QR) X Y Z l(XY) l(XZ) l(YZ)


15 21 24 21 24 15
Relationship:

23
CATapult
GEOMETRY

Similarity: Two triangles are said to be similar if they are the same in shape.
Tests for similarity include:
• SSS: All three pairs of sides should be in proportion
• SAS: Two pairs of sides should be in proportion, and the included angle should be equal
• AA: Two pairs of angles should be equal

Congruence: Two triangles are said to be congruent if they are the same in shape as well as
size. Tests for congruence include:
• SSS: All three pairs of sides should be equal
• SAS: Two sides should be equal and the included angle should be equal
• ASA: Two angles should be equal and the included side should be equal
• SAA: Two angles should be equal and a non-included side should be equal
• Hypotenuse-Side: Hypotenuses and one pair of sides of two right triangles should be equal
Note that there is no ASS test for congruence

VII* P Q R l(PQ) l(PR) l(QR) X Y Z l(XY) l(XZ) l(YZ)


11 13 7 39 33 21
Relationship:

VIII* P Q R l(PQ) l(PR) l(QR) X Y Z l(XY) l(XZ) l(YZ)


48° 6 8 48° 8 6
Relationship:

IX* P Q R l(PQ) l(PR) l(QR) X Y Z l(XY) l(XZ) l(YZ)


33° 44° 6 44° 33° 12
Relationship:

X* P Q R l(PQ) l(PR) l(QR) X Y Z l(XY) l(XZ) l(YZ)


90° 5 13 90° 13 5
Relationship:

10. In PQR, point S lies on side PQ such that P-S-Q. If mSRQ = mQPR, l(PQ) = 12 and l(QR)
= 8, calculate l(QS).

24
Chapter QA
TRIANGLES-I 3.1

Challengers

THEORY
1. In the adjacent figure, all the angles marked with a dot
are equal. Find the measure of angle ABC.

PRACTICE EXERCISE CLASS EXERCISE


1) 30° 2) 36°
3) 45° 4) 48°

2. In the adjacent figure, find m PDQ


1) 45° 2) 36°
3) 25° 4) 30°

3. All possible obtuse-angled triangles with sides of integer length are constructed, such that
two of the sides have length 5 and 12. How many such triangles exist?
1) 5 2) 6 3) 7 4) 8

4. In  LMN, l(LM) = l(LN). An altitude LP is drawn from L to MN.


PQR is an equilateral triangle such that Q lies on LM and R lies
on LP. It is known that S is the midpoint of LN, and that l(QR)
= l(RL). Find MPS.
1) 150° 2) 135°
3) 105° 4) 120°

5. How many triangles with integer sides p, q, r are possible such that p < q < r and the pe-
rimeter of the triangle is 27?
1) 12 2) 18 3) 15 4) 11

25
CATapult
GEOMETRY

PRACTICE EXERCISE

Directions for 1 to 9: Solve as directed.

1. State the measure of the angle (in terms of x), which is the complement of the angle whose
measure is 90° – x.

2. An angle is twice its complement. Find the angle.

3.
C D

E
b a
b a
A O B
In the figure, OE and OC are the bisectors of BOD and AOD respectively,
find mEOC.

4.
R

7c 8a
P Q
b 60° 2b

S T
Find a, b and c from the given figure

5.
P A
70°
T C R

B D

PA || CR and CB || RD. APC = 70°. Find mCRD.

26
Chapter QA
TRIANGLES-I 3.1

6.

THEORY
R S
P Q

CLASS EXERCISE
A B C D
60 90 120

PRACTICE EXERCISE
In the figure, if PS = 360, find PQ, QR and RS.
7.
A

x 2x
B C D
The interior and its adjacent exterior angle of a triangle are in the ratio
1 : 2. What is the sum of the other two angles of the triangle?

8.

A Q B
a a

P
C b
b R D

In the given figure, find mQPR given that AB || CD, PQ and PR are the bisectors of AQR
and CRQ respectively.

9.

From the information given in the figure, find BAC and XAY.

27
CATapult
GEOMETRY

Directions for 10 to 20: Choose the correct alternative.

10. One of the exterior angles of an isosceles triangle is 150°. What is the ratio of its unequal
interior angles?
1) 1 : 4 2) 5 : 2 3) Either (1) or (2) 4) Can’t say

11. In an isosceles triangle, if the vertex angle is increased by 20%, the base angles have to be
reduced by 25% each. The vertex angle, in degrees, is:
1) 80° 2) 90° 3) 100° 4) 110°

12. The two sides of a ABC are 7 and 10. Which of the following cannot be the 3rd side of
ABC?
1) 3 2) 4 3) 5 4) 6

13. The three sides of a PQR are 10, 12 and 20. Which type of triangle it is?
1) Acute Angled Triangle 2) Right Angled Triangle
3) Obtuse Angled Triangle 4) Cannot be determined

14. , and m are two parallel lines 3 cm apart as shown in the figure.
A B C
,

:
10 :
90
m
X O Y
OA is the bisector of BOX and OC is the bisector of BOY. Find ,(BC), if ,(AB) = 5 cm.
1) 3 cm 2) 5 cm 3) 4 cm 4) 20 cm

15. Perimeter of the right angled triangle is 80. Which of the following can be its sides?
1) 18, 25, 37 2) 15, 31, 34
3) 16, 30, 34 4) 11, 34, 35

16. In the given figure, find the measurement of AD?


A D

7 10


x° y°
B 10 C

1) 10 2) 8 3) 7 4) 12

28
Chapter QA
TRIANGLES-I 3.1

17. Which of the following is not necessarily true about this figure?

THEORY
A

D E

CLASS EXERCISE
O

B C
1) ADC and AEB are similar

PRACTICE EXERCISE
2) BOD and EOC are similar
3) AD × AB = AC × AE
4) BD × OE = CE × BO

18.
P

S
V
U
X
W
Q T R

In the given figure, PQR is an equilateral triangle.


Also ,(PS) = ,(QT) = ,(RU). Then, VWX is:
1) an equilateral triangle 2) an isosceles triangle
3) a scalene triangle 4) Cannot be determined

19. Amit walks 1 km towards North 2 km towards East, 2 km towards North and 1 km again
towards East in the given order. Sumit walks double the distance than that initially covered
by Amit towards North, half towards East, then half towards North and double towards East
following the same order. If the ratio of speeds of Amit and Sumit is 3 : 2, find the approx-
imate ratio of their distances from their individual starting points when Sumit has covered
half of the total distance.
1) 13 : 3
2) 3.2 : 5
3) 2 : 3
4) Cannot be determined

29
CATapult
GEOMETRY

20. Which of the following is/are true?

I. If PQ = a, PR = b, QS = c and SR = d, then (a – b)(a + b) = (c + d)(c – d)


II. If PQ = m2 – n2, PR = 2mn and QR = m2 + n2, then QPR = 90°
1] Only I 2] Only II 3] Both I and II 4] Either I or II

Directions for 21 and 22: Solve as directed.

21. In the adjoining figure, there are three parallel lines and two transversals. AC = 5, DF = 12,
BC = 4. Find EG.

22. The exterior angles of ABC are mentioned in the diagram (all angles in degrees). Find the
measure (in degrees) of CBA + CAB.

30
CATapult
GEOMETRY

QA-3.2 TRIANGLES-II THEORY

Properties and Theorems of Triangles


A
Theorem of 45°-45°-90° Triangle: If the angles of a triangle are 45°, 45°

and 90°, then the perpendicular sides are 1 times the hypotenuse. In 45°
2
ABC, AB = BC = 1
2
45°
B C

A
Theorem of 30°-60°-90° Triangle: If the angles of a triangle are 30°,
60°
60° and 90°, then the sides opposite to 30° is half the hypotenuse and

the side opposite to 60° is


3
times the hypotenuse. 30°
2 B C
3
In ABC, AB = 1 AC and BC = AC.
2 2

Ratio of sides: 1 : 3 : 2

Midpoint Theorem: The segment joining the midpoints of any two sides of
D E
a triangle is parallel to the third side and is half of the third side.

If AD = DB, AE = EC, then DE is parallel to BC and DE = 1 BC.


2 B C

Basic Proportionality Theorem: If a line is drawn parallel to one side of

a triangle and intersects the other sides in two distinct points, then the D E
other sides are divided in the same ratio by it.

If DE is parallel to BC, then AD = AE . B C


DB EC

31
CATapult
GEOMETRY

Altitude (height) of a triangle: The perpendicular drawn from the vertex of a triangle to the
opposite side (base) is called an altitude of the triangle.

D F

B C
E
Fig D
A triangle has three altitudes. In ABC, the three altitudes are AE, BF and CD.

Orthocentre: The point of intersection of the three altitudes of a triangle is called the orthocentre.
The angle made by any side at the orthocentre = 180 – the opposite angle to the side. In fig.D,
O is the orthocentre and mBOC = 180 – mA.

Median of a triangle: The line drawn from a vertex of a triangle


to the opposite side such that it bisects the side is called the
median of the triangle. A triangle has three medians. In ABC,
the three medians are AE, BF and DC.

A median divides the triangle into two triangles of equal area.


A(ABE) = A(AEC) = 1 A(ABC)
2

Fig E

Centroid: The point of intersection of the three medians of a triangle is called the centroid. The
centroid divides each median in the ratio 2 : 1.

In fig.E, G is the centroid of the triangle and AG = BG = CG = 2


GE GF GD 1

Interior Angle Bisector Theorem: The angle bisector of any angle A

of a triangle divides the side opposite to the angle in the ratio


E
of the remaining two sides.

In ABC, if BE is the angle bisector of ABC, then, BA = AE


BC CE
B C

32
Chapter QA
TRIANGLES-II 3.2

THEORY
B
Exterior Angle Bisector Theorem: The angle bisector of

any exterior angle of a triangle divides the side opposite

PRACTICE EXERCISE CLASS EXERCISE


to the angle (externally) in the ratio of the remaining two A C
E

sides.

In ABC, DBC is an exterior angle and BE is the exterior angle bisector. Here, BA = AE
BC CE
A

Incentre: The point of intersection of the angle bisectors of E


a triangle is called the incentre. I is the incentre of ABC. D
The distance of the sides from the incentre is called the
inradius. IE, ID and IF are the inradii of ABC. I

B C
F

A
Circumcentre: The point of intersection of the perpendicular bisectors
of the sides of a triangle is called the circumcentre.
The perpendiculat bisector of a side does not necessarily pass through
the oppositte vertex in a triangle in general D O E
O is the circumcentre of ABC. The distance of the vertex from the
circumcentre is called the circumradius. OA, OB and OC are the
circumradii of ABC. B C
F

Note: Circumcentre, incentre, circumradii and inradii will be covered later in detail in Chapter
of Circles

Some interesting facts about geometric centres


1. In an acute angled triangle, the circumcentre and the orthocentre lie within the triangle
2. In a right angled triangle, the circumcentre lies on the hypotenuse and is the midpoint of
the hypotnuse. Also, the orthocentre is the vertex where the right angle is formed.
3. In an obtuse angled triangle, the circumcentre and the orthocentre lie outside the triangle.
4. In a triangle, centroid, orthocentre and circumcentre are collinear. (Euler’s line). Centroid
divides the line joining orthocentre and circumcentre in ratio 2 : 1.

Perimeter of a Triangle

Perimeter: The perimeter is the sum of all the sides of a triangle.


Semiperimeter: Half of the perimeter is called semiperimeter(s).
If a, b and c are the sides of a triangle, then s = a + b + c
2

33
CATapult
GEOMETRY

Appollonius Theorem: The sum of the squares of any two sides of a triangle is A

equal to twice the sum of the square of the median to the third side and the

square of half the third side.

AD is the median of ABC. AB2 + AC2 = 2(AD2 + DC2) B C


D

Solved Examples
Q :
A

B D C

In a triangle ABC, AB = 9, BC = 10, AC = 13. Find the length of median AD. If G is the
centroid, find ,(GA) and ,(GD).
A : By Appollonius theorem,
AB2 + AC2 = 2 × (AD)2 + 2 × (DC)2
 81 + 169 = 2 × (AD)2 + 2 × (5)2
 250 = 2 × (AD)2 + 2 × (5)2
125 = AD2 + 25 ...(Dividing by 2)
 100 = AD2  10 = AD  ,(median) = 10
Since G divides AD in the ratio 2 : 1.

GA = 2 × AD = 2 × 10 = 20 ; GD = 1 × 10 = 10
3 3 3 3 3

Q : In the figure, QR = 10 cm. Find mTMR, mPST, mSQM and ,(ST).


P

33° T
S
78°

Q M R

A : ST || QR and TM || PQ ... (Midpoint theorem)


mPTS = mPRQ = 33° ... (Corresponding angles)
In TMR, TMR = 180 – (78 + 33) = 69°.

34
Chapter QA
TRIANGLES-II 3.2

mSQM = mTMR = 69° ...(Corresponding angles)

THEORY
mPST = mPQM = 69° ...(Corresponding angles)
1 1
ST = 2 × QR = 2 × 10 = 5cm. ...(Midpoint theorem)

PRACTICE EXERCISE CLASS EXERCISE


Q : In the figure AD is the external bisector of EAC intersects BC produced in D. If AB = 8,
AC = 6, BC = 3, find CD.
E
A : AB = BD A
AC DC
Let CD = x
8
8 3+x 6
6= x
8x = 18 + 6x; 2x = 18
 x = 9  CD = 9 B D
3 C

Concept Builder 1

1.
A
45°

45°
B C

In the figure AB = 6, Find the BC, AC

2.
A
60°
3
30°
B C

In the figure, AB = 3. Find BC, AC

35
CATapult
GEOMETRY

3.
A

D E

B C
If DE || BC and D is the mid point of AB, where AB = 10, AE = 6, BC = 8. Find AD, AC, DE

4.
A
6

B E

8
C

BE is the angle bisector of ABC. Find EC


EA

5.

AD is the median of side BC. Find the length of AD.

Answer key

5. AD = 5 4 : 3 4.
3. AD = 5, AC = 12, DE = 4 BC = 3 3 , AC = 6 2.
1. BC = 6, AC = 6 2

36
Chapter QA
TRIANGLES-II 3.2

Properties of Similar Triangles

THEORY
1. If two triangles are similar, Ratio of sides = Ratio of heights P

PRACTICE EXERCISE CLASS EXERCISE


= Ratio of Medians = Ratio of angle bisectors = Ratio of A
inradii = Ratio of circumradii = Ratio of perimeters T
E
If ABC ~ PQR
AB = AD = BE B C R
PQ PS QT D Q S

2. The ratio of the areas of two similar triangles is equal to the ratio of the squares of the
corresponding sides.
If ABC ~ PQR, then
A(3 ABC) (AB) 2 (BC) 2 (AC) 2
= = =
A(3 PQR) (PQ) 2 (QR) 2 (PR) 2

A(3 ABC) AD2 <a AB = AD F


also, = PQ PS
A(3 DQA) PS2

A
3. The triangles on each side of the altitude drawn from the vertex of
the right angle to the hypotenuse are similar to the original triangle
and to each other. ABC ~ ADB ~ BDC
The altitude from the vertex of the right angle to the hypotenuse is
the geometric mean of the segments into which the hypotenuse is D
divided.
In fig.G, (DB)2 = AD × DC
Also, (CB)2 = CA × CD B C
2
(AB) = AD × AC Fig G

37
CATapult
GEOMETRY

SOLVED EXAMPLES

Q : In the figure BAC = DEC D


A
A(TABC) 7.5
find BC, CE, 8
A(TCDE) 6
A : ACB ~ ECD 9
C
a ACB = ECD [Vertically opposite angles]
BAC = CED [Given] B
E
 AC = CB = AB
EC CD ED

 8 = CB = 6
EC 7.5 9

 EC = 12 and CB = 5

A(TABC) AB2 c 2 m2 4
Now, = = 3 =
A(TCDE) ED2 9

Area of a Triangle A
The region enclosed within a triangle is called the area of the triangle, denoted
by the symbol A().
c b
Some of the general formulae used in calculating area of a triangle are as below. F E

1. Area of a triangle = 1 × base × height. Area is measured in square units. B


D C
2
2. Heron’s formula: When three sides a, b, c are given, a

Area = s(s - a)(s - b)(s - c) where semiperimeter, s = a + b + c


2
3. Area = r × s, where r = inradius
s = semiperimeter

4. Area = abc , where R = circumradius


4R

Area of Isosceles Triangle


If ABC, is an isosceles triangle with AB  AC, then the angle bisector of BAC is the perpendicular
bisector of the base BC and is also the median to the base.
Area of an isosceles triangle = a 4c2 - a2 , Where c is the measure of equal sides and a is the
4
third unequal side.
The altitudes on the congruent sides are equal i.e., BE = CF.

38
Chapter QA
TRIANGLES-II 3.2

Area of Equilateral Triangle A

THEORY
For an equilateral triangle,
3 3
height = × side; area = × (side)2
2 4

PRACTICE EXERCISE CLASS EXERCISE


inradius = 1 × height; circumradius = 2 × height
3 3
perimeter = 3 × side
B C

Also, the altitude, median, angle bisector, perpendicular bisector of each base are the same and
the orthocentre, centroid, incentre and circumcentre are the same.

In case of triangles, given the perimeter, an equilateral triangle has maximum area.

Area of Right-angled Triangle


For a right-angled triangle, the median to the hypotenuse = 1 × hypotenuse.
2
The median to the hypotenuse is also the circumradius of the triangle.
Area = 1 × Product of perpendicular sides.
2

Relation between 2 triangles on the basis of height and base

1. The ratio of the areas of two triangles is equal A


to the ratio of the products of base and its cor- P
responding height.
A(TABC) AD×BC
=
A(TPQR) PS ×QR
B D C Q S R

Fig F

2. Triangles of equal heights have areas proportional to their corresponding bases.


A(TABC) BC
In fig.F, if AD = PS, then =
A(TPQR) QR

3. Triangles of equal bases have areas proportional to their corresponding heights.


A(TABC) AD
In fig.F if BC = QR, then = D
A(TPQR) PS A

4. Areas of two triangles having the same base and lying between
the same parallel lines will be equal.
A(ABC) = A(BCD) B C

39
CATapult
GEOMETRY

SOLVED EXAMPLES

Q : The sides of a triangle are 6 cm., 8 cm. and 10 cm. Find the area, inradius and circumradi-
us of the triangle.

A : s = 6 + 8 + 10 = 12
2
Area = s(s - a)(s - b)(s - c) = 12×6× 4×2 = 24 sq.cm.

Let , R be inradius & circumradius of the triangle.


A = r × s,  24 = r × s; 24 = r × 12; r = 2

A = abc , 24 = 6×8× 10 ; 4R × 24 = 6 × 8 × 10
4R 4R

 R = 6×8× 10 = 5 cm.
4 ×24

Q : ABC is right angled at A and AD is the altitude to BC. If ,(AB) = 8 cm and ,(AC) = 15
cm, find ,(BC) and altitude AD. If M is the midpoint of BC, find ,(AM).
A : By the theorem of Pythagoras
,((BC)2) = 82 + 152 = 64 + 225 = 289 cm  ,(BC) = 289 = 17 cm

Area of the triangle = 1 × Product of perpendicular sides


2
= 1 × 8 × 15 = 60 cm2
2
Also area = 1 × BC × AD = 60
2
= 1 × 17 × (AD) = 60  ,(AD) = 120 cm
2 17
Again, AM is the median to the hypotenuse.

 ,(AM) = 1 × hypotenuse = 1 × 17 = 8.5 cm


2 2

Q : Each side of an equilateral triangle is 2 3 cm. Find its height, area, inradius and circumradius.
3 3
A : Height = × side = ×2 3 = 3cm.
2 2
3 3
Area = ×(side)2 = × 4 × 3 = 3 3 cm2.
4 4

Inradius = 1 × height = 1 cm; Circumradius = 2 × height = 2 cm.


3 3

40
Chapter QA
TRIANGLES-II 3.2

Q : Area of ABC = 18 sq. cm. D is the midpoint of BC and E is the midpoint of AB.

THEORY
Find A(BDE).

A :

PRACTICE EXERCISE CLASS EXERCISE


A

B C
D

AD bisects the area of ABC.

A(ADB) = 1 × 18 = 9 sq.cm.
2

DE bisects the area of BDA

A(BDE) = 1 × 9 = 4.5 sq.cm.


2

Concept Builder 2

1. In ABC, AD is the median and M is the centroid and AD = 15. Find AM and DM
A

B D C

2. In ABC, B = 90°, AC = 10. BD is the median to AC. A


Find AD, DC and BD.

B C

Answer key

2. AD = DC = BD = 5cm
1. 10 and 5

41
CATapult
GEOMETRY

CLASS EXERCISE

Teaser

How many triangles are there in the figure below?

42
Chapter QA
TRIANGLES-II 3.2

• Basic Proportionality Theorem: A line parallel to one side of a triangle divides the other

THEORY
two sides in the same ratio.
• Midpoint Theorem: A segment joining the midpoints of two sides of a triangle is parallel
to the third side and half its length.

PRACTICE EXERCISE CLASS EXERCISE


1. In a triangle ABC, AB = 12, AC = 9 and BC = 13. D and E are points on AB and AC respec-
tively such that DE is parallel to BC. If l(AE) = 6, find l(BD)

Triangles : Elements

• Median : It is a line segment joining the vertex of a triangle with the midpoint of the
side opposite to it. Three medians of a triangle are concurrent and their
point of intersection is called 'centroid'.

• Altitude : It is a perpendicular drawn from a vertex of a triangle to the side oppo-


site to it. Three altitudes of a triangle are concurrent and their point of
interesction is called 'orthocentre'.

• Angle bisector : It is a ray originating at a vertex of a triangle passing through the interior
of the triangle and dividing the angle of a triangle into two angles having
equal measure. Three angle bisectors of a triangle are concurrent and their
point of intersection is called 'incentre'.

• Perpendicular
bisector : It is a line perpendicular to the side of a triangle that divides the side of
the triangle into two equal halves. Three perpendicular bisectors of a tri-
angle are concurrent and their point of interesction is called 'circumcentre'.

Triangles: Other useful theorems

2. In the triangle shown in the figure, l(AB) = 6, l(BC) = 7 and l(AC) = 8. Also AD is a perpen-
dicular to BC, AE is an angle bisector of A, and AF is a median of  ABC.
a) Find l(BD)
1) 1 2) 1.5
3) 2 4) 2.5

b) Find l(AD)
1) 25 2) 31.25
3) 37.75 4) 33.75

c) Find l(AF)
1) 41 2) 31.25
3) 37.75 4) 33.75

d) Find l(AE)
1) 36 2) 37.25 3) 39.75 4) 33

43
CATapult
GEOMETRY

In any triangle ABC


• Apollonius’ Theorem: If AD is the median from A to BC,
then
AB2 + AC2 = 2(AD2 + BD2)

• Internal Angle Bisector Theorem: If AD is the internal


angle
bisector of A, then
, (AB) , (DB)
=
, (AC) , (DC)
• External Angle Bisector Theorem: if AD is the external
angle
bisector of A, then
, (AB) , (DB)
=
, (AC) , (DC)

3. * In the adjoining figure, BM and CM are angle bisectors and


m BAC = 70°. Find the measure of BMC.

Special Triangles

4. A triangle VWX has V = 30° and W = 60°. If l(VW) = 10, find l(VX) and l(WX)

5. In the right-angled triangle LMN, l(LM) = l(MN)= 6 cm. Find l(LN) and the area of  LMN.

6. An equilateral triangle has perimeter 12 cm. Find its altitude and area.

44
Chapter QA
TRIANGLES-II 3.2

The 30° - 60° - 90° Triangle: If the angles of a triangle in order are 30°,

THEORY
60° and 90°, then the sides opposite to them will be in the ratio 1 : 3 : 2

The 45° - 45° - 90° Triangle: If the angles of a triangle in order are 45°,

PRACTICE EXERCISE CLASS EXERCISE


45° and 90°, then the sides opposite to them will be in the ratio 1 : 1 :
2 . This is also called an isosceles right-angled triangle.

The Equilateral Triangle: If the side of an equilateral triangle is of length


‘a’ then:
3
The altitude of the triangle is a
2
3 2
The area of the triangle is a
4
1
The circumradius of the triangle is a
3
1
The inradius of the triangle is a
2 3

7. * In Δ FGH, F = 30°, G = 90° and l(FG) = 5.19. What is the area of the triangle?

8. * In Δ PQR, P : Q : R = 3 : 2 : 1. If l(PQ) = 5, what is l(QR)?

9. * Find the ratio of circumradius and inradius of an equilateral triangle of side m.

10. * In a triangle HIJ, H = 30°. If l(HI) = l(IJ) = 12, find the length of HJ.

45
CATapult
GEOMETRY

Triangles: Area

11. Find the area of triangle ABC if


a) AB = 7, AC = 25 and B = 90°
b) AB = 8, AC = 12 and altitude BD = 5
c) AB = 5, BC = 12, AC = 13
d) AB = 13, BC = 14, AC = 15
e) AB = 13, AC = 13 and median AD = 12
f) AB + AC + BC = 24 and inradius r = 2
g) AB = AC = 10, BC = 12 and circumradius R = 6.25

Area of a triangle:
1
Standard Formula: A = × base × height (from the opposite vertex)
2
a+b+c
Heron’s Formula: A = s (s – a) (s – b) (s – c) where s = semi-perimeter = 2
1
For a Right-Angled Triangle: A = × product of perpendicular sides
2
a+b+c
In terms of Inradius: A = r × S where r is the inradius and S the semi-perimeter =
2
abc
In terms of Circumradius: A = where R is the circumradius
4R
Note that two triangles with the same base will have areas proportional to their heights, while
two triangles with the same height will have areas proportional to their bases.
1
In terms of sine of angle: A = ab.sin, where  is the angle between two sides having
2
lengths a and b.

12. ABC is a triangle and AD, BE and CF are the medians


from the vertices A, B and C respectively. If the area of
ABC is 24, then compute the area of:

a)  ABD b)  AOB

c)  AOF d)  AEF

e)  DEF f)  AED

g)  EOF h) o AEOF

46
Chapter QA
TRIANGLES-II 3.2

Areas of similar triangles

THEORY
Areas of similar triangles are proportional to the square of the ratio of their corresponding sides.

PRACTICE EXERCISE CLASS EXERCISE


ABC ~ PQR
2 2 2
A (3 ABC) , (AB) , (AC) , (BC)
 = 2 = 2 = 2
A (3 PQR) , (PQ) , (PR) , (QR)

13. In the following figure, PQ || BC. Also, 2 , (AB) = 3 , (AP).

If A(ΔAPQ) = 40 sq.units, calculate A(ΔABC) and A( PQCB)

14. Suppose ΔABC and ΔPQR are equilateral triangles such that A (ΔABC) : A(ΔPQR) = 1 : 2. If
, (AB) = 10, calculate , (PQ).

47
CATapult
GEOMETRY

Advanced properties of a right angled triangle


• Orthocentre of a right angled riangle is at a vertex where two perpendicular sides of the
triangle meet.
• Circumradius of a right angled triangle is half the hypotenuse. Circumcentre of a right angled
triangle is the midpoint of its hypotenuse.


When perpendicular BD is drawn on hypotenuse AC of right angled triangle ABC,
1) ABD ~ ACB ~ BCD
2) BD2 = AD × DC ; AB2 = AD × AC ; BC2 = AC × DC

15. In right angled PQR, QS is a perpendicular dropped on hypotenuse PR from vertex Q, as


shown. If l(PQ) = 6 and l(QR) = 8

1) Calculate l(PS), L(SR) and l(QS)


2) Calculate circumradii of PQR, PQS and QSR

16. If three sides of a triangle are 8, 15 and 17 cm, calculate the distance of the midpoint of
the hypotenuse from the vertex opposite the hypotenuse.

17. What is the ratio of the circumradius to inradius of an isosceles right angled triangle?

48
Chapter QA
TRIANGLES-II 3.2

Other advanced properties of triangles

THEORY
• In PQR, PS, QS and RS are angle bisectors, then:

PRACTICE EXERCISE CLASS EXERCISE


m+QPR
1) m QSR = 90 +
2
m+PRQ
2) m PSQ = 90 +
2
m+PQR
3) m PSR = 90 +
2

18. I is the incentre of a scalene triangle PQR. m PQR = 50°, m QRP = 60° and m RPQ
= 70°. Calculate
1) m QIR 2) m PIR 3) m PIQ

19. In the figure, AD, BE and CF are altitudes meeting at point O. If mACB = 70° find mAOB

49
CATapult
GEOMETRY

• Centroid divides the medians of triangles in the ratio 1:2


If three medians AP, BQ and CR intersect in point G (which is centroid of ABC),
, (PG) , (QG) , (RG) 1
then = = =
, (GA) , (GB) , (GC) 2
• Three medians divided a triangle into six small triangles having equal area i.e. A(ARG) =
A(BRG) = A(BGP) = A(PGC) = A(CGQ) = A(AGQ) = A(ABC)

20. ABC is a right angled triangle such that l(AB) = 6, l(BC) = 8 and l(AC) = 10. AD, BE and
CF are the medians of the triangle that intersect at point G. Calculate –
1) l(GD) 2) A(AGC)

50
Chapter QA
TRIANGLES-II 3.2

Challengers A

THEORY
1. In the adjacent figure, N and M are midpoints of AC and
BD respectively. BC = 8 units and AD = 6 units. Find length N

PRACTICE EXERCISE CLASS EXERCISE


(MN)
1) 3 2) 3.5
3) 5 4) 5.25
B M D C

2. In triangle ABC, the medians BP and CQ are perpendicular to each other and intersect at R.
If BP = 8 cm and CQ = 12 cm, find the area of triangle ABC.

3. In the given figure ABCD is a quadrilateral with BC = 4 cm and AD = 2 cm. What is the
length of AB (in cm)?
1 4 3 –2
1) 4 – 2)
3 3
+
2 3 1 2 3 –1
3) 4)
2 2

4. In the figure given below, P is a point inside the triangle ABC. Line segments DE, FG and
HI are drawn through P, parallel to the sides AB, BC and
CA respectively. The areas of the three triangles DPG, FPI
and EPH are 9, 16, and 25 respectively. What is the area
of the triangle ABC? (All the areas are in sq cm).
1) 100 2) 144
3) 150 4) 196

5. In the figure,l(AB) = l(BC) = l(CD) = l(DE) = lEF) = l(FG)


= l(GA). If m(DAE) is x°, find the approximate value of
x:
1) 20 2) 25
3) 30 4) 35

51
CATapult
GEOMETRY

PRACTICE EXERCISE

For question 1, use the following information:

Consider the triangle ABC shown in the figure.


AB = AC = 17 and ,(BC) = 16
AD is an altitude drawn from A to BC.
E is the midpoint of AC
F is a point on AC such that ,(DF) = 8.
a) Find ,(DE) and ,(EF)?

b) What will be the perpendicular distance from E to AD and from E to BC?

c) By how much does the altitude AD exceed the altitude of an equilateral triangle with base BC ?

d) What will be the ratio of areas of AED and DEC?

Directions for Questions for 2 to 19: Choose the correct alternative.

2. In PQS, the bisector of QPS meets QS in R. If PQ = 12, PS = 16 and QS = 21, find QR


and SR.
1) 9, 12 2) 8, 22 3) 10, 11 4) 7, 14

3. An altitude of a triangle of area 25 sq. cm. is equal to an altitude of a triangle of area 30


sq. cm. Find the ratio of their corresponding bases.
1) 1 : 2 2) 3 : 2 3) 2 : 3 4) 5 : 6

4. ABC ~ PQR and AB = 2 = . If A(PQR) = 175 sq. cm., find A(ABC).


PQ 5
1) 70 sq. cm. 2) 437.5 sq. cm.
3) 28 sq. cm. 4) 1093.75sq. cm.

52
Chapter QA
TRIANGLES-II 3.2

5.

THEORY
CLASS EXERCISE
In DEF, G is the centroid, DE = 7, EF = 8 and DF = 9. Find GP.

1) 7 2) 7 3) 7 4) 14

PRACTICE EXERCISE
3 2 3

6. In the figure, if NT = 9 and if MB = 10, find MN.


AB 5
M

B
A 95°

85° 65°
N T
1) 5.5 2) 4.5 3) 28 4) 18

7.
G D
x

6
20 F 22
y 12
E

A B C
In the given figure AG, BF and CD are parallel. Then xy = ?

1) 162 2) 360 3) 198 4) 283


11

8.
A

F
E a
b
B D C

If D and E are mid points of BC and AD in the figure then a = ?


b
1) 1 2) 1 3) 2 4) 3
3 2 3 5

53
CATapult
GEOMETRY

9. Find the ratio of perimeters of ABD and ADC.

1) 7 : 9 2) 5 : 7
3) 2 : 3 4) Cannot be determined

10. ABC is as shown below. BD is the angle bisector of ABC. Which of the following can be
concluded?
A

2x
D x

C x B

1) y = 2x 2) y < 2x 3) y = 0 4) None of these

11. A ladder resting along the wall at an angle of 45° slides such that it makes an angle of 30°
with the horizontal, as shown. Find xy.
B
x
B’

30° 45°
A’ A O
y
2-1 1- 2 2+1 1+ 2
1) 2) 3) 4)
3- 2 6- 3 3+ 2 6+ 3

54
Chapter QA
TRIANGLES-II 3.2

12.

THEORY
E

CLASS EXERCISE
D

C F A

PRACTICE EXERCISE
ABC is right angled triangle at B. EF  CA. Find mAEC, if mADB = 55°.
1) 110° 2) 55° 3) 125° 4) 35°

13.
P

G
Q R
X
In PQR, G is the centroid,
PQ = 7.5, QR = 8.0 and PR = 6.5. Find GX ?
1) 1.4 2) 2.5 3) 1.9 4) 1.2

14. Find the maximum area that can be enclosed in a triangle of perimeter 12 cm.
3
1) 4 3 cm2 2) cm2 3) 3 cm2 4) 2 3 cm2
4

15.
A
3
a E
B
2
45°
C 9.66 D

In the given figure BE is parallel to CD. Find area (in sq. units) of the ABE approximately.
1) 12.3 2) 41 3) 6.15 4) 82

55
CATapult
GEOMETRY

16. In ABC, length of side AB is 9 units, that of side BC is 6 units and that of side AC is 5
units. BA is extended to D such that ,(AB) = ,(AD) and BC is extended to E such that ,(BC)
= ,(CE). What is the area of DCE?

1) 20 2 sq. units 2) 15 3 sq. units 3) 40 3 sq. units 4) 18 2 sq. units

17. A flat piece of cardboard is placed on a mirror as shown below:

5 7
3

What is the area of the figure formed by combining the above triangle and its image?
1) 3(2 + 10 ) sq. units 2) 6(2 + 10 ) sq. units
3) 4(2 + 10 ) sq. units 4) None of these

18. Consider an isosceles triangle with integer sides and perimeter 20. Which of the following
cannot be the length of a side of such a triangle?
1) 4 2) 5 3) 6 4) 7

19. In the given triangle, AB = 8 and AC = 16 . Find EF.


3 3

3
1) 2) 1 3) 3 4) 2
2

56
Chapter QA
TRIANGLES-II 3.2

Directions for Questions for 20 and 21: Solve as directed.

THEORY
20. PQR is an equilateral triangle. T and U are points on QR such that TU = 1 QR. Another
3
equilateral triangle STU is drawn with TU as base. If PS intersects QR at V and SV = 3, find

CLASS EXERCISE
PS.

PRACTICE EXERCISE
21. In the given triangle, right-angled at B, the angle bisectors of all the angles intersect at I.
Find m BAI.

Directions for Questions for 22 to 24: Choose the correct alternative.

22. In ABC, which is right-angled at B, BD is drawn perpendicular to AC. BC = a, BD = b and


the area of ABC = R. Which of the following represents the ratio of the areas of ADB and
CDB?
2 2 2 2
1) R4 2) 4R4 3) 4a4 4) a 4
a a R R

23. ABC is right-angled at B. D is the midpoint of BC and E is the midpoint of AB. If CE = 15


and AC = 16, find AD.
1) 10 2) 95 3) 85 4) 9

57
CATapult
GEOMETRY

24 . In the given figure, ABC = ACB = DAB. If AB = 8 and BC = 4, find CD.

1) 8 2) 10 3) 12 4) 16

58
CATapult
GEOMETRY

QA-3.3 QUADRILATERALS & POLYGONS THEORY

Polygons
Definition of terms
A plane figure formed by three or more non-collinear points joined by line segments is called a
polygon.

A polygon with: 3 sides is called a triangle.


4 sides is called a quadrilateral.
5 sides is called a pentagon.
6 sides is called a hexagon.
7 sides is called a heptagon.
8 sides is called an octagon.
9 sides is called a nonagon.
10 sides is called a decagon.

Convex polygon D

A polygon, in which none of the interior angles is more than 180° is called a
E C
convex polygon.

A B

Any straight line drawn cutting a convex polygon passes through only two
sides of the polygon.

E
Concave polygon D
A polygon, in which at least one angle is more than 180°, (i.e., a reflex
> 180° C
angle) is called a concave polygon.
A B

E D
In a concave polygon, it is possible to draw lines passing through
more than two sides.
C

A B

59
CATapult
GEOMETRY

Regular polygon: A convex polygon which has all its sides and angles equal B
is called a regular polygon.
Perimeter: The perimeter of a figure is the sum of the lengths of all its sides. A C
Area: The region enclosed within a figure is called its area.

E D
Diagonal
The segment joining any two non-consecutive vertices is called a diagonal.
In the pentagon ABCDE, AC, AD, BE, BD and CE are the diagonals.
n (n - 3)
Number of diagonals of a polygon with n-sides = 2
n (n - 3) 5×2
In the given figure, number of diagonals in ABCDE = 2 = 2 = 5

Properties of a Polygon
1. Sum of all interior angles of a n-gon (polygon of side n) is given by (2n – 4)90° i.e.,(n –
2)180°.
(n - 2) 180°
Hence, each interior angle of a regular n-gon = n
Example
3-2
Equilateral triangle is a regular polygon, with 3 sides where each angle = 3 × 180 = 60°
2. Sum of an interior angle and its adjacent exterior angle is 180°.
3. Sum of all exterior angles of a polygon is 360°.
360°
4. For a regular polygon, each exterior angle = n
5. Area of a regular polygon
B
1
= 2 × perimeter × perpendicular from centre to any side.
Area of pentagon ABCDE
A C
O
1
= 2 × (AB + BC + CD + DE + EA) × OF

E F D

Types of Polygons

Quadrilateral
As defined earlier, a convex polygon of four sides is called a quadrilateral. It is denoted by the
symbol ‘ ’. The sum of the measures of all angles of a quadrilateral is 360°. Quadrilaterals will
be discussed in detail in the next section of this chapter.

60
Chapter QA
QUADRILATERALS & POLYGONS 3.3

Pentagon: A convex polygon of five sides is called pentagon. In a regular

THEORY
pentagon each interior angle is 108° and each exterior angle is 72°.

PRACTICE EXERCISE CLASS EXERCISE


108°

Hexagon: A convex polygon of six sides is called a hexagon. In a regular


hexagon, each interior angle is 120° and each exterior angle is 60°.

A regular hexagon is a combination of six equal equilateral triangles


A
 Area of regular hexagon a a
3
= 6 × Area of each triangle = 6 × 4 × a2
F VI a B
3 3 3 V
= 2 × a2 = 2 × (side)2 a
a
a I a
a a IV
a
II III
E C

a a
D

SOLVED EXAMPLES

Q : The sum of the measures of the angles of regular polygon is 2160°. How many sides does
it have?

A : Sum of all angles = 90(2n – 4) A

2160 = 90(2n – 4)
D E
2n = 24 + 4
 n = 14
The polygon has 14 sides.
B C

61
CATapult
GEOMETRY

Concept Builder 2

1. Find the number of diagonals of


a) Nonagon b) Decagon

2. Find the interior angle of a


a) Regular Nonagon b) Regular Decagon

3. Sum of all the exterior angles of hexagon is 720° (True/False)

4. Side of a hexagon is 6 units. Find area of the hexagon and length of the diagonal

Answer key
Diagonal = 12 units
Area = 54 3 sq. units 4.
False 3.
b) 144 a) 140 2.
b) 35 a) 27 1.

62
Chapter QA
QUADRILATERALS & POLYGONS 3.3

Quadrilaterals
A

THEORY
1 D
Area of a quadrilateral = 2 × one of the diagonals × sum of the
F
perpendicular drawn to that diagonal from the opposite vertices.
1
A( ABCD) = 2 × AC × (BF + DE)

PRACTICE EXERCISE CLASS EXERCISE


E
C B
Types of Quadrilaterals
The different kinds of quadrilaterals are Square, Rectangle, Rhombus, Parallelogram, Kite and
Trapezium.
Parallelogram
A quadrilateral is called a parallelogram if its opposite sides are parallel.
Properties of a parallelogram D C
1. Opposite sides are parallel and congruent.
2. Opposite angles are congruent.
3. Diagonals bisect each other.
4. Sum of any two adjacent angles is 180°.
A B
5. Each diagonal divides the parallelogram into two
triangles of equal area.
6. Straight lines joining the midpoints of the adjacent sides of any quadrilateral form a paral-
lelogram.
D C
7. Area of parallelogram = base × height.
AB × h
h
8. Sum of the squares of the diagonals is equal to the sum of the
squares of the 4 sides of a parallelogram. q
A B
2 2 2 2 2 2
AC + BD = AB + BC + CD + AD
= 2(AB2 + BC2)

9. Parallelograms that lie on the same base and between the same parallel lines are equal in
area.
10. If a triangle and parallelogram lie on the same base and between the same parallel lines,
1
Area(triangle) = 2 Area(parallelogram)
P Q
11. Diagonals of a parallelogram need not be the angle-bisectors
If any point inside a parallogram is taken and is joined to the
4 vertices the 4 resulting triangles will be such that the sum of
the areas of the opposite triangles is equal. T

i.e., Area of STR + Area of TPQ = Area of TSP + Area of RTQ


S R
= Half the area of the parallelogram

63
CATapult
GEOMETRY

Example
ABCD is a parallelogram. mBAD = 30°. BC = 5 cm and DC = 10 cm. Find the area of the
parallelogram.

Let DP be perpendicular to AB.


ADP is a 30° - 60° - 90° triangle.
1 1
DP = 2 × AD ... (side opposite to 30° is 2 of hypotenuse)
1
= 2 × 5 = 2.5. Area of ABCD = base × height = 2.5 × 10 = 25 sq. cm.

Rectangle
A parallelogram, in which each angle is a right angle is called a rectangle. Obviously, every rectangle
is a parallelogram.
A B
Properties of Rectangles
1. Opposite sides are parallel and congruent.
2. Each angle is equal to 90°.
3. Diagonals are congruent and bisect each other. D C
4. Perimeter = 2(, + b), where , is the length and b is the breadth.
5. Area = , × b
6. Diagonal = ,2 + b2

Note:
• A parallelogram is a rectangle if its diagonals are congruent.
• The quadrilateral formed by the points of intersection of the angle bisectors of a
parallelogram is a rectangle.

Example
The consecutive angles of a parallelogram are (2x + 30)° and (x + 60)°. What type of quadrilateral
is the parallelogram?

2x + 30 + x + 60 = 180
3x + 90 = 180
90
 x = 3 = 30°
 (2x + 30)° = 90° and (x + 60)° = 90°
 The parallelogram is a rectangle.

64
Chapter QA
QUADRILATERALS & POLYGONS 3.3

Rhombus

THEORY
A parallelogram in which all sides are congruent is called a rhombus.
Properties of Rhombus
1. Opposite sides are parallel.

PRACTICE EXERCISE CLASS EXERCISE


2. All sides are congruent.
3. Opposite angles are congruent.
4. Diagonals bisect each other at right angles (but they are not con-
gruent).
1
5. Area = 2 × Product of the diagonals.

Side2 = b 2 one diagonal l + b 2 other diagonal l


1 2
1 2
6.

Note: A parallelogram is a rhombus if its diagonals are perpendicular to each other.


Rhombus is not a regular polygon as its angles are not equal.

SQUARE
A rectangle in which all sides are congruent is called a square. Obviously, every square is a
rhombus, rectangle and parallelogram.

Properties of Square
1. All sides are congruent and opposite sides are parallel.
2. All angles are 90o.
3. The diagonals are congruent and bisect each other at right angles.
4. Perimeter = 4 × side.
1
5. Area = (side)2 = 2 × (diagonal)2

6. Diagonal = 2 × side

Note: A parallelogram is a square if its diagonals are congruent and bisect each other at right
angles.

65
CATapult
GEOMETRY

Example
5
A square swimming pool is surrounded by a footpath 2 m wide. The area of the footpath is 4
times that of the swimming pool. Find the area of the swimming pool.

Let the side of the swimming pool be x m.


Area of the swimming pool = x2
Area of footpath = (x + 4)2 – x2
5 2 2 2
4 x = (x + 4) – x
5x2 = 4(x + 4)2 – 4x2
9x2 = 4(x + 4)2
 3x = 2(x + 4) ...(taking square roots)
3x – 2x = 8  x = 8
 Area of swimming pool = 82 = 64 sq. m.

TRAPEZIUM
A quadrilateral is called a trapezium if two of the opposite sides are parallel but the other two
sides are not parallel.

Properties of Trapezium
1. The segment joining the midpoints of the oblique (non-parallel) sides is called the median of
the trapezium.
1
Median = 2 × sum of the parallel sides D C
1
ABCD is a trapezium, ,(EF) = 2 × [,(DC) + ,(AB)]
E F
1
2. Area of a trapezium = 2 × sum of the parallel sides × height
A B

Note: The trapezium is said to be an isosceles trapezium if the two non-parallel sides are
congruent.

Example
The angles of a ABCD are in the ratio 2 : 3 : 6 : 7. What type of a quadrilateral is it?

2x + 3x + 6x + 7x = 360o
18x = 360o
 x = 20o
 Measures of angles are 40o, 60o, 120o and 140o.
 AB || DC and AD is not parallel to BC
 ABCD is a trapezium.

66
Chapter QA
QUADRILATERALS & POLYGONS 3.3

KITE

THEORY
A quadrilateral is called a kite, if it has two pairs of equal and adjacent sides. B

Properties of a Kite

PRACTICE EXERCISE CLASS EXERCISE


1. Two pairs of adjacent sides are congruent. A C
2. The diagonals intersect at right angles.
3. The longer diagonal bisects the shorter diagonal.
1
4. Area = 2 × product of diagonals
D
Some interesting results
D G C
1. If the midpoints of the adjacent sides of a parallelogram are
joined, a parallelogram is formed. H
F

A E B

2. If the midpoints of the adjacent sides of a rectangle are joined, D G C


it forms a rhombus

H F

A E B

3. If the midpoints of the adjacent sides of a square joined, it forms a D G C


square.

H F

A E B

4. If the midpoints of the adjacent sides of a rhombus are joined, it forms


a rectangle

67
CATapult
GEOMETRY

Summarizing what we have learnt so far:

Properties Parallelogram Rectangle Rhombus Square


Opposite sides are congruent    
All sides are congruent – –  
Opposite sides are parallel    
Opposite angles are congruent    
All angles are 90° –  – 
Diagonals are congruent –  – 
Diagonals bisect each other    
Diagonals bisect each other at right angles – –  
Diagonals bisect vertex angles – –  
Diagonals form four triangles of equal area    
Diagoanls form four congruent triangles – –  

Concept Builder 2

1. Rectangle is a regular polygon (True/False)

D C
2. In parallelogram ABCD, x = 25° (True/False)

x
25°
A B

D
3. Find a) Perimeter of parallelogram ABCD C

b) Area of parallelogram ABCD 4


3

A 6 B

4. State true or false:


a) Every square is a rhombus b) Every kite is a parallelogram
c) Every rhombus is a parallelogram
A

5. In kite ABCD, C
B
BC = 4 units, AD = 8 units
Find area of the kite.

68
Chapter QA
QUADRILATERALS & POLYGONS 3.3

6. In the figure DF = 6, BE = 4 and the area of quadrilateral D

THEORY
ABCD = 40 sq.units. Find the length of the diagonal AC. A
E
6

PRACTICE EXERCISE CLASS EXERCISE


4 F

B
C
Answer key

6) 8 units Area = 16 sq. units 5)


c) True b) False a) True 4)
b) Area = 18 sq. units
a) Perimeter = 20 units 3)
2) False False 1)

SOLVED EXAMPLES

Q : If P and Q are the midpoints of sides AB and BC of square ABCD of area 4 cm2. Find the
area of DPQ.
A : Area of square ABCD = 4 cm2
A 2 D
 Side of square = 2 cm.
1
(DPQ) = A( ABCD) – A(APD + PBQ + DQC)
P 2
= 4 – b 2 × 1 × 2 + 2 × 1 × 1 + 2 × 1 × 2l
1 1 1
1

= 4 – b1 + 2 + 1 l = 1.5 sq. cm.


1
B 1 Q 1 C

Q : ABCD is a parallelogram. E is the midpoint of the diagonal DB. DQ = 10 cm, DB = 16 cm.


Find PQ.
A : EDQ  EBP ... (alternate angles)
A P B
DEQ  BEP ... (ASA test of congruence)
 , (PE) = , (EQ)
E
[ , (EQ)]2 = [ , (DQ)]2 – [ , (DE)]2
= 102 – 82 = 100 – 64 = 36
D Q C
 , (EQ) = 6  , (PQ) = 12 cm.

69
CATapult
GEOMETRY

Q : In ABC, AB = AC = 8, PR and PQ are parallel to lines AC A


and AB respectively. P is the midpoint of BC. Find the perimeter
of PRAQ.
A : R is the midpoint of side AB and Q is the midpoint of side
AC...(midpoint theorem)
1 1 R Q
 RP = 2 AC = 2 AB
1
PQ = 2 AB

PRAQ = 2(RP + PQ) = 2 b 2 AB + 2 AB l = 2AB = B


1 1
Perimeter of C
2 × 8 = 16 P

Q : In parallelogram PQRS, the angle bisectors of P and S intersect at T. Find PTS?


A : In parallelogram PQRS,
let P = R = 2x
S R
S = Q = 2y
now, TPQ = TPS = x T
TSR = TSP = y
In TPS, P Q
x + y + STP = 180° [angles of the triangle] ....(1)
But 2x + 2y = 180° [adjacent angles of parallelogram]
x + y = 90°
S R
Now (1)  90 + STP = 180° y 2x
y
 STP = 90°
T
x 2y
x
P Q

70
Chapter QA
QUADRILATERALS & POLYGONS 3.3

CLASS EXERCISE

Teaser

A farmer has a large L-shaped field which is formed of three square plots as shown below. He
wishes to divide it into four parts such that all four parts are congruent (i.e. of the same shape
and size).

Is it possible to divide the field as required?

71
CATapult
GEOMETRY

Quadrilaterals and polygons

1. State whether the following statements are True or False:

a) The diagonals of a parallelogram bisect each other

b) A quadrilateral whose sides are equal must be a square

c) A quadrilateral whose angles are equal must be a square

d) The diagonals of an isosceles trapezium must be congruent

e) The diagonals of a rectangle bisect the angles at the vertices

f) A quadrilateral whose diagonals intersect at right angles must be a rhombus

g) A quadrilateral whose opposite angles are supplementary must be a rectangle

h) The longer diagonal of a kite is the perpendicular bisector of the shorter diagonal

i) A quadrilateral whose adjacent angles are supplementary must be a parallelogram

j) On joining the midpoints of the sides of any random quadrilateral in order, we get a paral-
lelogram

k) The area of a parallelogram with diagonals 6 units and 8 units is 24 sq. units.

l) * The diagonals of a kite bisect the angles at the vertices

m) * A quadrilateral whose diagonals are equal must be a rectangle

n) * If the diagonals of a parallelogram intersect at right angles, it must be a rhombus

o) * Of all rectangles with a given perimeter, the square has the largest area

p) * The area of a kite with diagonals 10 cm and 14 cm is 70 sq cm

72
Chapter QA
QUADRILATERALS & POLYGONS 3.3

q) * The area of a parallelogram, having one side of length 7 inches, and a perpendicular

THEORY
distance of 6 inches between that side and its opposite side, is 42 square inches

2. In a parallelogram ABCD (with adjacent sides not equal), A B

PRACTICE EXERCISE CLASS EXERCISE


the angle bisectors are drawn. They intersect in points P
P, Q, R and S as shown. What kind of quadrilateral will S Q
PQRS be? R
D C
3. In the given septagon

a) Find the sum of the interior angles

b) Find the sum of the exterior angles

4. If each interior angle of a regular polygon with n sides measures 144°, find n

5. If three angles of a pentagon are right angles, and the other two measure z° each, find the
value of z.

6. Find the area of a regular hexagon of side 6 cm

A
7. Find the sum of measures of all the angles A, B, C, D, E at the
vertices of the figure shown alongside.
B

C
D

Sum of all interior angles of a polygon with n sides = 180 (n – 2)°

180 (n - 2) °
Measure of each interior angle of a regular polygon with n sides = n

Sum of all exterior angles of a polygon with n sides = 360° (independent of n!)

73
CATapult
GEOMETRY

8. * A regular hexagon PQRSTU has a side of 4 cm. Find

a) The perimeter of PQRSTU b) The longest diagonal of PQRSTU

c) The shortest diagonal of PQRSTU d) The area of PQRSTU

9. * If each exterior angle of a regular polygon with n sides measures 20°, find n

10. A regular hexagon ABCDEF has its centre at O. All the non-adjacent vertices are joined to
form the diagonals as shown in the adjacent figure. Identify the shapes formed by joining
the following sets of points:

A B
a) ACE b) ACDF P
U Q
c) AQDT d) BOF
F O C
e) ABEF f) ACF
T R
S
g) PQRSTU h) APDS
E D
i) AUP j) ABDF
J P K
11. In a square JKLM, P and Q are the midpoints of JK and LM as
shown in the adjoining figure. O is the point of intersection of the
diagonals JL and KM, while N is the point of intersection of JO and N
PM. O

If the area of PON = 1, find


M Q L
a) Area of MON

b) Area of POJ

c) Area of KOL

d) Area of JKLM

e) Area of JNM

f) Area of JOQM

74
Chapter QA
QUADRILATERALS & POLYGONS 3.3

12. * Find the area and perimeter of the adjoining figure: 3

THEORY
8

PRACTICE EXERCISE CLASS EXERCISE


3

15

Challengers

1. In the quadrilateral JKLM shown alongside, JL KM. If the area of the J K


triangular regions JOM, JOK and KOL are 12, 9 and 15 respectively,
find the area of MOL. O

1) 18 2) 20 3) 25 4) 21 M
L

G
Data for questions 2 and 3: A regular pentagon ABCDE has sides A
of length 1 cm. Sides AB and DC are extended to meet in a point B
F as shown in the figure. HG is drawn parallel to BC such that it
meets the sides BA and CD (extended) in G and H respectively. E F

C
2. Which of the following is true? D
H

1) m(AEB) > m(GEA) 2) m(DEH) > m(ECD)


3) m(BFC) > m(BEC) 4) None of the above

3. If l (AG) = x, find l (BF)


1
1) 3x 2) x2 3) x 4) x – 1

A B
4. ABCD and OMNP are squares of sides 8cm and 7cm respec- M
tively. O is the centre of square ABCD. PQ = 2cm. Find the
area of the shaded region.
O
1) 16 cm2 2) 14 cm2
3) 18 cm2 4) 12 cm2
N
D Q C

75
CATapult
GEOMETRY

PRACTICE EXERCISE-1

Directions for Questions for 1 to 13: Choose the correct alternative.

1.
A D
20
x 20 20

12
B C
Find the area of the ABCD.
1) 256 sq. units 2) 352 sq. units 3) 269 sq. units 4) 376 sq. units

2. One diagonal of a rhombus is 24 cm and its side is 13 cm. Find the area of the rhombus.
1) 25 sq. cm. 2) 312 sq. cm. 3) 125 sq. cm. 4) 120 sq. cm.

3. Diagonals of a parallelogram are 6 cm and 8 cm respectively. If one side is 5 cm, find its
area.
1) 48 sq. cm. 2) 30 sq. cm. 3) 24 sq. cm. 4) 40 sq. cm.

4.
A B

D C E

C is the midpoint of DE. Area of parallelogram ABCD = 16 sq. cm. Find the area of BCE.
1) 8 sq. cm. 2) 16 sq. cm. 3) 32 sq. cm. 4) 24 sq. cm.

5. In the adjoining figure: If AB, FC and ED are parallel, BC : BD = 2 : 5 and ED = 2AB then
FC : AB = ?
A B

F C
G

E D
1) 5 : 7 2) 7 : 5 3) 3 : 2 4) 2 : 3

76
Chapter QA
QUADRILATERALS & POLYGONS 3.3

6. A square and a rhombus have the same base and the rhombus is inclined at 45°. What is

THEORY
the ratio of area of the square to that of the area of the rhombus?
1) 1 : 1 2) 1 : 2 3) 2 : 1 4) 1 : 2

CLASS EXERCISE
7.
P Q

4 5

PRACTICE EXERCISE
O 10
S R

T
In the above figure PQRS, is a parallelogram with PQ = 10, QR = 5 and PO = 4. Find ST and
area of PQTS.
1) 8, 12 2) 8, 64 3) 6, 48 4) 6, 24

8. The sum of the measures of the angles of regular polygon is 2340°. How many sides does
it have?
1) 13 2) 14 3) 15 4) 16

9.
A 10 B C

E 5

F D
In the given figure, AC is parallel to FD and FB is parallel to DC. Area of BCDE is 238 sq.
units. ,(AB) = 10 units, ,(BE) = 5 units and ,(EF) = 7 units. Find the area of ABE.
1) 40 sq. units 2) 25 sq. units 3) 50 sq. units 4) 35 sq. units

10. Two squares, with side lengths A and B, where A > B are placed together such that the
right side of the square with side A touches the left side of the square with side B and their
bases are collinear. A line is drawn from the bottom left corner of square A to the top right
corner of squareB. What is the area below the line in the square with side A?
1 A2 B
1) 2 (A + B)B sq. units 2) sq. units
2 (A + B)
1 B2 A
3) 2 (A + B)A sq. units 4) sq. units
2 (A + B)

77
CATapult
GEOMETRY

11.
A a
B

F b
E

D C
x
AB || FE || DC. Find the value of x if A(XABEF) = A(XECDF).
2b - a 4a2 - b2 a+b
1) 2b2 - a2 2) 2 3) 4)
a2 2 ab

12. Alternate vertices of a regular octagon are joined to form a quadrilateral. Find the ratio of
the area of the quadrilateral formed to that of the octagon.
1) 1 : 2 2) 1 : 2 2 3) 1 : 4 2 4) 1 : 2

13. XABCD is an isosceles trapezium. Given that EG || DC, AE || BF, FH || GC and mAEG =
60°, find the area of the shaded region.
A 5 B

10 E F
G

D C
H
25 3
1) 25 3 sq. units 2) 4 3 sq. units 3) 75 3 sq. units 4) 75 4 sq. units

Directions for Questions for 14 and 15: Solve as directed.

14. The midpoints of all the sides of a regular hexagon are joined to form another smaller hexa-
gon. What percentage of the area of the outer larger hexagon is the area of the inner smaller
hexagon?
15.

Consider the following figure. PQRS is a parallelogram. Point B is on side RS. Diagonal PR
meets segment QB at M. Ratio RB:BS is 4:1. Calculate the area of triangle RMQ (in square
units) if the area of PQRS is 126 sq. units.

78
Chapter QA
QUADRILATERALS & POLYGONS 3.3

Directions for Questions for 16 to 19: Choose the correct alternative.

THEORY
16. Consider the triangular grid shown below where each side of each of the small triangles is
of one unit length.

CLASS EXERCISE
A B

PRACTICE EXERCISE
C D
What is the area of rectangle ABCD?

21 3
1) sq. units 2) 12 sq. units
4
15 3
3) sq. units 4) 10.5 sq. units
4

, (AP) 1
17. ABCDEF is a regular hexagon. Point P is a point on side AB such that = . What is
, (PB) 3
the ratio of the area of the hexagon to the area of triangle PED?
1) 3 3 : 2 2) 3 : 1 3) 3 : 2 2 4) Cannot be determined

79
CATapult
GEOMETRY

18. In the following diagram, ABCD is a square while points P and Q lie on sides BC and CD
respectively such that APQ is an equilateral triangle. What is the ratio of the area of square
ABCD to that of equilateral triangle APQ?
A B

D Q C

1+ 3 4+2 3 4+ 3 2+ 3
1) 2) 3) 4)
3 3 2 3 3

19. In the given figure containing all rectangles, AB = 2 cm and BC = 1 cm. Also, the ratio of
the length and breadth of rectangles ABCD, EFGH, IJKL, MNOP and QRST is the same. Find
the length IT.
Q M N R

I E F J
A B
D C
L K
H G

T P O S
1) 65 2) 2 61 3) 2 63 4) 2 65

80
Chapter QA
QUADRILATERALS & POLYGONS 3.3

Directions for Question for 20: Solve as directed.

THEORY
20. The given figure consists of 4 concentric squares whose sides differ by 2 units. The area (in
square units) of the shaded region is 64 square units. What is the difference in the areas

CLASS EXERCISE
(in square units) of the largest and smallest squares?

PRACTICE EXERCISE

81
CATapult
GEOMETRY

PRACTICE EXERCISE-2

Directions for questions 1 to 16 : Choose the correct alternative.

1. XABCD is a parallelogram, BD = 26, DC = 17, DA = 11. Find AC.


1) 12 2) 6 3) 7 4) 10.95

2.
D C

A B
XABCD is a trapezium, AB = 3DC. If A(OCD) = 6 sq. cm, find A(OAB).
1) 54 sq. cm. 2) 27 sq. cm. 3) 18 sq. cm. 4) 108 sq. cm.

3. The rectangular courtyard ABCD of side 50 ft and 42 ft encloses a lawn EFGH surrounded
by a 6ft wide gravel path. Find the cost of spreading gravel along the path if gravelling cost
is Rs.10 per sq. ft.
1) Rs.96 2) Rs.480 3) Rs.51.60 4) Rs.9600

4. The perimeter of a regular hexagon is 36 cm. Find its area.


1) 18 3 sq .cm. 2) 36 3 sq. cm. 3) 27 3 sq. cm. 4) 54 3 sq. cm.

5.
A M B
Q
P

L R N

D K C

ABCD is a parallelogram. M, N, K and L are the midpoints of the sides respectively. PQRS
is the quadrilateral formed by the intersections of AK, BL, CM and DN. Determine the area
of XPQRS if the area of quadrilateral ABCD is 3000 square units, and the areas of XAMQP
and XCKSR are 513 and 388 sq. units, respectively.
1) 599 sq. units 2) 799 sq. units
3) 2099 sq. units 4) 2567 sq. units

82
Chapter QA
QUADRILATERALS & POLYGONS 3.3

6.

THEORY
6m E F
A B

10m

CLASS EXERCISE
17m
D 12m C
AB and DC are two opposite sides of a road parallel to each other. To cross the road there
are zebra crossings at DE and CF. Somu who is standing at point A is unable to cross the
road to reach point D. So, he crosses the road from E. E is 6 m from A. If he crosses the

PRACTICE EXERCISE
road from A, he covers 10 m to reach D. Kanu who is standing at point B wants to meet
Somu, so she walks 17 m, to reach point C. Now, if Somu and Kanu are 12 m apart, what
is the distance AB and what is the area between the two zebra crossings?
1) 33 m, 96 m2 2) 15 m, 48 m2
3) 27 m, 54 m2 4) None of these

7.
A E B

D C
In the square ABCD, E and F are two points on sides AB and BC respectively such that DE
= DF, then which of the following is false?
1) EB = BF
2) AE = CF
3) Area of XDEBF = Area of XABCD – 2 Area of ADE.
4) Area of ADE = 1 Area of XABCD.
8

8. P, Q, R, S are mid-points of sides AB, BC, CD, AD respectively. PK, QL, RM and SN are bi-
sectors of P, Q, R and S of XPQRS thus formed by joining the mid-points in cyclic
order. K and N are points on the side RQ and L and M are the points on the side SP. If (RM)
= ,(SR) = 12 units, find mSRM.
1) 60° 2) 120° 3) 90° 4) 30°

9. Two regular polygons have their number of sides in ratio 5:3. The difference between their
angles is 8°. How many sides do the polygons have?
1) 25 and 15 2) 30 and 18 3) 35 and 21 4) 20 and 12

83
CATapult
GEOMETRY

10.
A D

G E

B C F
XABCD and XCFEG are squares. ,(HF) = 5 cm, ,(AH) : ,(HF) = 3 : 1
Which of the following could be the value of A(XABCD)? (All sides are integers)
1) 324 cm2 2) 81 cm2 3) 144 cm2 4) 256 cm2

11.
A E B

q
q
D C
XABCD is a parallelogram, ,(AB) = 8 cm, ,(AD) = 4 cm and ,(EC) = 4 cm. If DE is an bi-
sector of ADC then find ,(ED).
1) (2 + 2 3 )cm 2) 4 3 cm 3) 6 3 cm 4) (4 + 3 )cm

12. A regular polygon of 12 sides is formed by cutting off each corner of a hexagon with side
10. Find the perimeter of the 12 sided polygon.
120 120 3 60 3
1) 2) 3) 4) None of these
2+ 3 2+ 3 2+ 3

13. Two congruent rhombi intersect each other such that exactly one pair of parallel sides of one
rhombus is parallel to that of the other. One of their intersection points is the mid-point of
the intersecting sides of both rhombi. If the rhombus has its side equal to 10 cm and one
of its angles is equal to 60°, then find the common area of the quadrangle formed due to
the intersection.

1) 25 15 cm2 2) 25 3 cm2 3) 75 3 cm2 4) None of these


4 4 4

84
Chapter QA
QUADRILATERALS & POLYGONS 3.3

14.

THEORY
A

B
F

CLASS EXERCISE
C E
G D
In the figure, B and D are midpoints of side AC and side CE respectively and FD || BG. Find
the area of the shaded region if A(ACD) = 40 sq. units

1) 30 sq. units 2) 50 sq. units

PRACTICE EXERCISE
3
3) 35 sq. units 4) Cannot be determined

15. What is the approximate area of the largest regular hexagon that can be inscribed in an
equilateral triangle of each side 9 cm?

1) 12 3 cm2 2) 18 3 cm2 3) 27 3 cm2 4) 39 3 cm2


2 2

16. ABCD and PQRS are two rhombuses. The lengths of their smaller diagonals are in the ratio
3:5. The lengths of the diagonals of ABCD are in the ratio 4:7 and the lengths of the diag-
onals of PQRS are in the ratio 4:5. What is the ratio of the area of ABCD and the area of
PQRS?
1) 13:17 2) 39:47 3) 63:125 4) None of these

Directions for question 17: Solve as directed.

17. ABCD is a rhombus. E, F, G and H are the midpoints of DA, AB, BC and CD respectively. I,
J, K and L are the midpoints of EF, FG, GH and HE respectively. The area of the ELI is 2.25
cm2. What is the area (in cm2) of rhombus ABCD?
A

E I F

D L J B

H K G

85
CATapult
GEOMETRY

Directions for questions 18 to 21 : Choose the correct alternative.


18 . ABCD is a rhombus with DAB = 120° and side ‘x’. If a rectangle PQRS is drawn such that
the diagonal of the rectangle is equal to the longer diagonal of the rhombus, and if the sides
of the rectangle are in the ratio 3 : 1, what will be the smaller side of the rectangle in terms
of x?

1) 2x 2) 3
10 x 3) x 5 4) x
3

19. In the following diagram, ABCDEF is a regular hexagon. What is the ratio of the area of the
shaded region to the area of the hexagon?
B

A C

F D

E
1) 1 : 2 2 2) 1 : 3 3) 1 : 3 2 4) 1 : 2 3

20. In quadrilateral ABCD, BO, CO, AP and DP are the angle bisectors of ABC, BCD, EAD and
ADF respectively. What is the relation between BOC and APD?
E
P
A

O F

D
B

C
1) They are equal. 2) They are complementary.
3) They are supplementary. 4) Nothing conclusive can be said

21. Each of the points P, Q, R and S divides a different side of a rectangle ABCD in the ratio 1
: 2 in such a way that PR and QS intersect at a point inside ABCD. Which of the following
cannot be the ratio of the areas of quadrilateral PQRS and rectangle ABCD?
1) 1 : 2 2) 5 : 9 3) 4 : 9 4) 2 : 3

86
CATapult
GEOMETRY

QA-3.4 CIRCLES - I THEORY

Circle
The set of points in a given plane, which lie at a fixed distance from a fixed point, forms a circle.
The fixed point is called the centre of the circle and the fixed distance is called the radius (r) of
the circle. In the figure, O is the centre of the circle and OR is the radius.
A circle is uniquely determined by three non-collinear points i.e. only one circle passes through
three non-collinear points.

Fig. A

Chord
A chord is a segment whose endpoints lie on the circle. In fig.A, CD is a chord of the circle.
Diameter (d)
The diameter is the chord passing through the centre of the circle. The length of the diameter of
a circle is twice the radius of the circle. In fig.A, PR is a diameter of the circle. The diameter is
the largest chord of a circle.
Circumference (c)
The circumference of a circle is the distance around the circle. It contains 360° at the circle.
The value of the circumference is equal to 2r.
22
c = 2r, where  = or 3.14
7
Semicircle
Half of a circle cut off by a diameter is called the semicircle. In fig.A, PAR is a semicircle.
The measure of a semicircle is 180o.
Arc
An arc is a part of a circle. A minor arc is an arc less than the semicircle and a major arc is an
arc greater than a semicircle. In fig.A, arc CMD is a minor arc and arc CAD is a major arc formed
by the chord CD.

87
CATapult
GEOMETRY

Congruent Circles
Circles with equal radii are called congruent circles.
Concentric Circles
Circles lying in the same plane with a common centre are called concentric
circles.
C1 and C2 are concentric circles with the same centre O and radius r1 and
r2, respectively.
Tangent
Tangent is a line in the plane of a circle and having one and only one point
common with the circle. The common point is called the point of contact. In fig.A, line n is a
tangent to the circle and M is the point of contact.
Secant
A secant is a line, which intersects the circle in two distinct points. In fig.A, line , is a secant,
which intersects the circle in points A and B.
Tangent Circles
Circles lying in the same plane and having one and only one point
in common are called tangent circles.
In the given figure, C is the common point of the circles with
centres O and P.

Properties of Chords
1. The perpendicular from the centre of a circle to a chord of the circle bisects the chord.
If OM  AB, then AM = MB.
Conversely, the line joining the centre of the circle and the
midpoint of a chord is perpendicular to the chord.
If AM = MB, then OM  AB.

2. Equal chords of a circle or congruent circles are equidistant from


the centre.
If AB = CD, then OM = ON.
Conversely, two chords of a circle or congruent circles that are
equidistant from the centre are equal.
If OM = ON, then AB = CD.

88
Chapter QA
CIRCLES-I 3.4

Properties of Tangents

THEORY
1. The tangent at any point of a circle and the radius through
that point are perpendicular to each other. (Tangent Per-
pendicularity Theorem)

PRACTICE EXERCISE CLASS EXERCISE


If O is the centre of the circle, A is the point of contact of
the tangent , , then OA , .
2. Given a point on the circle, there is one and only one tan-
gent to the circle passing through that point.
3. From an exterior point of the circle, two tangents can be drawn onto the circle.

4. The lengths of two tangent segments, from the exterior


point to the circle, are equal. AC = BC.
Now, OA  AC, OB  BC. So here we get a kite OACB

Common Tangents
1. If two circles are such that one lies completely inside the other without
touching each other then there will be no tangent common to these
circles.
2. If the two circles touch internally, then they have only one common
tangent. Distance between their centres = difference of the radii.
Distance between centres = AC – BC.

3. For the two circles with centres A and B, PQ and RS are the
direct common tangents, and CD and EF are the transverse
common tangents. (Only two of both transverse common
tangents and direct common tangents are possible.) Where
r1 and r2 are the radii of the two circles.
Length of direct common tangent

=
2 2
(distance between centres) – (r1 – r2)

Length of transverse common tangent

=
2 2
(distance between centres) – (r1 + r2)

89
CATapult
GEOMETRY

4. If the two circles touch externally, distance between their centres =


sum of their radii. Distance between centres = AC + CB.

Central Angle
An angle in the plane of the circle with its vertex at the centre is called a
central angle. In figure shown above, AOB is a central angle subtended by
arc ACB at the centre.

Measure of an arc
(i) The measure of a semicircle is 180°.
(ii) The measure of a minor arc is equal to the measure of its central angle.
m(arc ADB) = mAOB
(iii) The measure of a major arc = 360° – (measure of corresponding minor
arc)
m(arc ACB) = 360° – m(arc ADB)

Inscribed Angle
An angle with vertex (C) as a point of the arc other than its endpoints (A and
B) and each side of the angle containing one endpoint (A and B) of the arc is called the angle
inscribed in the arc.
ACB is inscribed in the arc ACB.

Intercepted Arc
An arc is said to be intercepted by an angle if each side of the angle
contains an endpoint of the arc, and the arc but for its endpoints,
lies in the interior of the angle.
Arc BD and arc AC are intercepted by AOC.

90
Chapter QA
CIRCLES-I 3.4

Inscribed Angle theorem and its corollaries

THEORY
1. Inscribed Angle theorem
The measure of an inscribed angle is half the measure of its intercepted

PRACTICE EXERCISE CLASS EXERCISE


arc. (Inscribed angle theorem).
1 1
mACB = m(arc ADB) = mAOB
2 2

2. Angles inscribed in the same arc are equal.


mAOB = mACB as they are inscribed in the same arc ADB.

3. The angle subtended by a diameter.


a) A diameter of a circle subtends an obtuse angle at a point in the
interior of the circle. mAEB > 90°.
b) A diameter of a circle subtends an acute angle at a point in the
exterior of the circle. mACB < 90°.
c) A diameter of a circle subtends a right angle at a point on the circle.
mADB = 90°.
Conversely, if a chord subtends a right angle at a point on the circle, then
the chord is a diameter of the circle.
If mADB = 90°, then AB is a diameter of the circle.

4. If a line segment joining two points subtends equal angles at two


other points lying on the same side of the line containing the seg-
ment, then the four points lie on the same circle.
Points A, B, C and D lie on the same circle. i.e., they are concyclic.

5. Equal arcs of a circle or congruent circles have equal chords.


If m(arc AXB) = m(arc CYD), then AB = CD.
Conversely, equal chords of a circle or congruent circles have
their corresponding arcs equal.
If AB = CD, then m(arc AXB) = m(arc CYD)

91
CATapult
GEOMETRY

Alternate-Segment theorem
The angle formed by a tangent and secant is half the intercepted arc.
1
mDBA = m(arc BEA) = mACB
2

Important formulae
Area of a circle: The area of a circle = r2

Length of an arc: The length of the arc is given by the formula:


i
, = 360 × 2r
where,  is the angle subtended at the centre by the arc in degrees. Also, , = r, where  is in
radians.
Sector of a circle: An angle subtended at the centre of the circle divides the
circular region into two parts called the sectors of the circle. The sector bounded
by a minor arc is called a minor sector and the sector bounded by a major arc
is called a major sector.
Sector (O-ACB) is a minor sector and sector (O-ADB) is a major sector.
i
Area of the sector = × 2r 2, where  is the angle subtended at the
360
centre, in degrees.
1
Area of the sector = × r2, where  is in radians.
2
Radius r
Area of the sector = Length of arc × = , ×
2 2

Segment of a circle: A chord of the circle divides the circular region into two
parts called the segments. The part, which contains the centre of the circle, is
called the major segment and the other part is called minor segment.
ACB is a minor segment and ADB is a major segment.

Area of segment = r2 : ri – sini D , where  is the measure of the corresponding arc in degrees.
360 2

Note: sin will be discussed in the chapter on Trigonometry of this module.

Also, Area of minor segment = Area of corresponding sector – Area of triangle formed
 Area of segment ACB = A(O-ACB) – A(AOB)
Area of major segment = Area of corresponding sector + Area of triangle formed
 Area of segment ADB = A(O-ADB) + A(AOB)

92
Chapter QA
CIRCLES-I 3.4

SOLVED EXAMPLES

THEORY
Q : In the figure given alongisde, RS = 12 and radius of the circle is 10.
If C is the centre of the circle, the find PB.

PRACTICE EXERCISE CLASS EXERCISE


A : RP = PS = 6
(CS)2 = (PC)2 + (PS)2
102 = PC2 + 62
(PC)2 = 100 – 36 = 64
PC = 8
 PB = 8 + 10 = 18

Q : In the figure, AB = 16, CD = 12 and OM = 6. Find ON.


1
A : MB = × AB = 8 ... (Perpendicular from the centre of the circle
2
bisects the chord)
, (OB)2 = , (OM)2 + , (MB)2
, (OB)2 = 62 + 82 = 36 + 64 = 100  , (OB) = 10
, (OB) = , (OD) = 10 ... (Radii of the same circle)
, (OD)2 = , (ON)2 + , (ND)2
102 = (ON)2 + 62
 , (ON)2 = 100 – 36 = 64  , (ON) = 8

Q : In the figure, , (AB) = 16 cm, , (BC) = 11 cm and , (CA) = 19 cm.


Find , (BP), , (CQ) and , (AR).
A : Tangents from external points are equal.
Let, , (AR) = , (AQ) = x; , (BR) = , (BP) = y and , (CP) = , (CQ) =
z
x + y = 16 ... (i)
y + z = 11 ... (ii)
z + x = 19 ... (iii)
Adding (i), (ii) and (iii), 2x + 2y + 2z = 46
 x + y + z = 23 ...(iv)

93
CATapult
GEOMETRY

Subtracting (i) from (iv), z = 7


Similarly, x = 12 and y = 4.
, (BP) = 4 cm; , (CQ) = 7 cm; , (AR) = 12 cm

Q : Two parallel chords of a circle are 10 cm and 24 cm in length and they are on the same side
of the centre. If the distance between them is 7 cm, find the
radius of the circle.
A : , (AP) = 12, , (CQ) = 5 (perpendicular from the centre bisects
the chord)
, (OQ)2 = r2 – 25 ... (i)
2 2
, (OP) = r – 144 ... (ii)
, (OQ)2 – , (OP)2 = 119 ... [subtracting (ii) from (i)]
, (OQ + OP)(OQ – OP) = 119
, (OQ + OP) × 7 = 119
 , (OQ) + , (OP) = 17 and , (OQ) – , (OP) = 7
 , (2(OQ)) = 24  , (OQ) = 12
 144 = r2 – 25  169 = r2
 r = 13 cm.

Q : Two tangents of length 21 inches from a point P to the circle with centre O are inclined at an
angle of 60°. Find the circumference of the circle. b r = l
22
7
A : AOP  BOP ... (SSS test of congruence)
 mAPO = mOPB = 30°  OAP is a 30°-60°-90°
triangle.

3
Side opposite to 60° = of hypotenuse
2
3 42
21 = × OP  OP =
2 3
1 1 42 21
OA = × OP = × =
2 2 3 3
22 21
 Circumference = 2 × × = 44 3 inches.
7 3

94
Chapter QA
CIRCLES-I 3.4

Q : In a circle of radius 7 cm a rectangle is inscribed such that its area is maximum.

THEORY
Find the area of the rectangle.
A : The area will be maximum when the rectangle is a square.
The diagonal of the square = Diameter of the circle = 14 cm.

PRACTICE EXERCISE CLASS EXERCISE


2 × side = 14
2
 Area = d n
14 14
 side = = 98 sq. cm.
2 2
Alternatively
1 1
Area of square = × (diagonal)2 = × 14 × 14 = 98 sq. cm.
2 2

Q : The circumference of the front wheel of a wagon is 2 ft and that of the back wheel is 3
ft. Find the distance travelled when the front wheel has made 10 more revolutions than the
back wheel.
A : Distance travelled in one revolution = Circumference of the wheel
If the back wheel makes n revolutions
Distance travelled = 2 × (n + 10) = n × 3
 2(n + 10) = 3n  n = 20
 Distance travelled = 20 × 3 = 60 ft.

Q : Find the radius of the circle which has an arc making an angle of 90° at
the centre of the circle and the area of the segment corresponding to this
arc is 28.5 sq cm. (sin90° = 1)
A : Area of segment

= r2 : D
r × 90 1
360 – 2

28.5 = r2 : D = r2 : r – 2 D = r2 1.14
r 1
4 – 2 4 4
4 × 285 × 100
 r2 =
114 × 10
2 × 285 × 10
= = 2 × 5 × 10 = 100
57
 r = 10 cm.

95
CATapult
GEOMETRY

Q : In the figure, P is the centre of a circle of radius 40 cm.


If m(arc QXR) = 90°, find the area of the shaded region.
A : m(arc QXR) = 90°  mQPR = 90°
1
 A(PQR) = × 40 × 40 = 800 sq. cm.
2
i 90 22
A(P-QXR) = × r2 = × × 40 × 40  1257 sq.cm.
360 360 7
Area of shaded region = A(P-QXR) – A(PQR)
= 1257 – 800 = 457 sq. cm.

Q : Find the length of the arc ACB if radius of the circle is 7 cm.
i
A : , (arc ACB) = × 2r
360
90 22
= × 2 × × 7 = 11 cm.
360 7

96
Chapter QA
CIRCLES-I 3.4

Concept Builder

THEORY
1. The circumference of a circle is 16. Find the radius of the circle.

PRACTICE EXERCISE CLASS EXERCISE


2. In the figure OM = ON = 3 units
If CD = 8 units
Find:
a) ND b) AB
c) AM d) Radius of the circle

3. In the figure (i).


IfAOB = 120°, Find COD, MOB

4. In the figure O is the centre of the circle.


If BAC = 50°, Find BCA

5. In the figure O is the centre of the circle


If ABC = 25°. Find ADC and AOC

97
CATapult
GEOMETRY

6. If CBA = 110° and DCB = 30°


Find CDB, CBD, DBE

7. AB is a chord of a circle of a radius 14 units.


The central angle of the arc is 90°.
Find:
a) length of the arc AEB
b) Area of the sector OAEB
c) the area of minor segment AEB

Answer Key

56 sq. units c)
154 sq. units b)
22 units a) 7.

CDB = 110°, CBD = 40°, DBE = 30° 6.

ADC = 25°, AOC = 50° 5.

BCA = 40° 4.

COD = 120°, MOB = 60° 3.

d) Radius = 5
c) AM = 4
b) AB = 8
a) ND = 4 2.

Radius = 8 units 1.

98
Chapter QA
CIRCLES-I 3.4

CLASS EXERCISE

Teaser

Preeti has a circular birthday cake. While cutting it, she decides to make only 3 straight cuts. What
is the maximum number of pieces she can cut the cake into?

99
CATapult
GEOMETRY

Circles

Circles: Basic elements


• Circle is a bounded geometric figure lying in one plane such that the distance of each point
on the figure from a fixed point called 'centre' is equal.
• The boundary of the circle is called 'Circumference' (denoted by 'C')
• Line segment joining any point on the circle with the centre of the circle is called 'radius'
of the circle (denoted by 'r')
• Line segment joining any two points on the circle is called 'chord' of a circle.
• A chord of the circle passing through the centre of the circle is called 'diameter' of a circle
(denoted by 'd').
• Diameter is the longest chord of a circle.
• Diameter of a circle is two times the radius of the circle.
• Each circle has infinite radii and diameters.
• The ratio of the length of the circumference of a circle to its diameter is a constant, de-
22
noted by ''. Approximately   or 3.14.
7
C = d = 2r
• Area of a circle =  r2
• Any two points on the circumference of a circle divide the circle into two arcs. The longer
arc is called 'major arc' and the shorter arc is called 'minor arc'.
• The angle formed at the centre of the circle by joining the end points of an arc is called
'central angle'.

Central angle of minor arc 'AXB' is .


i
Length of an arc with central angle θ = × 2r
360
i
Area of a sector with central angle θ = × r2
360
• Area of segment AXB = Area of sector OAXB – Area of OAB

100
Chapter QA
CIRCLES-I 3.4

1. Diameter of each wheel of a horse-cart is 1m. If the wheel completes 200 revolutions, what

THEORY
is the distance travelled by the horse-cart?

2. A chariot has two front wheels and two rear wheels. The radius of each rear wheel is 25%
more than that of the front wheel. If the front wheels makes 100 revolutions to travel certain

PRACTICE EXERCISE CLASS EXERCISE


distance, how many revolutions will the rear wheels make to cover the same distance?

3. The circle shown in the figure has a radius of 3.5 cm. O is the
centre of the circle and PQ is a chord such that POQ = 60°.
Find
a) The circumference of the circle
b) The area of the circle
c) The length of the minor arc PRQ
d) The area of the sector O-PRQ
e) * The area of the segment PRQ
22
(take  = and 3 = 1.73 where necessary)
7

4. Find the area and perimeter of the sector of the circle shown below:

Theorems on chords:
Given a circle with centre O and chords PQ and RS as shown:
• If OA  PQ, then l (PA) = l (QA)
The perpendicular from the centre to a chord bisects the chord
• If l (PQ) = l (RS), then l (OA) = l (OB) (where OA and OB are perpendiculars)
Equal chords of a circle are equidistant from the centre
• If l (PQ) = l (RS), then POQ = ROS
Equal chords of a circle subtend equal angles at the centre.
For each of the above theorems, the converse is also true. Hence:
• A line joining the midpoint of the chord to the centre is perpendicular to the chord
• Chords of a circle which are equidistant from the centre are equal in length
• Chords which subtend the same angle at the centre are equal in length

101
CATapult
GEOMETRY

5. A circle has radius 10 and centre O. A chord AB is drawn as shown.


A perpendicular is dropped from O to AB, meeting AB at point Z.
a) If l(OZ) = 6, find l(AB)

b) If l(AB) = 12, find l(OZ)

c) If l(OZ) = 5, find AOB

d) If AO  OB, find l(AB)

6. * In a circle of radius 65, find the distance from the centre of a chord of length:
1) 32 2) 50 3) 66 4) 78 5) 126

7. In the adjacent figure, two parallel chords KL and MN are drawn in a circle with centre O
and radius 25. If l(KL) = 48 and l(MN) = 40, find the distance between the two chords.

Tangent : If a line touches a circle in one point, that line is said to be a 'tangent' to the circle.
Secant : If a line interescts a circle in two points, that line is said to be a 'secant' to the circle.

8. How many tangents can be drawn from point P to circle Q in the following cases?

9. How many common tangents can be drawn to two circles Q and R in the following cases?

102
Chapter QA
CIRCLES-I 3.4

Theorems on tangents:

THEORY
From an external point, two tangent segments can be drawn
Given a circle with centre O and tangents PA and PB:

PRACTICE EXERCISE CLASS EXERCISE


• OA  AP and OB  BP i.e. the tangent to a circle is perpendicular to the radius at the point
of contact
•  OAP  OBP. As a result, mOAP = mOBP and l(PA) = l(PB) i.e. the two tangent seg-
ments drawn to a circle from an external point are equal in length.

10. The circle in the adjacent figure has a radius of 5 and is centred at O. From an external
point P, tangents PA and PB are drawn (A and B are points on the circle)

a) If l(AP) = 7, how much is l(PB)? b) If l(AP) = 12, how much is l(OP)?


c) If l(AP) = 5, how much is APB? d) If l(AP) = 5 3 , how much is APB?
5
e) If l(AP) = , how much is APB?
3
11. Two tangents XY and XZ are drawn from a point X to a circle with centre W. If the length
of XZ is 156 and the length YZ is 120, find the radius of the circle.
1) 65 2) 75 3) 60 4) 72

103
CATapult
GEOMETRY

For the two circles with centres A and B, PQ and RS


are the direct common tangents, and CD and EF are
the transverse common tangents. (Only two of both
transverse common tangents and direct common tangents
are possible.) Where r1 and r2 are the radii of the two
circles.

Length of direct common tangent

=
2 2
(distance between centres) – (r1 – r2)

Length of transverse common tangent

=
2 2
(distance between centres) – (r1 + r2)

12. In the adjacent figure, find the length of the direct common tangent MN to the two circles with
centres O and P and radii ‘R’ and ‘r’, given that the distance between their centres is ‘d’.

13. * In the adjacent figure, find the length of the transverse common tangent ST to the two
circles with centres O and P and radii ‘R’ and ‘r’, given that the distance between their cen-
tres is ‘d’.

14. Two circles with radii 9 and 6 and centres at A and B respectively touch each other externally
as shown in the figure.

104
Chapter QA
CIRCLES-I 3.4

Two direct common tangents PQ and RS are drawn and they meet at C.

THEORY
The transverse common tangent MN is also drawn (where M and N are points on PQ and RS
respectively)
a) Find l(AC)

PRACTICE EXERCISE CLASS EXERCISE


b) Find l(PQ)
c) Find the ratio l(MN) : l(PQ)
d) In the region enclosed by the circle centred at B and the two tangents, another circle of
the largest possible size is drawn. Find its radius.

15. An ant is trying to reach a drop of honey 4 m away. At 1 and 3 m on the straight line joining
the two, there are drops of insect repellant. The and cannot come within 1 m of these. What
is the minimum distance it will have to travel to get the honey?

Inscribed Angle Theorem:


• The measure of an inscribed angle is half that of its intercepted arc
1 1
m(APB) = m(arc AB) = m(AOB)
2 2

Corollaries of Inscribed Angle Theorem:


• Angles inscribed in the same arc are congruent.
• Angle inscribed in a semi-circle is always a right angle.

16.

Calculate m(arc SQ)

17.

If length of chord AB is equal to the radius of the circle, calculate mACB.

105
CATapult
GEOMETRY

18. Arc ADC is a semicircle and DB  AC. If AB = 9 and BC = 4, find DB.

19. Find the length of the common chord of the two circles of radii 6 cm and 8 cm with their
centres 10 cm apart.

20. In the figure, the perpendicular bisectors of AB, BC and AC meet at point O. If mACB =
70° find mAOB

Alternate-Segment theorem
The angle formed by a tangent and secant is half the intercepted arc.
1
mDBA = m(arc BEA) = mACB
2

21. Line DC is a tangent to a circle touching the circle at D, while DB is a secant, intersecting
the circle at points D and B, as shown. Point E is on line DC extended such that D-C-E, as
shown. If mBCE = 110° and mDBC = 35°, calculate mDAB, if point A lies on the cir-
cumference of the circle.

106
Chapter QA
CIRCLES-I 3.4

Challengers

THEORY
1. A circular table is pushed into the corner of a rectangular room in such a way that it just
touches two adjacent walls. An ant crawling on the table rim (on the minor arc between

PRACTICE EXERCISE CLASS EXERCISE


the two points of contact) observes that it is 8 inches from one wall and 1 inches from the
other. Find the diameter of the table.
1) 13 inches 2) 5 inches 3) 26 inches 4) either (1) or (2)

2. Three points P, Q and R are taken such that the lengths PQ, QR and PR are 5, 12 and 13 cm
respectively. Semicircular arcs are constructed using the segments
PQ, QR and PR as diameters (as shown in the figure). Find the
area enclosed by the 3 arcs.
1) 30 cm2 2) 36 cm2
3) 30 cm2 4) 12 – 6 3 cm2

3. Two circles of radii 13 and 15 centered at O and P have a common chord MN of length 24.
Find the distance between O and P.
1) 14 2) 13 3) 15 4) Cannot be determined

4. A regular pentagon ABCDE is inscribed in a circle. Point Z lies somewhere on the minor arc
AB. Find mAZB
1) 120° 2) 126° 3) 144° 4) 150°

5. The two adjacent sides of a quadrilateral are 6 cm and 8 cm long. What is the maximum
possible area of the quadrilateral (in sq cm) if it is inscribed in a circle of radius 5 cm?
1) 45 2) 48 3) 49 4) 50

6. In the figure, a regular hexagon is inscribed in a circle and a smaller circle is inscribed in
the hexagon formed by joining its diagonals. If x is the ratio of the
area of the bigger circle to that of the smaller circle, find x.

107
CATapult
GEOMETRY

PRACTICE EXERCISE
DIRECTIONS for questions 1 to 19: Choose the correct alternative.

1. The diameter of the rear wheel of a cart is 1.5 m. In travelling a certain distance the rear
wheel makes 80 revolutions while the front wheel makes 240 revolutions. Find the diameter
of the front wheel.
1) 0.9 m 2) 0.75m 3) 0.5 m 4) 0.66 m

2.

B, O and P are centres of semicircles AXC, AYB and BZC respectively. AC = 14 cm. Find area
of shaded region.
1) 77sq.cm 2) 154 sq.cm 3) 57.35sq.cm 4) 38.5 sq.cm

3.

Chords PQ and XY intersect at right angles.


Find mQPX + mPQY.
1) 60° 2) 90°
3) 120° 4) Cannot be determined

4. If length of chord AB is 3 times the length of radius of the circle then what will be the
angle inscribed in the arc AB?
1) 60° 2) 120° 3) 60° or 120° 4) None of these

5. A wire of length , cm can form a circle of area 154 cm2 when joined end to end. If a cer-
tain end portion of the wire is cut, it forms a circle of area 38.5 cm2. Find the length of the
portion of the wire which was cut.
1) 11 cm 2) 20 cm 3) 22 cm 4) 44 cm

108
Chapter QA
CIRCLES-I 3.4

6. In a circle with radius 7 units and centre O, the length of a minor arc is 14 units. The area

PRACTICE EXERCISE CLASS EXERCISE THEORY


of the (minor) sector constituted by this arc, is same as the area of sector (O – ACB) of a
circle with centre O of radius 14. Find the length of the arc ACB.
1) 7 units 2) 14 units 3) 18 units 4) None of these

7. Two circles with diameters 16 cm and 12 cm touch each other internally. Find the distance
between their centres.
1) 10 cm 2) 2 cm 3) 14 cm 4) 3 cm

8.

P is the centre of the circle mACB = 65°. Find m(arc AXB).


1) 105° 2) 115° 3) 65° 4) 245°

9.

In the given figure, mEDC = 54°, mDCA = 40°. Find x, y and z.


1) 20°, 27°, 86° 2) 40°, 54°, 86° 3) 20°, 27°, 43° 4) 40°, 54°, 43°

109
CATapult
GEOMETRY

10. For the figures shown below:

OPOQ = OAOB = 15
If R = 9 and r = 2, which one of the following relations is true?
1) PQ = 13.3 2) AD = 10.2 3) PQ > AD 4) All of these

11.

A, B, C and D are wheels of equal diameter. A and B are tied with a rope as in
figure 1. C and D are also tied as in figure 2. AB = CD = 2 × diameter of wheels. Two ants
are moving on the marked path as in figure 1 and 2. Find the ratio of the distance covered by
the ants in one round (i.e. ants travelling paths in figure 1 and 2 respectively). (take  = 3.14)
1) Less than 0.5 2) Between 0.5 and 1
3) Between 1 & 1.5 4) Greater than 1.5

12. Find the approximate area of the shaded region if the two circles are identical and , (O1O2)
= 3 cm. O1 and O2 are the centres of the circles (O1O2 is the diameter of the circle in the
common area of intersection of circles having centres O1 and O2) .

1) 4.5 cm2 2) 6 cm2 3) 4.0 cm2 4) 3.5 cm2

13. Four 50 paise coins are arranged in a way such that each coin touches two others and the
centres of the coins when joined, form a square. What is the area of the square so formed
that is not covered by the coins? (The maximum distance between any two centres is 392
cm.)
1) 84 cm2 2) 42 cm2 3) 14 cm2 4) 28 cm2

110
Chapter QA
CIRCLES-I 3.4

14. A square gymnasium has a circular swimming pool of area 154 sq. units, at it’s centre. A, B,

PRACTICE EXERCISE CLASS EXERCISE THEORY


C and D (the corners) are four entrances to the gym. The shortest distance from any of the
four entrances to the swimming pool is 10 2 – 7 units. Find the area of the gymnasium,
not covered by the swimming pool.

1) 400 sq. units 2) 154 sq. units 3) 246 sq. units 4) None of these

15.

In the figure, square ABCD is divided into 5 equal parts all having same area. ‘T’ is central
part is circular and lines AE, BF, CG, DH lie along the diagonals AC and BD of the square.
If , (AB) = 11 cm. Find radius of the circle of central part.
11 21 5r 5r
1) cm 2) cm 3) cm 4) cm
5r 5r 11 21

16. If a circle is inscribed in a ABCD, then which of the following is true?


1) , (AD) + , (BC) = , (AB) + , (CD) 2) , (AD) + , (AB) = , (CD) + , (CB)
3) , (AC) = , (BD) 4) mD + mB = 180°

17. Four dogs are tied with chains of equal length such that the points where they are tied, when
joined, form a square whose side is twice the length of a chain. If the length of a chain is
, units then what is the area enclosed within the square which cannot be accessed by any
dog?
1) (4 – ) , 2 sq. units 2) (2– 4) , 2 sq. units

3) b l , 2 sq. units
2r
4) None of these
3 –1

111
CATapult
GEOMETRY

18. An ant is standing at the centre O of the circle shown here. It needs to reach point C which
is exactly midway between A and B. The ant can move only along the circumference of the
circle and radii OA and OB. What is the ratio of the shortest distance to the longest distance
that the ant can travel in order to reach point C from point O if it is known that it doesn’t
travel along any stretch twice?

1) 1 : 5 2) 3 : 5 3) 21 : 110 4) 43 : 131

19. A circle has a chord of length 20 cm that subtends an angle of 60° at the centre of the
circle. Another circle is drawn with the chord as diameter. Find the approximate area (in cm2)
of the region common to both the circles.
22
Use  = and 3 = 1.73.
7
1) 366.67 2) 293.67 3) 193.67 4) 155.67

DIRECTIONS for question 20: Solve as directed.

20. In the given figure, BC is a diameter of the circle with centre O. AB and AC are two equal
chords. Also, POQ = 90°. If the area of ABC = 50 square units, find the area of POQ (in
square units).

112
CATapult
GEOMETRY

QA-3.5 CIRCLES - II THEORY

Cyclic Quadrilateral

A quadrilateral is said to be cyclic if all its vertices lie on a


circle. The points lying on a circle are called concyclic.

1. The opposite angles of a cyclic quadrilateral are sup-


plementary.
Conversely, if the opposite angles of a quadrilateral are
supplementary then it is a cyclic quadrilateral.
mA + mC = 180°
mB + mD = 180°

2. An exterior angle of a cyclic quadrilateral is equal to the angle opposite to its adjacent interior
angle. mBCE = mDAB.

3. Ptolemy’s theorem:
In a cyclic quadrilateral the product of diagonals is equal to sum of products its opposite
sides
In the above figure, AC × BD = AB × CD + AD × BC
The converse is also true

4. If a circle touches all the four sides of a quadrilateral the sum of


the two opposite sides is equal to sum of the other two AD + BC
= AB + CD

5. Area of a cyclic quadrilateral =

(s – a) (s – b) (s – c) (s – d) ; where s is the semiperimeter

a+b+c+d
s = where a, b, c and d are the sides of the quadrilateral.
2

113
CATapult
GEOMETRY

6. Area of a cyclic quadrilateral in which a circle can be inscribed


= a × b × c × d , where a, b, c and d are the sides of the quadrilateral.
7. When a square or rectangle is inscribed in a circle, the diagonal of the square/rectangle is
equal to the diameter of the circle.

8. Cyclic Hexagon: A regular hexagon is also a cyclic hexagon. The side


of the regular hexagon is equal to the radius of the circle.

Note: A regular polygon can be inscribed in a circle.

Properties of Secants

A secant is a line, which intersects the circle in two distinct points.

1. If two secants intersect in the exterior of a circle, the angle


so formed is equal to half the difference of the measures of
the arcs intercepted by them.
1
mAOC = [m(arc AC) – m(arc BD)]
2

2. If two secants intersect in the interior of the circle, the


angle so formed is equal to half the sum of the measures
of the arcs intercepted by them.
1
mAOC = [m(arc AC) + m(arc BD)]
2

Note: O need not be the origin.

3. If a tangent and a secant intersect in the exterior of a circle,


the angle so formed is equal to half the difference of the
measure of the arcs intercepted by them.
1
mAOC = [m(arc AC) – m(arc BC)]
2

114
Chapter QA
CIRCLES-II 3.5

4. If AB and CD are two secants intersecting at point O then, AO × BO = CO × DO.

THEORY
PRACTICE EXERCISE CLASS EXERCISE
5. If OC is a tangent and OA is a secant intersecting the circle
at B, then OC2 = OA × OB.
This is called Tangent Secant theorem

115
CATapult
GEOMETRY

Circles and Triangles


Circumcircle: A circle passing through the three vertices of a triangle is called the circumcircle
of the triangle. The centre of the circumcircle is called its circumcentre and the radius is called
the circumradius.
The circumcentre is also the point of intersection of the perpendicular bisectors of the sides of
the triangle.

In the figure, O is the circumcentre and OA, OB and OC are the circumradii of ABC
By definition area of triangle:
abc
A = , where R  circumradius and a, b and c are lengths of sides of a triangle.
4R
abc
 circumradius (R) =
4A
Incircle: A circle touching three sides of a triangle internally is called an incircle of the triangle.
The centre of the incircle is called incentre and the radius is called inradius. The incentre is also
the point of intersection of the angle bisectors of the triangle.
The three sides of the triangle are tangents to the circle. The angle subtended at the centre of
the circle by any side of a triangle is equal to the sum of 90o and half the measure of the angle
opposite to that side.

1
mBOC = mA + 90°
2
The incentre, O divides the bisector of A in the ratio (b + c) : a where a, b and c are the length
of BC, AC and AB.
By definition, area of triangle:
A = r × s, where r = inradius and s = semiperimeter
A
 Inradius (r) =
s

116
Chapter QA
CIRCLES-II 3.5

Incentre of a right angled triangle:

THEORY
PRACTICE EXERCISE CLASS EXERCISE
In right angled triangle ABC, the incircle is touching the triangle at D, E, F on AB, AC, BC respectively.
Now, let AD = x, BD = y and CF = z
then AE = x, BF = y, CE = z
Now, inradius, OD = OF = y

Now AB + BC + AC = 2x + 2y + 2z
AB + BC + AC
 = x + y + z
2
AB + BC + AC
Now, – AC = (x + y + z) – (x + z) = y = inradius
2
 If the measure of the sides of a right angled triangle are given
Inradius = Semiperimeter – hypotenuse

117
CATapult
GEOMETRY

SOLVED EXAMPLES

Q : In a circle with centre ‘O’ shown alongside, OB = 15 and


the radius of the circle is 9.
Find BC × BD.

A : (OB)2 = (OA)2 + (AB)2


152 = 92 + (AB)2
(AB)2 = 225 – 81 = 144
(AB)2 = BC × BD
 BC × BD = 144

Q : In the figure, mAPC = 45°, mAOC = 15°. Find mAQC.


1
A : mAPC = [m(arc AC) + m(arc BD)]
2
2 × 45° = m(arc AC) + m(arc BD)
m(arc AC) + m(arc BD) = 90° ... (i)
1
mAOC = [m(arc AC) – m(arc BD)]
2
2 × 15° = m(arc AC) – m(arc BD)
m(arc AC) – m(arc BD) = 30° ... (ii)
Adding (i) and (ii),
2m(arc AC) = 120°  m(arc AC) 60°  mAQC = 60°

Q : Two concentric circles with centre P have radii 6.5 cm and 3.3 cm.
Through a point A of the larger circle, a tangent is drawn to the
smaller circle touching it at B. Find , (AC).

A : mPBC = 90°
(A tangent is perpendicular to the radius at the point of contact)
 (6.5)2 = (3.3)2 + (BC)2
(BC)2 = (6.5)2 – (3.3)2 = (6.5 + 3.3)(6.5 – 3.3) = 9.8 × 3.2
 BC = 5.6  AC = 2 × 5.6 = 11.2 cm.

118
Chapter QA
CIRCLES-II 3.5

Q : Find the exterior angle at point C i.e., BCE, of a cyclic quadrilateral ABCD inscribed in a

THEORY
circle such that the m(arc BCD) is 240o.
A : m(arc BCD) = 240°

PRACTICE EXERCISE CLASS EXERCISE


1
 mBAD = m(arc BCD) = 120o
2
 mBCE = 120° ... (exterior angle of a cyclic quadrilateral
is equal to the angle opposite to its adjacent interior angle).

Q : In a ABC, the sum of , (AB) and , (AC) is 8 cm. The radius of the incircle drawn in the ABC,
with centre O, is 2 cm and , (AO) is 4 cm. Find the length of the side BC.
A : Let the incircle touch the sides of the triangle AB, AC & BC in points P, Q & R respectively.
Since OP  AB – AP2 + OP2 = AO2 or AP2 + (2)2 = (4)2 or AP2 = 12 or AP = 2 3
 AQ = 2 3 .
Also BP = BR and CQ = CR.
BC = BR + CR = BP + CQ
AB + AC = 8 or AP + PB + AQ + QC = 8
 BP + CQ = 8 – (AP + AQ) = 8 – 4 3
 BC = 8 – 4 3

Q : A regular hexagon is inscribed in a circle of radius 10 cm.


Find the area of the shaded portion. ( = 3.14)
A : Area of shaded region
= Area of circle – Area of hexagon
3 3
= r2 – × (side)2
2
3 3
= 3.14 × 10 × 10 – × 10 × 10
2
3 × 1.73 × 100
= 314 – = 314 – 259.5 = 54.5 sq. cm.
2

Q : In the cyclic quadrilateral ABCD, mBCD = 120°,


m(arc DZC) = 70°, find DAB and m(arc CXB).
A : mDAB = 180° – 120° = 60°
... (opposite angles of a cyclic quadrilateral)
m(arc BCD) = 2 mDAB = 120°
 m(arc CXB) = m(arc BCD) – m(arc DZC) = 120° – 70° = 50°

119
CATapult
GEOMETRY

Concept Builder

1. If PA = 6, PB = 3, CE = 5
Find a) BD b) PE

2. A triangle has sides 24, 10, 26. Find the inradius

3. In the fig D = 95°. Find a) ABE b) EBC

4. Find the product of diagonals of the above cyclic quadrilateral, if EB = 8, BC = 10, CD =


6, ED = 7

Answer Key
4. 118
3. ABE = 95°, EBC = 85°
2. 4 units
PE = 9
1. BD = 9

120
Chapter QA
CIRCLES-II 3.5

CLASS EXERCISE

Teaser

In a clock, a mouse standing at the 12 o’clock position runs in a straight line across the face
of the clock to the 7 o’clock position. Then it runs in a straight line from there to the 3 o’clock
position. What angle must it have turned through?

121
CATapult
GEOMETRY

Cyclic Quadrilateral

• A quadrilateral is said to be cyclic if all its vertices lie on a circle


• Opposite angles of a cyclic quadrilateral are supplementary
• Exterior angle of a cyclic quadrilateral is equal to the remote interior angle
• Area of a cyclic quadrilateral with sides a, b, c and d = (s – a) (s – b) (s – c) (s – d) where
a+b+c+d
s is the semi-perimeter =
2

1. PQRS is a cyclic quadrilateral in a circle with centre O. PQ and SR, when extended, meet at
T. If POR = 100° and PTR = 30°, find QRS

2. PQRS is a cyclic quadrilateral. Point T lies between points P and S on the circle such that
mSTQ = 50°. Calculate mQRS.

3. In the adjoining figure, chord ED is parallel to the diameter AB of the circle. If BCE = 75°,
then what is the value of DEB?

4. In a circle of radius 28 cm, the largest possible hexagon and the largest possible triangle
are inscribed. What is the ratio of their areas?

122
Chapter QA
CIRCLES-II 3.5

Tangential quadrilateral

THEORY
If all the four sides of a quadrilateral are tangent to a circle, that quadrilateral is called 'tangential
quadrilateral'.

PRACTICE EXERCISE CLASS EXERCISE


The sum of the lengths of opposite sides of a tangential quadrilateral is constant.

5. In the following quadrilateral, l(PQ) = 8, l(QR) = 10, l(RS) = 12. Calculate l(PS).

Theorems on Secants:
If two secants AB and CD intersect each other at a point P outside
the circle as shown, then
• l(PA) × l(PB) = l(PC) × l(PD)
1 1
• APC = m(arc AC) – m(arc BD)
2 2

If two secants AB and CD intersect each other at a point P inside


the circle as shown, then
• l(PA) × l(PB) = l(PC) × l(PD)
1 1
• APC = m(arc AC) + m(arc BD)
2 2

123
CATapult
GEOMETRY

6. In the adjoining figure, AB, a diameter of the given circle, is extended to a point C. A tan-
gent CE is drawn, touching the circle at D. The measure of BAD is 36°. Find the following
angles:

a) ADO b) DOA c) DOB d) DBA e) ADB


f) ODC g) BDC h) DBC i) BCD j) ADE

7. In the adjoining figure, secants AB and CD meet at P, a point outside the circle. AD and BC
intersect inside the circle, in point Q.

a) If QCD = 15° and QDC = 25°, find BPD


b) If QCD = 15° and QDC = 25°, find AQC
c) If BQD = 60° and BPD = 12°, find ADC
d) If l(AB) = 5, l(BP) = 4 and l(PD) = 3, find l(DC)
e) If l(AB) = 5 and l(BP) = 4, find the length of a tangent drawn from P

8.

O is the centre of the circle. OP = 7.5 cm, AB = 5 cm and radius OT = 4.5 cm. Find PB.
1) 14 cm. 2) 9 cm. 3) 7 cm. 4) 8 cm.

124
Chapter QA
CIRCLES-II 3.5

9.

THEORY
PRACTICE EXERCISE CLASS EXERCISE
ABC is a right angled triangle. A semicircle with centre O is inscribed inside the triangle as
shown in the figure. Find A( ABOD) if , (AB) = 30 units and mC = 30°.
300
1) 300 sq. units 2) sq. units
3
3) 150 3 sq. units 4) 300 3 sq. units

10.

In ABC, , (AB) = , (AC) = 16 cm. Points D, E and F are the midpoints of sides AB, AC and
BC respectively. Find the ratio of area of ADFE to the area of ABC.
1) 1 : 1 2) 1 : 3 3) 1 : 2 4) 2 : 3

11.

Point P is outside the circle of diameter 14 cm as shown in the figure. m(arc AXB) = 60°,
, (DC) = 10 cm, , (PC) = 24 cm, , (PA) = 12 cm and , (PB) = 15 cm. Find the approximate
value of A( ABCD).
1) 120 cm2 2) 131 cm2 3) 97 cm2 4) Data insufficient

Tangent-secant theorem: If line PC is a tangent and line PB


is a secant to a circle such that P-A-B, then PC2 = PA × PB

125
CATapult
GEOMETRY

12. In the figure, the diameter of the circle is 8 units.


m(arc AB) = m(arc BC). CK is tangent to the circle at C. What is the length of DB?

1) 2 3 units 2) 4 2 units 3) 4 3 units 4) 6 2 units

13.

3
If ADC is 120°, O is the centre of the circle and OE = and AB and BC are tangents
4
to the circle, what is the length of OB?
3
1) 3 2) 3) 1 4) 2
2

14.

In the figure, two circles with centres P and Q touch each other and the common tangent
from O meets the circles at points A and B. The diameters of the circles are in the ratio 5
: 2 and , (OP) = 35 cm. Find the area of the shaded region.
1) 63 10 cm2 2) 53 10 cm2 3) 90 cm2 4) 40 10 cm2

126
Chapter QA
CIRCLES-II 3.5

Circles: Miscellaneous

THEORY
For questions 15 to 17, refer to the figures below:

PRACTICE EXERCISE CLASS EXERCISE


15. Find the area of the shaded region in figure (i). Note that semicircles are drawn with KM,
MN and KN as diameters and KM = MN = 4 cm.

16. Find the area of the shaded region in figure (ii). Note that semicircles are drawn with AB,
BC and AC as diameters and ABC is a right angled triangle with AB = 3 cm, BC = 4 cm.

17. Find the area of the shaded region in figure (iii). Note that semicircles are drawn with OP,
OR and PR as diameters and OP = OR = 4 cm.

18. In the given figure, find the radius of the smallest circle if the two larger circles each have
a radius 1.

19. * A square of side 1 is taken. It is circumscribed by a circle. The circle in turn is circum-
scribed by another square. This process is repeated till there are 5 squares. What is the side
of the largest square?

20. * A hexagon of side 1 is taken. It is circumscribed by a circle. The circle in turn is circum-
scribed by another hexagon. This process is repeated till there are 5 hexagons. What is the
side of the largest hexagon?

21. In the adjacent figure, ABCD is an isosceles trapezium. All the circles are of radius = 1 cm
and touch the sides of the trapezium. Find the height of the trapezium.

127
CATapult
GEOMETRY

Challengers

1. Two circles with centres O and P and radii R and r respec-


tively (where R > r) intersect in points A and B as shown. AB
is extended to a point X. Tangents XY and XZ are drawn to
the two circles. Which of the tangents is greater in length?
1) XY 2) XZ
3) XY = XZ 4) Can’t say

2. Two secants CB and ED of a circle meet in point A outside


the circle. DF is a tangent to the circle at D. BDF = FAD
= 30°. Which of the following statements is necessarily true?

1] G is the centre of the circle


2] BE is perpendicular to CD
3] CEB measures 45°
4] l(AB) + l(BE) = l(AD) + l(DC)

3. In the figure, AB is a chord of the circle with centre O. It


is extended to meet the diameter PQ at R, such that l(OB)
= l(BR). If ARP = x°, and AOP = kx°, then what is the
value of k?
1] 3 2] 4
3] 1 4] 2

4. ABCD is a square of side 4. A semicircle is drawn inside it with diameter CD. A tangent AE
is drawn from A to the semicircle touching it at E, where E is a point inside the square. AE
is extended to F, where F is a point on side BC. If l(AF) = y, what can be said about y?
1] 3.75 < y ≤ 4.25 2] 4.25 < y ≤ 4.75
3] 4.75 < y ≤ 5.25 4] 5.25 < y ≤ 5.75

128
Chapter QA
CIRCLES-II 3.5

5. * In the adjacent figure, ABCD is a square of side 1. A circle is drawn such that it just

THEORY
touches all the 4 sides of the square (at points E, F, G and H). Also, four circular arcs are
drawn with centres A, B, C and D, and radius 1, as shown in the figure.

PRACTICE EXERCISE CLASS EXERCISE


a) Find CJD b) Find JBK
c) Find JDK d) Find IJK
e) Find JIK f) Find AJB
g) Find l(EJ) h) Find l(IK)

6. In the adjacent figure, PQ is a chord of a semi-circle with diameter ST.


If l(ST) = 9 and l(PS) = l(QT) = 3, find l(PQ).

129
CATapult
GEOMETRY

PRACTICE EXERCISE-1
DIRECTIONS for questions 1 to 13: Choose the correct alternative.

1.

In the given figure, m(arc PAS) = m(arc SBQ), m(arc PCT) = m(arc TDR) and mQPR = 50°.
Find mUVR.
1) 100° 2) 85° 3) 115° 4) Data insufficient

2. A, B, C, E and F are points on the circle as given in the figure.


, (FG) = 5 cm, , (AG) = 8 cm, , (BG) = 24 cm, , (ED) = 7 cm and , (CD) = 5 cm. What is
length of BC?

1) 12 cm 2) 24 cm 3) 35 cm 4) 37 cm

3. ABCDEF is a regular hexagon with sides ‘a’ units. M and N are midpoints of AB and ED. What
is the area of the shaded region?

a b 1 – l sq. units a b 1 – l sq. units


3 2 r 2 2 r
1) 2)
2 6 3 6
3) b 3 – r l a sq. units 3a 1 – l sq. units
2b
7 r
4)
2
16 4

130
Chapter QA
CIRCLES-II 3.5

4.

PRACTICE EXERCISE CLASS EXERCISE THEORY


In the figure, ABCD is a square of side ‘a’. What will be the area of shaded region? , (ED)
= , (EC) and F is the center of the circle. (Take 2 = 1.5)
2 2 2 2
a a a a
1) (3 – 2) 2) ( – 2) 3) (5 – 11) 4) (2 – 1)
4 8 40 3

5. In the figure not drawn to scale, ABCD is a cyclic quadrilateral while ABF and CEF are
similar triangles. Which of the following, cannot be the value of mBAC, if mBEC = 30°?

1) 20o 2) 40o 3) 50o 4) 60o

6.

AB and CD are two arcs of the circle with centre O. AC is joined and extended to meet the
line, which is an extension of BD at P. Find mCPD.
1) 5° 2) 10° 3) 15° 4) 30°

131
CATapult
GEOMETRY

7. Two circles with centres at O and P intersect each other at points A and B such that mAOB
= 60°, r1 = 21 units and r2 = 7 3 units. NM and QM are tangents to the smaller circle.
What is the area of the shaded region?

1) (126 3 – 56) sq. units 2) (147 3 – 49) sq. units


3) (126 3 – 49) sq. units 4) (147 3 – 56) sq. units

8.

In rectangle ABCD, M is the midpoint of AB. A square PQRS is inscribed in a circle such that
it touches MD, BC and DC. What is the area of the shaded region?
1) (4 – 8) sq. units 2) (4 – 6) sq. units
3) (8 – 4) sq. units 4) (8 – 6) sq. units

9. If the areas of circles with centres O1 and O2 are equal and that of circles with centres O3
and O4 are equal, then calculate the area of the shaded region if the radius of the circle
with centre O1 = 7 cm.

1) 54.4 cm2 2) 32.5 cm2 3) 58.9 cm2 4) None of these

132
Chapter QA
CIRCLES-II 3.5

10.

PRACTICE EXERCISE CLASS EXERCISE THEORY



 ABCD is an isosceles trapezium with AB || CD. A circle with centre ‘O’ lying at the centre
of the median of the trapezium as shown in the figure. AE and BF are perpendiculars to DC.
If AB = 8 cm, DC = 24 cm and AI = 4 cm. Find the approximate area of the circle enclosed
within the trapezium.
1) 16( – 2) cm2 2) 16( + 2) cm2
3) 16 cm2 4) Data insufficient

11.

In the given figure, not drawn to scale, mAOC = 120°. O is the centre of the circle and
ABCD is a square. Find , (DP) : , (PC).
1) 2 : ^1 + 3 h 2) ^ 3 + 1h : 3
3) 1 : ^ 3 – 1 h 4) 3 : ^1 + 2 h

12.

In the given figure, not drawn to scale, BC is tangent to the circle at D and AD passes through
the centre of the circle such that , (AD) : , (BD) : , (DC) = 3 : 1 : 3. Find the ratio of area
of AEF to that of ABC.
1) 3 : 16 2) 1 : 4 3) 3 : 8 4) 2 : 5

133
CATapult
GEOMETRY

13. Let AB be a diameter of a circle. Point C divides diameter AB in the ratio 1 : 2. Let DE be
another diameter of the circle such that DE is perpendicular to AB. What is the ratio of the
area of DCE to area of ABD?
1) 1 : 2 2) 1 : 4 3) 1 : 6 4) 1 : 3

14. In the given figure, the two circles touch each other internally such that the diameter of the
inner circle is two-thirds the diameter of the outer circle. CD, PA and PB are tangents to the
inner circle. What is the ratio of the areas of PMN and the smaller circle?

r 3 1
1) 3r 2) 3) 4)
3 r 3r
15. AB = 12 cm is a diameter of a circle with centre O. Another circle touches the given circle
internally at B and passes through O. If C is a point on the larger circle such that AC is a
tangent to the smaller circle at D, then find AC (in cm).
1) 6 2 2) 9 3) 8 2 4) 3 15

16. The largest possible square is drawn inside a circle of radius 6 units. An equilateral trian-
gle is drawn inside the square with one of the sides of the square as its base. The largest
possible circle is drawn inside this equilateral triangle. What is the distance between the
centres of the two circles?
1) 2 + 1 2) 3 3) 3 – 1 4) 2

134
Chapter QA
CIRCLES-II 3.5

17. In the following diagram, two tangent segments PQ and PR of length 12 cm each are drawn

PRACTICE EXERCISE CLASS EXERCISE THEORY


from an external point P to a circle of radius 9 cm. O is the center of the circle. Segment
MN, which is perpendicular to OP, touches the circle at T, as shown. What is ,(MT)?

1) 7.5 cm 2) 4.5 cm
3) 6 cm 4) Cannot be determined

18. ABCD and EFGH are two rectangles such that the points A, B, C, D, E, F, G and H all lie on
a circle. The ratio of the areas of rectangles ABCD and EFGH is 75 : 42. If AB = 15 and
EH = 7, find the area (in square units) of the circle that passes through A, B, C, D, E, F, G
and H.

625
1)  2) 100  3) 144  4) 625 
4

135
CATapult
GEOMETRY

19. In the following diagram, point E is on the extended side DC of the quadrilateral such that
mDAB = mBCE = 76°. Point P lies in the interior of the quadrilateral such that ,(PA) =
,(PB) = ,(PD) = 4 cm. What is ,(PC)?
B

E
D C

1) 4 3 cm
2) 2 cm
3) 4 cm
4) More information is needed to answer this question

20. In the given figure, the angle bisectors of all the three angles of the equilateral triangle ABC
meet at D. A circle is drawn with CD as diameter. Find the area (in square units) of the
shaded region if the length of each side of ABC = 14 3 units.

c m 2) 49 b 3l 3) 49 b l 4) 49 c 2r – m
49 2r 3 2r 2r 1 3
1)
2 3 – 2 3 – 3 – 2 3 2

136
Chapter QA
CIRCLES-II 3.5

PRACTICE EXERCISE-2

DIRECTIONS for questions 1 to 3: Choose the correct alternative.

1. In the following figure, five circles are adjacent to one another and have lines L1 and L2 as
common tangents. If the radius of the largest circle is 18 units and that of the smallest circle
is 8 units, what is the radius of the middle circle?

1) 11 units 2) 11.5 units 3) 12 units 4) 12.5 units

2. Find the ratio of the area of the largest triangle that can be inscribed in a circle of radius 7
cm to the area of the largest triangle that can be inscribed in a semicircle of radius 7 cm.
1] 3 : 4 2] 3 : 4 3] 3 3 : 2 4] None of these

3. AB is the line joining the points of intersection of a semicircle and a circle as shown in the
figure. , (AB) = m. Find the area of a circle with radius AB.

1) 40 sq. units 2) 80 sq. units 3) 120 sq. units 4) 160 sq. units

137
CATapult
GEOMETRY

DIRECTIONS for questions 4 and 5: Refer to the data below and answer the questions that
follow.

In a village, there are two circular and concentric ring roads as shown. The inner ring road has a
circumference equal to half that of the outer ring road. The village has four headmen (HMs) and
four chiefs (Ls). The positions of their houses are marked on the figure.
A vehicle can move at the rate of 40 kmph on the inner ring road, 60 kmph on the outer ring
road and 30 5 kmph on the chord roads.

4. I am in L3’s house and want to go to HM1’s house. If I take the route from L3 to L2 along
the outer ring road and then take the chord road from L2 to HM1, I will take a total of 3
hours to reach my destination. What is the radius of the outer ring road in km?
1) 30 km 2) 40 km 3) 60 km 4) 80 km 5) 120 km

5. A car wants to reach HM2 from L1 using first, the chord L1HM4 and then, the inner ring road.
How many minutes will it take? (Use the data from the previous question)
1) 120 min 2) 90 min 3) 180 min
4) 210 min 5) None of these

DIRECTIONS for questions 6 to 20: Choose the correct alternative.

6. Mohanlal has a field having dimensions as given in the figure. He has four horses and four
ropes of lengths 12, 16, 24, 28 metres. What can be the minimum area which will remain
ungrazed, if they are tied to the corners A, B, C and D using the four ropes such that no
two horses ever come in contact?

1) (5678 – 520) m2 2) (5678 – 480) m2


3) (5678 – 314) m2 4) (5678 – 494) m2

138
Chapter QA
CIRCLES-II 3.5

7. A circle is inscribed in a given square and another circle is circumscribed about the square.

PRACTICE EXERCISE CLASS EXERCISE THEORY


What is the ratio of the area of the inscribed circle to that of the circumscribed circle?
1) 2 : 3 2) 3 : 4 3) 1 : 4 4) 1 : 2

8. A one-rupee coin is placed on a table. The maximum number of similar one-rupee coins
which can be placed on the table, around it, with each one of them touching it and only two
others is _____.
1) 8 2) 6 3) 10 4) 4

9. The line AB is 6 m in length and is tangent to the inner one of the two concentric circles
at point C. It is known that the radii of the two circles are integers. The radius of the outer
is:

1) 5 m 2) 4 m 3) 6 m 4) 3 m

10. PQRS is a square. SR is a tangent at point S to the circle with centre O and TR = OS. Then,
the ratio of area of the circle to the area of the square is _____.

r 11 3 7
1) 2) 3) 4)
3 7 r 11

139
CATapult
GEOMETRY

11. In the given figure, AB is diameter of the circle and the points C and D are on the circum-
ference such that CAD = 30° and CBA = 70°.

What is the measure of ACD in degrees?


1) 40 2) 50 3) 30 4) 90

12. From a circular sheet of paper of radius 20 cm, four circles, each of radius 5 cm are cut out
What is the ratio of the areas of the uncut to the cut portion of the sheet?
1) 1 : 3 2) 4 : 1 3) 3 : 1 4) 4 : 3

13. The figure shows a circle of diameter AB and radius 6.5 cm. If chord CA is 5 cm long, find
the area of the triangle ABC.

1) 60 sq. cm 2) 30 sq. cm 3) 40 sq. cm 4) 52 sq. cm

14. The figure shows the rectangle ABCD with a semi-circle and a circle inscribed inside it. What
is the ratio of the area of the circle to that of the semi-circle?

^ 2 – 1h
2
2 2
1) ( 2 – 1) 2) 2( 2 – 1) 3) 4) None of these
2

140
Chapter QA
CIRCLES-II 3.5

15. In the adjoining figure, points A, B, C and D lie on the circle. AD = 24 and BC = 12. What

PRACTICE EXERCISE CLASS EXERCISE THEORY


is the ratio of the area of the triangle CBE to that of the triangle ADE.

1) 1 : 4 2) 1 : 2 3) 1 : 3 4) Insufficient data

16. AB is the diameter of the given circle, while points C and D lie on the circumference as
shown. If AB is 15 cm, AC is 12 cm and BD is 9 cm, find the area of the quadrilateral ACBD.

1) 54 sq. cm 2) 216 sq. cm 3) 162 sq. cm 4) None of these

17. The sum of the areas of two circles which touch each other externally is 153. If the sum
of their radii is 15, find the ratio of the larger to the smaller radius.
1) 4 : 1 2) 2 : 1 3) 3 : 1 4) None of these

18. Three circles, each of radius 20 cm and have their centres at P, Q and R. Further, AB = 5
cm, CD = 10 cm and EF = 12 cm. What is the perimeter of the triangle PQR?

1) 120 cm 2) 66 cm 3) 93 cm 4) 87 cm

141
CATapult
GEOMETRY

19. Four identical coins are placed in a square. For each coin the ratio of numerical value of
area (in sq. units) to numerical value of circumference (in units) is the same as the ratio of
numerical value of circumference (in units) to numerical value of area (in sq. units). Find the
area of the square that is not covered by the coins.

4) 16 b 4 – l
r
1) 16( – 2) 2) 16( – 2) 3) 16(4 – )
2

20. The figure below shows two concentric circles with centre O. PQRS is a square, inscribed in
the outer circle. It also circumscribes the inner circle, touching it at points B, C, D and A.
What is the ratio of the perimeter of the outer circle to that of polygon XABCD?

r 3r r
1) 2) 3) 4) 
4 2 2

142
CATapult
GEOMETRY

QA-3.6 3-DIMENSIONAL FIGURES THEORY


AND MENSURATION

Definition of Terms
Mensuration deals with the measurement of areas and volumes of plane and solid figures.

The areas of plane figures have been covered in previous chapters.

Solids: Solids are three-dimensional objects, bound by one or more surfaces. When plane surfaces
bound a solid, they are called its faces. The lines of intersection of adjacent faces are called its
edges. The points of intersection of the edges are called vertices.

Euler’s Formula
For any regular solid,

Number of faces + Number of vertices = Number of edges + 2

This formula is called Euler’s formula.

However this does not hold for a solid with curved surface.

Different solids and their areas and volumes

For solids, two different types of areas are defined:


1. Lateral Surface Area (L.S.A) or Curved Surface Area (C.S.A) and
2. Total Surface Area (T.S.A).
Lateral surface area is the area of the vertical faces of the solid. In solids, where the lateral
surface is curved, the lateral surface is referred to as the Curved Surface Area.
Total surface Area = Lateral/Curved S.A + Area of top face + Area of Bottom face

Volume of a solid figure is the amount of space enclosed by its bounding surfaces. Volume is
measured in cubic units.

Note: • Weight = Volume × Density.


• 1cu. m = 100 × 100 × 100 = 106 cu. cm.
• If a solid is melted into another solid, volume remains the same.

143
CATapult
GEOMETRY

Rectangular Parallelopiped or CUBOID: A rectangular parallelopiped is bound by 6 rectangular


faces. The opposite faces of a rectangular solid are equal rectangles lying in parallel planes.

A cuboid has,
8 vertices i.e., A, B, C, D, E, F, G and H.
12 edges i.e., AE, AB, AD, BF, BC, FE, FG, EH, DH, DC, CG and GH.
2 horizontal faces; AEFB, DHGC.
4 vertical faces; AEHD, ABCD, BFGC and EFGH.
4 body diagonals; DF, EC, AG and HB.
DC is the length of the cuboid ( , ).
DH is the breadth of the cuboid (b).
DA is the height of the cuboid (h).

Surface area of vertical faces of cuboid = perimeter of base × height = 2(b + , ) h


T.S.A of cuboid = L.S..A + 2 × Base Area = 2(b + , )h + 2 × , × b = 2[ , × b + b × h + h × , ]
Volume of cuboid = Area of base × height = , × b × h = , bh
Length of base diagonal, AF =
2 2
, +b
Length of body/ longest diagonal, AG =
2 2 2
, +b +h

CUBE: A cube is a special case of a parallelopiped in which the length, breadth and height are
equal i.e., it is bound by six square faces.

Surface area of vertical faces of a cube = 4 , 2


Total surface area of a cube = 6 , 2

Volume of a cube = Area of base × height


= ,2 × , = ,3
Body diagonal = 3 ,

Right PRISM: A prism is a solid, whose top and bottom faces are identical polygons and parallel.
Their vertical faces are rectangular.
A prism is said to be triangular prism, quadrilateral prism pentagonal prism, hexagonal prism,
octagonal prism according to the number of sides of the polygon that form the base.
In a prism with a base of n sides, Number of vertices = 2n, Number of faces = n + 2.
Cube and Cuboids are prisms with a square and rectangle as a base respectively

144
Chapter QA
3-DIMENSIONAL FIGURES AND MENSURATION 3.6

THEORY
PRACTICE EXERCISE CLASS EXERCISE
Triangular Prism Pentagonal Prism Hexagonal Prism

In the figure above, OO’ is the perpendiclar height (h) of the prism

Surface area of vertical faces of a prism = perimeter of base × height (h)


Total surface area of a prism = perimeter of base × height + 2 × area of base
Volume of a prism = area of base × height.

Example
In a equilateral triangular prism, the side of base is equal to 6 units. The height of the prism is
8 units. Find the volume, L.S.A. and T.S.A. of the prism.
Lateral Surface Area = Base perimeter × height
= (3 × 6) × 8
= 144 sq. units
Total Surface Area = L.S.A + 2 × Area of base
3
= 144 + 2 × × 62
4
= 144 + 18 3 sq.units
Volume = Area of base × height
= 18 3 × 8
= 144 3 cubic. units

Right Circular CYLINDER: The base and upper face of a right circular cylinder are equal circular
regions lying in parallel planes. The perpendicular distance between these parallel faces is the
height of the cylinder.
If r is the radius of the base and h is the height of the cylinder
The vertical surface of the cylinder is a curved surface.
C.S.A of a cylinder = perimeter of base × height = 2rh
T.S.A of a cylinder = C.S.A + 2 × B.ase Area = 2rh + 2 × r2 = 2r(r + h)
Volume of a cylinder = Area of base × height = r2h
Volume of material of a hollow cylinder = (R2 – r2)h
where, R is the outer radius and r is the inner radius of the cylinder.

145
CATapult
GEOMETRY

Example
The radius of a right circular cylinder is increased by 50%. Find the percentage increase in volume.
Let the original radius be r and height be h.
 New radius = 1.5r
Inscrease in volume
Percentage Increase in volume = × 100
Original volume
2 2 2 2
r (1.5r) h – rr h 2.25rr h – rr h
= 2 × 100 = 2 × 100
rr h rr h
2
rr h (2.25 – 1)
= 2 × 100 = 1.25 × 100 = 125.
rr h
Right Circular CONE: The base of a right circular cone is a circular region.
A is the centre of the circle and OA  AB where O is the vertex of the cone and AB is the radius (r).
OA is the perpendicular height (h) of the cone (i.e., the segment
joining the vertex of the cone and the centre of the base). OB is the slant
height ( , ) of the cone (i.e., a segment joining the vertex of the cone and any
point on the circumference of the base).

Slant height, , =
2 2
r +h
Curved surface area = r ,
Total surface area = r (r + , )
1 1
Volume of cone = × Volume of cylinder = × r2h.
3 3

Example

Find the volume of the largest right circular cone that can be cut out of a cube of edge 42 cm.
The base of the cone will be a circle inscribed in a face of the cube and its height will be equal
to the edge of the cube.
Radius of cone = 21 cm.
Height = 42 cm.
1
Volume of cone = r2h
3
1 22
= × × 21 × 21 × 42 = 19404 cu. cm.
3 7

146
Chapter QA
3-DIMENSIONAL FIGURES AND MENSURATION 3.6

FRUSTUM OF A CONE: A frustum is the lower part of a cone, containing the base, when it is cut

THEORY
by a plane parallel to the base of the cone.

Slant height, , =
2 2
h + (R – r)

PRACTICE EXERCISE CLASS EXERCISE


Curved Surface area of frustum = (R + r) ,
Total surface area of frustum = Base area + Area of upper circle
+ Area of lateral surface = (R2 + r2 + R , + r , )
rh 2
Volume of frustum = [R + r2 + Rr]
3

SPHERE: The set of all points in space, which are at a fixed distance
from a fixed point, is called a sphere.

The fixed point is the centre of the sphere and the fixed distance is
the radius of the sphere.

The entire surface of a sphere is curved.

Surface area of sphere = 4r2


4 3
Volume of a sphere = r
3

HEMISPHERE: A sphere cut by a plane passing through its centre forms two hemispheres. The
upper surface of a hemisphere is a circular region.

Curved surface area of a hemisphere = 2r2


Total surface area of a hemisphere = C.S.A + Area of circular base
= 2r2 + r2 = 3r2
1 2 3
Volume of a hemisphere = volume of sphere = r
2 3

Example

A toy has a hemispherical base and a conical top as shown in the figure. The perpendicular height
of the cone is 10 cm and radius of the hemisphere is 4 cm. Find the volume of the toy.

147
CATapult
GEOMETRY

Volume of toy = Volume of cone + Volume of hemisphere


1 2 2 3 1 2
= r h + r = r (h + 2r)
3 3 3
1 22
= × × 4 × 4(10 + 2 × 4)
3 7
1 22
= × × 16 × 18  302 cu. cm.
3 7

SPHERICAL SHELL: If R and r are the outer and inner radii respectively of a hollow sphere, then

4
Volume of material in a Spherical Shell = (R3 – r3).
3

SOLID RING: If R and r are the outer and inner radii of a solid ring
(can be considered as a cylindrical rod joined end to end), then

2
r
Volume = (R – r)2(R + r).
4
Curved Surface area = 2(R2 – r2).

PYRAMID: A pyramid is a solid, whose base is a polygon and lateral faces are triangular with a
common vertex.

A pyramid is said to be tetrahedron (triangular base), square pyramid, hexagonal pyramid etc.,
according to the number of sides of the polygon that form the base.

Tetrahedron Square Pyramid Regular Hexagonal Pyramid

In the above figures, O is the common vertex and OO' is the perpendicular height (h) of the pyramid.
The distance measured along the lateral face from the base to the apex or common vertex (O)
along the center of the face is called the slant height of the pyramid ( , ). In other words, it is
the altitude of the triangle comprising the lateral face.

148
Chapter QA
3-DIMENSIONAL FIGURES AND MENSURATION 3.6

In a pyramid with a base of n sides,

THEORY
Number of vertices = n + 1; Number of faces including the base = n + 1.
1
Surface area of lateral faces = × perimeter of base × slant height
2

PRACTICE EXERCISE CLASS EXERCISE


1
Total surface area of pyramid = Base area + × perimeter of base × slant height
2
1
Volume of pyramid = × Base area × height.
3

Concept Builder

1. A solid with 8 faces has 16 edges. What are the number of vertices of the solid
2. The height of a right prism is 8 units.
a) Find the L.S.A if perimeter of base is 10 units
b) Area of base, if volume = 320 cubic units.
3. A cuboid of dimensions 2 unit × 4 unit × 8 units is melted in to a cube. Find
a) Volume of cuboid
b) Surface area of cuboid
c) Length of the diagonal of cuboid
d) Volume of cuboid = Volume of the new cube (T/F)
e) Side of the new cube
f) Surface area of the cube
g) Length of diagonal of the cube
4. The radius and height of a cylinder are 7cm and 10cm respectively. Find
a) C.S.A b) T.S.A c) Volume
5. A tetrahedron (with equilateral triangular base) with side 6 cm has slant height of 7cm.
Find a) L.S.A b) T.S.A
6. Find the volume, C.S.A, T.S.A of a right circular cone of height and radius 4 and 3
respectively.
7. The radius of sphere is 9 and that of a hemisphere is 6 units. Find the ratio of their
volumes and ratio of their T.S.A’s
8. Find the Slant height, volume, curved S.A., T.S.A of the given frustum

Answer Key
T.S..A = (64 10 + 136)
8. Slant height = 4 10 , V = 784, C.S.A = 64 10 r
TSA = 3 : 1
4
6. CSA = 15, TSA = 24, Volume = 12 7. Volume =
27
b) 63 + 9 3 a) 63 c) 490 5. b) 238 a) 140 4.
f) 96 g) 4 3 e) 4 c) 2 21 d) T b) 112 a) 64 3.
b) 40 a) 80 2.
10 1.

149
CATapult
GEOMETRY

SOLVED EXAMPLES

Q : A road roller of diameter 1.75 m and length 1 m has to press a ground of area 1100 sq.
meter. How many revolutions does it make?

A : Area covered in one revolution = Curved surface area


Total area to be pressed
 Number of revolutions made =
Curved surface area
1100
=
2rrh
1100
=
22 1.75
2× 7 × 2 ×1

= 200.

Q : A rectangle 7 cm × 5 cm is rotated about its smaller edge as axis. Find the curved surface
area and volume of solid generated.

22
A : Curved Surface area = 2rh = 2 × × 7 × 5
7
= 220 sq. cm.

Volume of solid = r2h

22
= × 7 × 7 × 5 = 770 cu. cm.
7

Q : The perpendicular sides of the base of a right triangular metallic pyramid are 6 cm and 8 cm.
It weighs 810 g. Find its height, if density of the metal is 13.5 g/cc.
Weight 810g
A : Volume = = = 60 cc
Density 13.5g/cc
1
Volume = Base area × height
3
1 1
60 = × × 6 × 8 × height  Height = 7.5 cm.
3 2

150
Chapter QA
3-DIMENSIONAL FIGURES AND MENSURATION 3.6

Q : A rectangular sheet of paper of length 8 cm and breadth 22 cm is rolled end to end to form

THEORY
a right circular cylinder of height 8 cm. Find the volume of
the cylinder.
A : Circumference = 22 cm i.e., 2r = 22 cm.

PRACTICE EXERCISE CLASS EXERCISE


22 × 7 7
 r = = cm
2 × 22 2
22 7 7
 Volume = r2h = × × × 8 = 308 cu. cm.
7 2 2

Q : A regular hexagonal prism has perimeter of its base as 600 cm and height equal to 200 cm.
How many litres of petrol can it hold? Find the weight of petrol if density is 0.8gm/cc.

Perimeter 600
A : Side of hexagon = = = 100 cm.
Number of sides 6

3 3
Area of regular hexagon = × 100 × 100 = 25950 sq. cm.
2
Volume = Base Area × height = 25950 × 200 = 5190000 cu. cm = 5.19 cu.m.
Weight of petrol = Volume × Density = 5190000 cc × 0.8 gm/cc = 4152000 gm = 4152 kg.

Q : A solid metallic right circular cone of height 45 cm and radius 15 cm is melted and two solid
cylinders of height 15 cm are prepared. If the volume of one is 8 times that of the other, find
the radius of smaller one.

A : Volume of cone = Volume of smaller cylinder + 8 times of volume of the smaller cylinder =
9 times volume of smaller cylinder.

1
 × 45 × 15 × 15 = 9 ×  × r2 × 15
3
45 × 15 × 15
 r2 =
9 × 3 × 15
 r = 5 cm.

151
CATapult
GEOMETRY

Q :

An iron ball of diameter 6 inches is dropped into a cylindrical vessel of diameter 1 ft filled
with water. Find the rise in water level. (Assume density of water and material used in sphere
is the same)

1 feet 12 inches
A : Radius of vessel = = = 6 inches.
2 2
Volume of water that has risen = Volume of sphere

4 2
 R2h = r
3
4
 (6)2 × h = × 33
3
4
 36 × h = × 33
3
 h = 1 inch.

152
Chapter QA
3-DIMENSIONAL FIGURES AND MENSURATION 3.6

CLASS EXERCISE

Teaser

A cube is painted with six different colours on its six faces (red, yellow, blue, green, black and
purple). Then it is cut into 125 identical smaller cubes. How many of these cubes will have:

a) No face painted?
b) One face blue and exactly one other face painted?
c) One face purple and at least one other face painted?
d) One face red and one face yellow?

153
CATapult
GEOMETRY

Prisms

1. A physics laboratory has a triangular prism made of glass. The


base is an equilateral triangle of side 2 cm, while the distance
between the two triangular faces is 10 cm.
a) What is the volume of glass used in making the prism?
b) If the entire surface of the prism is to be coated with black
paint, what area needs to be painted?

2. An open (hollow) cylindrical barrel has a base radius of 14 cm


and a height of 33 cm
a) Find the volume and the surface area of the barrel.
b) If a fly is at a point on the bottom inner rim of the barrel, how
much distance (minimum)
will it have to travel to reach the farthest point on the barrel?
c) If an ant is at a point on the bottom inner rim of the barrel,
how much distance (minimum) will it have to travel to reach
the farthest point on the barrel?

A prism is a polyhedron formed by connecting the corresponding vertices of two congruent


polygonal figures (the bases) in parallel planes. (A cylinder also can be considered prismatic).

The following table gives the general formulae for a right prism and some examples of prisms

Volume Surface Area


General Right Prism Area(base) x Height 2 × Area(base) + Perimeter(base) × Height
Cuboid (length l, breadth b, (l x b) x h 2lb + 2(l +b) h
height h)
Regular Hexagonal Prism (side 3 3s
2
3 3 s2 + 6sh
s, height h) × h
2
Equilateral Triangular Prism 3s
2
3s
2

(side s, height h) × h + 3sh


4 2
Cylinder (radius r, height h) r2 × h 2r2 + 2rh = 2r(r + h)

3. * An open jewelry box in the shape of a square-based prism has a base side of length 5
inches and a height of 4 inches. The walls are of negligible thickness.
a) What is the capacity of the box in cubic inches?
b) What is the cost of painting the surface of the box at a cost of 20 paise per square inch?

154
Chapter QA
3-DIMENSIONAL FIGURES AND MENSURATION 3.6

Pyramids

THEORY
4. A square pyramid is discovered in Egypt with a base of 60 m and
a height of 40 m.

PRACTICE EXERCISE CLASS EXERCISE


a) If the pyramid is solid stone, what is the volume of stone used
in the construction?
b) If on further examination, it is found that the interior of the
pyramid contains a hollow chamber of similar shape with a
square base of 45 m and a height of 30 m, then how much
stone is used in the construction?

5. A conical vessel (with its vertex at the bottom) has a height of 24


units and a volume of 216 litres. What will be the volume of water in it if:
a) water is filled to a height of 8 units?
b) water is filled to a height of 12 units?
c) it is turned upside down and water is filled to a height of 8 units?
d) it is turned upside down and water is filled to a height of 12 units?

A pyramid is a polyhedron formed by connecting the vertices of a polygonal figure (the base)
with a point (the vertex) outside the plane of the base. (A cone also can be considered pyramidal
in nature).
The following table gives the general formulae for a right pyramid (with a regular polygonal
base), and some examples of pyramids

Volume Surface Area


General Right Pyramid 1 1
× Area(base) × Height Area(base) + × Perimeter(base)
3 2
× Slant Height
Regular Hexagonal Pyramid (side 3s
2
3 3s
2

s, height h, slant height l) × h + 3sl


2 2
Equilateral Triangular Prism 3s
2
3s
2
1
(side s, height h, slant height l) × × h + 3sh
3 4 2
Cone (radius r, height h, slant 1 r2 + rl = r(r + l)
× r2 × h
height l) 3

6. * An ice-cream cone has a radius of 42 mm and a height of 100 mm. The seller claims that
the cone contains 200 ml of ice-cream. Is he telling the truth? (Note: 1 ml = 1 cubic cm)

7. *A tetrahedron has all faces as equilateral triangles of side 1, as shown.


Find its volume.

155
CATapult
GEOMETRY

Spheres and Frustums

8. A hemisphere of radius 14 is melted down and formed into 4 smaller identical spheres.
a) What is the radius of each smaller sphere?
b) What is the ratio of the new total volume to the original total volume?
c) What is the ratio of the new total surface area to the original surface area?

9. A cone of base radius 7 cm and height 24 cm is cut parallel to the base to form a frustum
or truncated cone. If the cut is made:
a) at a distance of 6 cm from the vertex, find the volume of the frustum formed.
b) at a distance of 4 cm from the base, find the ratio of the volumes of the two parts formed.
c) at a distance of 12 cm from the vertex, find the upper radius of the frustum formed.
d) at a distance of 4.8 cm from the base, find the slant height of the frustum so formed

Other standard figures which are neither pyramidal nor prismatic include the sphere (and hemisphere),
and the frustum or truncated pyramid.

The following table gives general formulae for spheres, hemispheres and conical frustums:

Volume Surface Area


2
Sphere (radius r) 4 3 4r
r
3
Hemsphere (radius r) 2 3 3r2
r
3
Frustum of cone (radii R and r, 1 (R2 + r2 + Rl + rl)
× πh(R2 + Rr + r2)
height h, slant height l) 3

Note that in many cases it is far easier to use the basic principles of similarity of figures to solve
problems involving frustums, rather than applying a cumbersome formula.

10. * An Aztec temple made of stone is shown in the figure.


It is in the shape of the frustum of a square pyramid. The bot-
tom surface is a square of side 16 m, while the top surface is
a square of side 4 m. The height of the pyramid is 9 m. Find
the volume of the pyramid.

156
Chapter QA
3-DIMENSIONAL FIGURES AND MENSURATION 3.6

Compound Figures

THEORY
11. A cube of side 3 inches is taken and a hole of circular cross-section, with diameter 1 inch,
is drilled right through from the centre of one face to the centre of the opposite face. Find

PRACTICE EXERCISE CLASS EXERCISE


the volume and surface area of the resultant block.

12. A child’s spinning top is made in the shape of a cylinder of radius ‘r’ and length ‘l’, topped
at each end by a cone of base radius ‘r’ and height ‘l’
as shown in the figure. Find:
a) The volume of the top
b) The surface area of the top

13. What is the maximum possible volume of a sphere that can fit inside:
a) A cube of side 6 cm
b) A prism of height 8 cm whose base is an equilateral triangle of side 6 cm

14. * What is the maximum possible volume of a cube that can fit inside:
a) A sphere of radius 6 cm
b) A cylinder of radius 6 cm and height 8 cm

15. * Find the volume and surface area of the adjoining figure if each of the small cubes has a
side of length 1 cm, and each cube in an upper
layer has a cube below it.

16) * A molded plastic toy is made as shown in the


adjacent figure. The base is a cuboid of dimensions
16 mm x 8 mm x 4 mm, while the projections
5
are cylinders of diameter 2 mm and height
r
mm. Find the volume of the block. If the cost of
painting such blocks is 2 paise per sq cm, find the
cost of painting a box of 100 such blocks.

157
CATapult
GEOMETRY

Challengers

1) A metal cylinder of radius 12 cm and height 16 cm is melted down and formed into identical
smaller spheres of radius 3 cm.
a) How many smaller spheres are formed?
1) 48 2) 54 3) 72 4) 64

b) What is the percentage change in the total surface area?


1) 143% increase 2) 243% increase
3) 343% increase 4) No change

2) A right-angled connector made of rubber can be used to connect two hose-pipes. If the
connector has an outer diameter of 1 cm, a length of 2.5 cm
and a thickness of 1 mm, find the volume of rubber used in its
construction.
1) 1.44  cc 2) 18  cc
(5 + 2 3) r
3) cc 4) 0.36  cc
16

3. There is a cube with sides of length 5 units. An ant on one vertex wants to reach the far-
thest (opposite) vertex, traversing the minimum distance in the process. How far will it have
to crawl?

1) 15 2) 5 ^ 2 + 1 h 3) 5 5 4) 5 3

4. A cone and a sphere have equal heights and equal volumes. Find the ratio of their radii.
1) 1 : 1 2) 2 : 1 3) 3 : 1 4) 2 : 1

5. * The ticket for the Chess World Cup final is in the shape of a regular hexagon, with a side
of 6 inches. A circular stamp, of radius 2 inches, is meant to be used
to stamp the ticket in the centre. By mistake, the stamp get misaligned
and stamps the ticket in such a way that the edge of the stamp ex-
actly touches a vertex of the hexagon, while the centre of the stamp
lies on the line joining that vertex to the opposite vertex (as shown in
the figure). Find the portion of the stamp, in square inches, which falls
outside the ticket.

4 3 4 3
1)  – 2 3 2) 4 – 3)  – 3 4) 2 –
3 2 3 2

158
Chapter QA
3-DIMENSIONAL FIGURES AND MENSURATION 3.6

PRACTICE EXERCISE-1
DIRECTIONS for questions 1 to 14: Choose the correct alternative.

1. Find the length of the longest rod that can be fit into a box of dimensions 6 cm × 3 cm ×
2 cm.
1) 45 cm 2) 7.8 cm 3) 7 cm 4) 8 cm

2. The lower part of a tent is a circular cylinder and its upper part is a right cone. The diameter
of the base is 70 m. The total height is 15 m and the height of the cylindrical part is 3 m.
Find the cost of canvas required at Rs.10 per sq. m.
1) Rs.47300 2) Rs.162800
3) Rs.16280 4) Rs.44000

3. A thermal chimney of circular cross section has outer and inner radii 3 m and 2 m respec-
tively. Find the cost of cement finishing for the the inner and outer surface at Rs.20 per sq.
meter, if the height of the chimney is 56 m.
1) Rs.80000 2) Rs.45760 3) Rs.35200 4) Rs.7040

4. Water flows out at the rate of 10 m/minute from a cylindrical pipe of diameter 5 mm. Find
the time taken to fill a conical tank whose diameter at the base is 40 cm and depth is 24
cm.
1) 153.6 minutes 2) 51.2 minutes 3) 12.8 minutes 4) 128 minutes

5. The sides of a cube were uniformly increased to increase the total surface area by 44%.
What was the percentage increase in the volume of the cube?
1) 33.1% 2) 57.6% 3) 66.3% 4) 72.8%

6. A spherical iron ball of radius 56 m is to be painted. However, it is realised that the require-
ment is for balls of radii 14 m. So, the bigger ball is melted and made into smaller balls of
3
radii 14 m each. The rate per sq. meter for painting the smaller balls is reduced to th of
8
the original rate. What is the impact on the original painting cost?
1) decreases by 50% 2) increases by 50%
3) decreases by 37.5% 4) doubles

7. A bowl in the shape of a hemisphere and of inner radius 3 33 inch is filled with soup of
volume 18 inch3. At the most how many Manchurian balls, of 1 inch radius, can be added
to the soup so that it does not spill over?
1) 1 2) 2 3) 3 4) 4

8. 1 m3 of metal is used to make four wires of equal length whose cross-sectional area is 250
cm2 each. Find the length of the wires made.
1) 0.05 m 2) 0.5 m 3) 0.01 m 4) 10 m

159
CATapult
GEOMETRY

9. A right circular cone, a right circular cylinder and a hemisphere, all have the same radius,
and the heights of the cone and the cylinder equal their diameters. Then their volumes are
proportional, respectively to _____.
1) 1 : 3 : 1 2) 2 : 1 : 3 3) 3 : 2 : 1 4) 1 : 2 : 3
(Past CAT question)

10. If a right circular cone of height h is cut by a plane parallel to the base and at a distance
h
from the base, then the volumes of the resulting cone and the frustum are in the ratio
3
_____.
1) 1 : 3 2) 8 : 19 3) 1 : 4 4) 1 : 7
(Past CAT question)

11. A wooden box (open at the top) of thickness 0.5 cm and length 21 cm, width 11 cm, and
height 6 cm, is painted on the inside. The expenses of painting are Rs. 70. What is the ap-
proximate rate of painting in rupees per sq cm?
1) 0.72 2) 0.51 3) 0.13 4) 0.26
(Past CAT question)

12. A cube is painted red on all sides. It is then cut into 8 identical smaller cubes. All unpaint-
ed areas are painted blue. Then each cube is further cut into 8 identical smaller cubes. All
unpainted areas are painted green now. What is the ratio of the total areas painted red, blue
and green?
1) 1 : 2 : 1 2) 1 : 2 : 2 3) 2 : 1 : 1 4) 1 : 1 : 2

13. A right angled triangle ABC of perpendicular sides AB and BC is rotated about AB as the
axis. If AB = 12 and BC = 5. Find the total surface area of solid generated.
1) 90 2) 45 3) 30 4) 85

1
14. A right circular cone is immersed in clear water to th of its height, such that its tip is in
4
water and the circular base is at the top. Find the volume of the portion of the cone above
water if the full height of the cone is 12 cm and the radius of the base of the cone is 7 cm.
4851 77 4851
1) cm3 2) cm3 3) 616 cm3 4) cm3
8 4 4

DIRECTIONS for question 15: Solve as directed.

15. 2112 m3 of mud is dug out from the middle of a circular playground to build an 8 metre-deep
rectangular swimming pool. The remaining 3586 m2 area of the playground is covered in
grass. What is the radius (in metres) of the playground?

160
Chapter QA
3-DIMENSIONAL FIGURES AND MENSURATION 3.6

DIRECTIONS for questions 16 to 21: Choose the correct alternative.

PRACTICE EXERCISE CLASS EXERCISE THEORY


16. A hollow hemispherical bowl of outer radius 6 cm and uniform thickness 2 cm is remolded
to form a hollow spherical ball of same outer radius. By what percentage will the thickness
reduce?
1) Between 35% and 40% 2) Between 45% and 50%
3) Between 55% and 60% 4) Between 65% and 70%

17. Two identical rectangular sheets of the form A’ABB’ are folded along C’C to form a triangular
prism whose bases are equilateral triangles. It is known that AC = AA’. If the volume of the
prism is 27 3 cubic units, find the area of the rectangle A’ABB’ (in square units).

1) 18 2) 24 3) 27 4) 36

18. A square based pyramid is cut horizontally into 2 parts as shown in the diagram. The ratio
of height of the original pyramid to that of the new pyramid is the same as the ratio of side
of square of original pyramid to that of new pyramid, and the volume of new pyramid is
1
th of the volume of original pyramid. What is the ratio of the sum of surface areas of the
8
triangular faces of part 1 to the sum of the area of the inclined faces of part 2?

1) 2:1 2) 3:1 3) 1:3 4) 2:3

161
CATapult
GEOMETRY

19. ABCDEFGH is a cuboid. String 1 is wound around four faces of the cuboid starting from
point A and ending at point D, as shown in the figure. String 2 is wound around four faces
of the cuboid starting from point E and ending at point F, as shown in the figure. What is
the ratio of the smallest possible lengths of string 1 and string 2?

145 148 153 153


1) 2) 3) 4)
104 109 116 116

20. A toy is made up of a hollow cylindrical base and a hollow hemispherical top as shown in
the figure. The toy is open only on one end and has uniform thickness “t” throughout. The
outer curved surface area of the toy is 2013 cm2 and the inner curved surface area of the
toy is 1188 cm2. The total height of the toy is 30.5 cm. Find the thickness of the toy. (Take
22
 = )
7

1) 2.8 cm 2) 3.15 cm 3) 3.5 cm 4) 4.2 cm

21. Three cones of base radii 1 cm, 2 cm and 3 cm and same height 5 cm are placed next to
each other such that the base of each cone touches the bases of the other two cones. What
is the circumference of the circle that passes through the tips of all the three cones?
1) 10 2) 7.5 3) 5 4) 3

162
Chapter QA
3-DIMENSIONAL FIGURES AND MENSURATION 3.6

PRACTICE EXERCISE-2
DIRECTIONS for question 1: Solve as directed.

1. If the surface area of a blue-coloured sphere is 800% more than that of a red-coloured
sphere, by what percent is the volume of the blue-coloured sphere more than that of the
red-coloured sphere?

DIRECTIONS for questions 2 to 20: Choose the correct alternative.

2. A slab of ice 8 inches in length, 11 inches in breadth, and 2 inches thick was melted and
re-solidified in the form of a rod of 8 inches diameter. The length of such a rod, in inches
is nearest to:
1) 3 2) 3.5 3) 4 4) 4.5
(Past CAT question)

3. The diameter of a hollow cone is equal to the diameter of a spherical ball. If the ball is
placed at the base of the cone, what portion of the ball will be outside the cone?
1) 50% 2) Less than 50%
3) More than 50% 4) Cannot be determined
(Past CAT question)

4. A solid square pyramid having base area of 49 cm2 is cut by a plane parallel to the base.
The upper part of the pyramid is melted to form a sphere. If the diameter of the sphere is
equal to the side of base of melted part of the pyramid then, what was the height of the
solid square pyramid?
1) 7 cm 2) 22 cm 3) 11 cm 4) 15 cm

5. A room with dimensions 24 m, 20 m and 14 m is ventilated by a rectangular opening with


4 5
area m × m. At what rate should air pass through the opening per unit area, so that
3 4
1
th of the air in the room would change every hour?
5
1) 14.5 m/min 2) 15 m3/min 3) 13.4 m3/min 4) 13.44 m/min

6. A metal ball, 6 cm in diameter, is dropped into a cylindrical vessel of diameter 12 cm and


height 20 cm. The water level, inside the cylinder, is 19.5 cm from the bottom face of the
cylinder. Find the volume of the water that spills out.
1) 18 cm3 2) 36 cm3 3) 100 cm3 4) No water spills out.

7. A regular tetrahedron of length 12 cm is accommodated in a sphere such that all its vertices
touch the sphere. Find the radius of the sphere.
1) 7.2 cm 2) 7 cm 3) 7.5 cm 4) 6.5 cm

8. An ice-cream, in the shape of right circular cone is cut (parallel to its base) into 5 slices,
all of equal heights. What is the ratio of the volume of middle slice to that of the volume of
the biggest slice?
1) 4 : 25 2) 41 : 93 3) 19 : 61 4) 27 : 125

163
CATapult
GEOMETRY

9. A hollow cone with base radius 9 cm and height 12 cm is filled with water up to a level of
8 cm height when kept on closed circular base. What height does the water come upto, if
this cone is kept in an inverted position such that the apex of the cone will be facing down?

1) 3 3 26 cm 2) 8 cm 3) 7 3 13 cm 4) 4 3 26 cm

10. An iron pillar has some part in the form of a right circular cylinder and the remaining part in
the form of a right circular cone. The radius of the base of the cone as well as the cylinder
is 8 cm. The cylindrical part is 240 cm high and the conical part is 36 cm high. Find the
weight of the pillar, if 1 cm3 of iron weighs 7.8 gm.
1) 3.6 kg 2) 163.7 kg 3) 253 kg 4) 395 kg 5) 300 kg

11. A glass showpiece in the shape of a sphere of radius 5 cm is packed into a cardboard cubic
box whose sides are of length are 12 cm. To prevent the showpiece from rolling around,
8 identical rubber spheres are placed in the corners of the box. What is the radius of the
rubber spheres?
6 3 –5 12 3 – 10 6 3 –5 12 3 – 10
1) cm 2) cm3) cm 4) cm
1+ 3 2+ 3 1+ 2 2+ 2

12. The ratio of the volume of a hexagonal pyramid, P, to that of a hexagonal prism, Q, is 4 :
25. If the heights of P and Q are interchanged, then the ratio of their respective volumes
becomes 9 : 100. What is the ratio of their respective volumes, if the sides of P and Q are
interchanged?
1) 100 : 9 2) 100 : 81 3) 100 : 27 4) 23 : 36

13. What is the length of side of a cube, if the area of the largest equilateral triangle that can
perfectly fit inside the cube is 50 3 cm2?

1) 5 cm 2) 5 3 cm 3) 10 3 cm 4) 10 cm

14. One of the base edges of a metallic plate, which is in the form of a right triangle is soldered
to a wire. The wire is rotated around its own axis. What is the maximum volume that can
be generated by the metallic plate given that the smallest side of the triangular plate is 8
cm and the largest side is 2 cm greater than the second largest side?
1) 600 sq.cm 2) 320 sq.cm 3) 60 sq.cm 4) Data insufficient

164
Chapter QA
3-DIMENSIONAL FIGURES AND MENSURATION 3.6

15. The given three-dimensional figure has bases which are regular hexagons. One of the bases is

PRACTICE EXERCISE CLASS EXERCISE THEORY


placed on the x-y plane. The coordinates of the points A, B and C are (1, 0, 0), (3, 0, 0) and
(3, 0, 4). What will be the coordinates of the point on the 3-D figure that is farthest from A ?

1) (3, 2 3 , 4) 2) (3, 2 3 , 3) 3) (4, 2 3 , 3) 4) (3, 3 , 4)

16. A fish tank in the shape of a cuboid has length, breadth and height of 60 cm, 40 cm and
80 cm respectively and it is filled with water. The base of the tank is tilted by 45° along the
breadth and water coming out from the tank is collected in an empty vessel. The volume
of water collected in the empty vessel is 32 litres. What was the height of water that was
initially there in the cuboid tank? [Assume thickness of the water tank to be negligible].
1) 80 cm 2) 60 cm 3) 63.33 cm 4) 76.66 cm

17. A solid sphere is cut along three planes perpendicular to each other to form 8 identical parts.
The curved surface of each part is painted red and the flat surfaces are painted blue. Find
the ratio of the area of the red surface to the area of the blue surface in each part.
1) 2 : 3 2) 3 : 8 3) 1 : 2 4) 1 : 3

18. A right circular cylinder of radius 10 cm and height 40 cm is placed on the ground. A sphere
of largest possible size is enclosed in the cylinder and is on the base of the cylinder. A sec-
ond sphere having half the radius of the first sphere is stuck to the first sphere. Continuing
this, a number of spheres are stuck to one another such that each sphere touches the two
adjoining spheres and each sphere has radius equal to half the radius of the earlier sphere.
If the line joining the centres of all the spheres is parallel to the axis of the cylinder and
the total height of the “tower of spheres” is equal to the height of the cylinder, what is the
sum of the surface areas of all the spheres?
1600
1)
3 r
800
2)
3 r
3200
3)
3 r
4) More information is needed to answer the question

165
CATapult
GEOMETRY

19. A cube is cut into three parts as shown below, such that the ratio of the total surface areas
of the resulting cuboids is 13 : 17 : 15 (a, b and c are integers). Which of the following
can be the volume of the original cube, given that the length of the side of the cube is an
integer value?

1) 343 cu. units 2) 1000cu. units 3) 512 cu. units 4) 729 cu. units

20. Seven identical solid spheres are fit inside a larger hollow sphere such that they touch each
other and the outermost spheres touch the larger sphere. The centres of all the spheres lie
on the same plane. Find the ratio of the volume inside the larger sphere that is occupied by
the smaller spheres to the volume of air inside the larger sphere.

1) 7 : 27 2) 1 : 3 3) 7 : 20 4) 7 : 10

DIRECTIONS for question 21: Solve as directed.

21. A cone of radius 21 cm is inserted inside a larger cone (as shown in the figure) such that
when it can’t go further in, the vertex of the smaller cone is on the same plane as the base
of the larger cone. If the radius and height of the larger cone are 35 cm and 45 cm respec-
tively, find the volume of the larger cone (in cm3) that is not occupied by the smaller cone.

166
CATapult
ARITHMETIC

QA-3.7 TIME, SPEED AND DISTANCE THEORY


Time, Speed & Distance
Definition of Terms
The speed of a body is defined as the distance covered by it in unit time.
Distance
Speed = ;
Time
Distance
Time = ;
Speed
Distance = Time × Speed

Units of measurement
• Time is measured in seconds (s), minutes (min) or hours (hr).
• Distance is usually measured in metres (m), kilometres (km), miles, yards or feet.
• Speed is usually measured in metres/sec. (m/s), kilometres/hour (km/hr) or miles/hr.

Conversion of units
1. 1 hour = 60 minutes = 60 × 60 seconds.
2. 1 kilometer = 1000 metres
3. 1 kilometre = 0.6214 mile
1 mile = 1.609 kilometre
i.e., 8 kilometres 5 miles
4. 1 yard = 3 feet
km = 5m
5. 18s
hr
m = 18km
6. s =
5hr
km 5 miles
7. .
hr 8hr
miles = 22 = ft
8. 15 sec
hr

Relation between Time, Speed & Distance

1
1. If distance is constant, Speed 
Time

167
CATapult
ARITHMETIC

Example
3
Walking at 4 th of his usual speed, a man is 1 1 hrs late. Find his usual travel time.
2
If the usual time is t hrs, and usual speed is x kmph, then,

Distance travelled = xt = 4 × x b t + 2 l & t = 4 b t + 2 l


3 3 3 3

9
 4t = 3t + 2  t = 4 1 hrs
2
 Original time taken for journey = 4 1 hrs and new time taken for journey = 6 hrs
2
With distance being the same, decrease in speed leads to increase in time i.e., with distance being
constant, speed is inversely proportional to time.

2. If time is constant, Distance  Speed


Example
A car travels at 30 kmph for the first 2 hours of a journey and then travels at 40 kmph for the
next 2 hours of the journey. Find the ratio of the distances travelled at the two speeds.
Since Time is constant in both the intervals, ratio
Distance Travelled in the 1st interval = Distance Travelled in the 2nd interval

Speed during the 1st interval Speed during the 2nd interval
st
Distance Travelled in the 1st interval = Speed during the 1 interval
 nd nd
Distance Travelled in the 2 interval Speed during the 2 interval
30 3
 40 = 4 

i.e., we can say that ratio of the distance travelled is equal to the ratio of the speed of the vehicle
in two equal time intervals or that distance is directly proportional to speed if time is constant.

3. If speed is constant, Distance  Time


Example
A car moves for 3 hours at the speed of 20 kmph and a truck moves for 4 hours at the same
speed. Find the ratio of the distances covered by the car and the truck.
Since Speed is constant for both the vehicles
Distance Travelled by the car Distance Travelled by the truck
 =
Time taken by the car Time taken by the truck
Distance Travelled by the car Time taken by the car
 =
Distance Travelled by the truck Time taken by the truck
3× 20 3
= 4 × 20 = 4
i.e., we can say that ratio of the distance travelled is equal to the ratio of the time taken by the
two vehicles or that distance is directly proportional to time if speed is constant.

168
Chapter QA
TIME, SPEED AND DISTANCE 3.7

Average speed

THEORY
If a body travels d1, d2, d3... dn distances, with speeds s1, s2, s3... sn in time t1, t2, t3 ... tn
respectively then the average speed of the body through the total distance is given by:

PRACTICE EXERCISE CLASS EXERCISE


Total distance travelled
Average Speed =
Total time taken
d1 + d2 + d 3 + ...dn s1 t1 + s2 t2 + s 3 t 3 + ... + sn tn d1 + d2 + d 3 + ... + dn
= t + t + t + ...t = t1 + t2 + t 3 + ... + tn =
1 2 3 n d 1 + d 2 + d 3 + + dn
s1 s2 s 3 ... sn

Cases of an object travelling with different speeds during different time intervals
1. If a body covers the distance d1 and d2 at a speed of s1 and s2 km/hr, respectively, in time
t1 and t2 then the total time taken T is given by:
d d
T = t1 + t2 = s11 + s22 .
The total distance covered is given by: D = d1 + d2 = s1t1 + s2t2.
2. While travelling a certain distance d, if a man changes his speed in the ratio m : n, then the
ratio of time taken becomes n : m.
3. If a certain distance (d), say from A to B, is covered at ‘a’ km/hr and the same distance is
covered again say from B to A in ‘b’ km/hr, then the average speed during the whole journey
is given by:

Average speed = b a + b l
2ab km
... (which is the harmonic mean of a and b)
hr
Also, if t1 and t2 is time taken to travel from A to B and B to A, respectively, the distance
‘d’ from A to B is given by:

d = (t1 + t2) b a + b l
ab

d = (t1 – t2) b a - b l
ab

d = (a – b) b t 1- t2 l
t t
1 2

4. If a body travels a distance ‘d’ from A to B with speed ‘a’ in time t1 and travels back from
m
B to A i.e., the same distance with n of the usual speed ‘a’, then the change in time taken
to cover the same distance is given by:

Change in time = b m – 1 l × t1; for n > m


n

= b1 - m l × t1; for m > n


n

169
CATapult
ARITHMETIC

SOLVED EXAMPLES

Q : If a boy goes to school at 6 km/hr and returns home at 4 km/hr, find his average speed.
2 × 6 × 4 = 48
A : Average speed = 6+4 10 = 4.8 km/hr

Q : A man starts from B to K, another from K to B at the same time. After passing each other
1 4
they complete their journeys in 3 3 and 4 5 hours, respectively. Find the speed of the
second man if the speed of the first is 12 km/hr.

4
1 st man's speed b b = 45 24 3 = 36 = 6
A : = = a 1
=
5 × 10 25 5
2nd man's speed a 33
12 =6 60
  2nd man’s speed = 6 = 10 km/hr.
2nd man's speed 5

Q : I shall be 40 min late to reach my office if I walk from my house at 3 km/hr. I shall be 30
min early if I walk at 4 km/hr. Find the distance between my house and the office.
A : Let the usual time taken be ‘t’ hours and speed be x km/hr.

Distance = xt = 3 b t + 60 l = 4 b t - 60 l
40 30

 3t + 2 = 4t – 2
 4 = t

 Distance = 3 b4 + 3 l = 14 km.
2

Q : A man travels 120 km by ship, 450 km by rail and 60 km by horse taking altogether 13
hrs 30 min. The speed of the train is 3 times that of the horse and 1½ times that of the
ship. Find the speed of the train.

A : If the speed of the horse is x km/hr; that of the train is 3x and that of the ship is
3x
1 = 2x km/hr
12
120 + 450 + 60 = 27 60 + 150 + 60 = 27
 2x 3x x 2  x x x 2

270 = 27
 x 2  x = 20  Speed of the train = 60 km/hr

Q : A and B walk from P to Q, a distance of 21 km at 3 and 4 km/hr. B reaches Q, and imme-


diately returns and meets A at R. Find the distance from P to R.

A : When they meet, both together have walked 2 × 21 = 42 km Since their speeds are as 3
: 4, distances travelled are also as 3 : 4

3
 Distance travelled by A = PR = 7 × 42 = 18 km.

170
Chapter QA
TIME, SPEED AND DISTANCE 3.7

4
Q : A train after travelling 50 km from A meets with an accident and proceeds at 5 th of the

THEORY
former speed and reaches B, 45 min late. Had the accident happened 20 km further on,
it would have arrived 12 min sooner(than if the accident occured at C). Find the original

PRACTICE EXERCISE CLASS EXERCISE


speed and the distance.

A : Let the speed be x km/hr.

20

A C D B

4 5
When the speed becomes 5 th of the usual, time taken would become 4 th the usual, i.e.,

1
4 th more of the usual time.

1
So, 4 th of the usual time taken to travel CB = 45 min.

1
 4 th of usual time taken to travel CD (i.e. 20 km) = 12 min

 Usual time to travel 20 km = 48 min.

60
 Usual speed = 20 × 48 = 25 km/hr.

Usual time taken to travel CB = 45 × 4 = 3 hrs.

 Distance CB = 25 × 3 = 75 km.

 Total distance = 50 + 75 = 125 km.

Q : A man travelling from A to B at 3 mph, takes half an hour rest at B, and returns to A at 5
mph. Total time taken is 3 hrs 26 min. Find the distance from A to B. (m  miles)

A : Total time taken for travelling = 3 hrs 26 min. – 30 min. = 2 hrs. 56 min.

56 176
= 2 60 = 60 hrs.

 Distance from A to B

176 b 3× 5 l = 176 15
= 60 3 + 5 60 × 8 = 5.5 miles

171
CATapult
ARITHMETIC

8
Q : Walking 7 th of his original rate, a man reaches his office 3 minutes early. Find the usual
time he takes to reach office.

A : b1 - 8 l × Usual time = Change in time


7

 Usual time = 3 × 8 = 24 minutes.

Q : The ratio between the speed of Meena and Teena is 2 : 3. Meena takes 20 minutes more
than Teena to walk from A to B. If Meena had walked at double her speed, find the time
she would take to walk from A to B.

A : Ratio of speed of Meena and Teena is 2 : 3.


 Ratio of time taken = 3 : 2
If Teena takes x minutes to walk from A to B, then Meena takes x + 20 minutes.
x + 20 = 3
 x 2  2x + 40 = 3x  x = 40 minutes.
 Meena takes 60 minutes walking at her usual speed.
 At double the speed, she would take 30 minutes.

172
Chapter QA
TIME, SPEED AND DISTANCE 3.7

Concept Builder 1

THEORY
1. A distance is covered by man in 2 hrs and 45 minutes at 4 kmph. How much time will be

PRACTICE EXERCISE CLASS EXERCISE


taken to cover same distance at 16.5 kmph?
2. The ratio of speeds of A and B is 4 : 5. If the time taken by B to cover a certain distance
is 40 minutes, then the time taken by A to cover the same distance is:
4 5
3. A man performs 7 of the total journey by train, 21 by car and the remaining 8km on
foot. His total journey is:
4. A person goes from X to Y at 20 kmph and comes back to X at 30 kmph. Find his average
speed?
5. The ratio between the speeds of two trains is 4 : 5. If the first train runs 200 km in 2 hours,
then the speed of the second train is:
6. A person starts at 6 km/hr from a particular city Q towards city R. After travelling 4 hours
he realises that he will not reach in an estimated time and for this he increases his speed
by 2 km/hr and travels for the next 4 hours and reaches city R in the estimated time. If
next day he wants to travel from city Q to city R in the estimated time then at what speed
should he travel?
7. Travelling at a speed of 4 km/hr from his home Rakesh reaches his office 45 minutes late.
Next day, Rakesh travels at a speed of 5 km/hr from his home and reaches his office on
time. What is the distance between his home and office?

Answer key

15 km. 7.
7 km/hr. 6.
125km/hr. 5.
24 kmph. 4.
42 km. 3.
50 minutes. 2.
40 mins. 1.

173
CATapult
ARITHMETIC

Relative speed
The word ‘relative’ means one with respect to another. Relative Speed means the speed of an
object A with respect to another object B, which may be stationary, moving in the same direction
as A or in the opposite direction as A.
Cases related to Relative Speed
Case 1
When one object is stationary and the other is moving.
Consider a boy standing on a platform and a train passing by. Here, the boy is stationary, while
the train is moving. The relative speed of the train and the boy will be the speed of the train.
Relative speed of a stationary object and a moving object = Speed of the moving object.
Case 2
When the two objects are moving in the opposite direction.
Consider two boys, A and B, standing at two opposite ends of a ground. Now, if they start walking
towards each other in a straight line, they would meet sooner than had one of them been stationary
and their relative speed will be the sum of their speeds.
Relative speed of two objects moving in opposite direction = Sum of their speeds.
Case 3
When the two objects are moving in the same direction.
Consider a boy ‘A’ walking from x to y. Now, if another boy ‘B’ walks from a point z which is
behind x, in the same direction as A at a speed greater than A’s, they would meet later than had
A been stationary at x. Their relative speed is the difference of their speeds.
A

z x y
B
Relative speed of two objects moving in the same direction = Difference of their speeds.
Rules and Formulae for Relative Speed
1. Time taken by a moving object ‘x’ metres long in passing a stationary object of negligible
length from the time they meet is same as the time taken by the moving object to cover ‘x’
meters with its own speed.
2. Time taken by a moving object ‘x’ metres long in passing a stationary object ‘y’ metres long
from the time they meet, is same as the time taken by the moving object to cover ‘x + y’
metres with its own speed.
3. If two objects of length ‘x’ and ‘y’ metres move in the same direction at ‘a’ and ‘b’ m/s,
then the time taken to cross each other from the time they meet
Sum of their length x+y x+y
= i.e., if a > b or else, .
Relative speed a -b b-a

4. If two objects of length ‘x’ and ‘y meters, move in the opposite direction at ‘a’ and ‘b’ m/s,
then the time taken to cross each other from the time they meet
Sum of their length x + y
= =
Relative speed a+b

174
Chapter QA
TIME, SPEED AND DISTANCE 3.7

5. If the speed of a boat in still water is x km/hr. and the speed of the stream is y km/hr Speed

THEORY
while travelling with the stream i.e., speed downstream = (x + y) km/hr.
Speed while travelling against the stream i.e., speed upstream = (x – y) km/hr.
1
6. Also, speed of the boat in still water = 2 (Speed with stream + Speed against stream)

PRACTICE EXERCISE CLASS EXERCISE


1
Speed of the river = 2 (Speed with stream – Speed against stream)

SOLVED EXAMPLES

Q : I row from A to B against the current in 8 hrs. and from B to A in 2 hrs. If the speed of
the river is 9 m/sec., what is the speed of the boat in still water?
A : Let the speed of the boat in still water be x
Time taken for up - journey
=8=4
Time taken for down - journey 2 1
Speed for up - journey
 = 1 (inverse)
Speed for down - journey 4
x-9 1
 x + 9 = 4 (x is the speed of the boat)
 3x = 45  x = 15 m/sec.

Q : A train travelling at 25 km/hr, leaves Delhi at 9 a.m. and another leaves Delhi at 35 km/hr.
at 2 p.m. in the same direction. How many kms from Delhi do they meet?
A : The first train has a start of 5 × 25 = 125 km. Relative speed = 35 – 25 = 10 km/hr.

Distance = 125 = 12.5 hrs. from 2 p.m.


 Time taken to meet = 10
Relative speed

 Distance from Delhi = 12 1 × 35 = 437 1 kms.


2 2

Q : A man rows 27 km. with the stream and 15 km. against the stream taking 4 hrs each time.
Find his speed in km/hr in still water and the speed in km/hr at which the stream flows.
27 = 3
A : Speed with the stream = 4 6 4 km/hr.
15 = 3
 Speed against the stream = 4 3 4 km/hr.

 Speed of the man in still water = 2 b6 4 + 3 4 l = 5 4 = km/hr.


1 3 3 1

 Speed of the stream = 2 b6 4 - 3 4 l = 1.5 km/hr.


1 3 3

175
CATapult
ARITHMETIC

Q : Two trains 121 metres long and 99 metres long are running in opposite directions, the first
at 40 km/hr. and the second at 32 km/hr. In what time will they completely clear each
other from the moment they meet?
A : Total distance to be travelled = 121 + 99 = 220 metres.
5
Relative speed = Sum of speeds = 72 km/hr. = 72 × 18 = 20 m/s.
220
 Time required = 20 = 11 seconds.

Q : How long does a train 110 metres long running at 36 km/hr. take to cross a bridge 132
metres in length?
A : Distance to be covered = 110 + 132 = 242 metres
5 242
Speed = 36 × 18 = 10 m/sec.  Time taken = 10 = 24.2 seconds.

Q : A car which was driven in fog passed a man walking at 3 km/hr. in the same direction. He
could see the car for 4 minutes and upto a distance of 100 m. What was the speed of the
car?
4 12000
A : Distance travelled by the man in 4 mins. = 60 × 3000 = 60 = 200 m.
Distance travelled by the car in 4 mins. = 200 + 100 = 300 m.
300 300 1
 Speed of the car = 4 m/minute = 4 × 1000 × 60 = 4 2 km/hr.

1
Q : A person can row 7 2 km/hr. in still water. It takes him twice as long to row up a distance
as to row down the same distance. Find the speed of the stream.
1
A : Speed up-stream + Speed down-stream = 2 × 7 2 = 15 km/hr.
Since the times taken are in the ratio 2 : 1, the speeds will be in the ratio 1 : 2.
1
 Speed up-stream = 3 × 15 = 5 km/hr.
2
Speed down-stream = 3 × 15 = 10 km/hr.
1
Speed of stream = 2 (10 – 5) = 2.5 km/hr.
Alternatively,
1 1
72 +x 72 -x
If the speed of the stream is x km/hr., = 2
=
1
 x = 2.5 km/hr.

176
Chapter QA
TIME, SPEED AND DISTANCE 3.7

Q : A hare sees a dog 100 metres away from her, and scuds off in the opposite direction at a

THEORY
speed of 12 km/hr. A minute later, the dog sees the hare and chases the hare at a speed
of 16 km/ hr. After how much time does the dog catch up with the hare?
1000
A : 12 km/hr. = 12 × 60 = 200 metres/min.

PRACTICE EXERCISE CLASS EXERCISE


Distance of the hare from the dog when the dog sees the hare
= (100 + 1 × 200) = 300 metres
Since both are running in the same direction,
1000 200
Relative speed (16 – 12) = 4 km/hr. = 4 × 60 m/min. = 3 m/min.
300
 Time required to overtake = 200 = 300 × 3 = = 4½ mins.
3 200

Q : A train leaving L at 3:10 p.m. reaches W at 5:00 p.m. One leaving W at 3:30 p.m. arrives in
L at 5:50 p.m. At what time do they pass each other?
A : Let the distance be d and let them meet t mins after 3:10 p.m. or (t – 20) mins. after 3:30p.m.
3:10 p.m. d km. 5:00 p.m.

L t W
5:50 p.m. 3:30 p.m.

d d
Their speeds are 110 km/mins. and 140 km/mins.
d d t t - 20
 Distance = 110 × t + 140 (t – 20) = d  110 + 140 = 1
Solving t = 70.4 mins.
 They meet at 3:10 + 70.4 mins. = 4 hrs 20.4 mins.

Q : A train moving at uniform speed takes 20 secs. to pass a cyclist riding in the same
direction at 11 km/hr but only 9 secs. to pass a post. Find the length of the train.
A : If the length of the train is , km. and its speed x km/hr., then,
, = 20
x - 11 3600 ... (i)
and
, 9
x = 3600 ... (ii)
, x = 20 x 20
Dividing (i) by (ii), x - 11 × , 9  x - 11 = 9
 x = 20 km/hr.
 Length = 0.05 km = 50 metres.

177
CATapult
ARITHMETIC

Concept Builder 2

1. Two trains which are running in oppositite direction meet each other after 2 mins, total dis-
tance travel by both trains is 1.8 km. Find average of speed of both trains (in m/s).
2. A goods train runs at the speed of 54 km/hr and crosses a 110 metres long platform in 20
seconds. What is the length of the goods train?
3. Two trains running at the rate of 30 kmph and 36 kmph respectively in the same direction.
The length of second train is 130 metres and the time taken by them to cross each other
is 150 seconds (from the time they meet). Then the length of first train is:
4. Two trains of 400 m and 475 m in length runs at the speed of 45 km/hr and ‘X’ km/hr
respectively in opposite directions on parallel tracks. The time taken by them to cross each
other (from the time they meet) is 42 seconds, then find ‘X’.
5. A man can row at a speed of 8 km/hr in still water to a certain upstream point and back
to the starting point in a river which flows at 4 km/hr. Find his average speed for the total
journey.
6. A man can row upstream at 7 km/hr and downstream at 15 km/hr. Then the speed of the
stream is:
7. A boat takes 7 hours for travelling downstream from point ‘A’ to point ‘B’ and coming back
to point ‘A’ travelling upstream. If the speed of the stream is 3 kmph and the speed of the
boat in still water is 7 kmph, what is the distance between A and B?
8. The speed of a river is 3 kmph. If a man takes twice as long to row up as to row down the
river, the rate of man in still water is:

Answer Key

9 kmph 8.
20 km 7.
4 kmph 6.
6 km/hr. 5.
30 m/s. 4.
120 meters. 3.
190 metres. 2.
7.5 m/s. 1.

178
Chapter QA
TIME, SPEED AND DISTANCE 3.7

CLASS EXERCISE

Teaser

“That clock is going to strike noon”


Old Grandpa said to little Pete
“The two hands will be meeting soon
How often, each day, will they meet?”

“Twelve times, of course!” said Pete at once


Grandpa replied “No - try once more?”
“Oh dear” cried Pete “I’m such a dunce
The answer should be twenty-four!”

Is Pete correct?

179
CATapult
ARITHMETIC

Time, Speed and Distance: Basics


1. Raghu walks at 4 kmph and reaches his school in 1hr 45 min. How far is his school?
2. Ramu strolls at 4 kmph to reach his school, 1.4 km away. How long will he take?
3. Raju takes 4 sec to cover certain distance at 72 kmph. What is the total distance covered
by him?
4. A man goes for a jog at 6 a.m. He jogs for 15 minutes and covers 2.8 km. He then walks
backs to his starting point at a speed 25 % lower than his jogging speed. At what time will
he reach back?
5. Akbar, Babar and Chandragupta set out for a certain place. Akbar reached in 20 minutes at
45 kmph. If Babar and Chandragupta reached 5 and 10 minutes later than Akbar respectively,
find their speeds.
6. “The Great Indian Endurance Rally” is a new TV show which requires participants to run,
cycle and swim. Jignesh swims for 3 hours covering 4 km. He then cycles for an hour at 12
kmph and runs 20 km in 8 hours. What is his average speed over the entire event?
7. At 7:42 a.m., Lucky gets into a bus to go to school. The bus travels at 42 kmph, and drops
Lucky a short way away from school at 7:51 a.m. He then walks the remaining distance at 6
kmph, thereby reaching school exactly at 8 a.m. What is his average speed for the journey?
8. In a 2-man relay race, the first man covers his 400 m stretch at 18 kmph while the second
covers his 400 m stretch at 22 kmph. What is their average speed for the entire race?

Distance
Time, Speed and Distance are related by the formula Speed =
Time
Total Distance
Average speed can be found, in general, by the formula Average Speed =
Total Time
Specifically, if the time is the same in two legs of a journey, the average speed is the Arithmetic
Mean while if the distance is the same in two legs of a journey, the average speed is the
Harmonic Mean
5 18
Some useful conversions: 1 kmph = 18 m/s, 1 m/s = 5 kmph, 1 mile = 1.6 km, 1 km =
1000 m

9. * A boy reaches his school, 1.4 km away, in 1hr 40 min. At what speed does he travel?
10. * A boy leaves for school at 10:40 a.m. He spends 5 hrs in school and comes back at 5:20
pm on the same day. He travels at a constant speed of 1.4 km/hr. How far away is his
school?
11. * A boy travels at 4 kmph to reach school. How many metres does he travel per second?
12. * A boy travels to school at 4 kmph and returns back at the speed of 6 kmph. What is his
average speed?
13. Ravi usually takes 1.4 hrs to reach school. If he walks at 75% of his usual speed, how much
time will he take?
14. If Raj walks at 5/4th of his usual speed, he reaches 10 min early. How long does he usually
take?

180
Chapter QA
TIME, SPEED AND DISTANCE 3.7

15. Akbar and Birbal start towards each other from their homes, 15 km apart, at 10 a.m. Akbar

THEORY
walks at a constant speed of 4 kmph, while Birbal walks at a constant speed of 6 kmph.
When will they meet?
16. In the previous question, after Akbar and Birbal meet, they chat for half an hour and then

PRACTICE EXERCISE CLASS EXERCISE


leave for their respective homes. At what time will they reach home?
17. Daisy Duck and Donald Duck start moving towards each other from their homes, 24 km apart,
at noon. Daisy walks at a constant speed of 5 kmph, while Donald walks at a constant speed
of 7 kmph. How far away from each other will they be 10 minutes before they meet?
18. Ricky snatches Vicky’s wallet and starts running away at a speed of 6 m/s. After 5 seconds,
Vicky starts chasing Ricky at 8 m/s. After how much more time will Ricky be caught?
19. Mike is showing off his new car. He drives along the road at a speed of 20 m/s, overtaking a
truck 12 m long which is moving in the same direction with a speed of 15 m/s. If he takes
3 seconds to overtake the truck, what is the length of his car?

Relative Speed:
The relative speed of two objects is the speed with which one object moves with respect to
the other.
When two objects are moving in opposite directions, their relative speed is the sum of their
speeds
When two objects are moving in the same direction, their relative speed is the difference of
their speeds
However, the lengths of the objects involved always add!

20. * One fine day Anil started late by half an hour for work. By what % should he increase his
speed to reach office in time if he usually takes 2 hrs to reach office?
21. * The ratio of time taken by P and Q is 7: 9 to cover the same distance. What is the ratio
of their speeds?
22. * A bus 6 m in length takes two seconds to pass a truck, 8 m long, going in the opposite
direction. If the bus driver is driving at 4m/s, at what speed is the truck moving?
23. * A deer sees a tiger 100m away and starts running away from it at the speed of 15 mps.
The tiger sees the deer 5 seconds later and pursues it at the speed of 40 mps. How much
later will the tiger catch the deer?

Directions for questions 24 – 26: Each of the following sets of questions is based on an independent
scenario. Information given in a question is valid for all later questions in the set.

24. A 180 m long train is travelling at 36 kmph:


a) How long will it take to pass an electric pole 6 m tall?
b) If it is initially 100 m away from a signal, at what time will it have completely passed
the signal?
c) How long will it take to completely pass a 300 m long platform?
d) A man is standing in the middle of the above platform. How long will the train take to
cross him?

181
CATapult
ARITHMETIC

25. The Howrah Mail is a 180 m long train that travels at 54 kmph and the Coromandel Express
is a 200 m long train that travels at 72 kmph. These trains run on parallel tracks.
a) How long will the Howrah Mail take to cross the Coromandel Express if they are running
towards each other?
b) How long will the Howrah Mail take to cross the Coromandel Express if they are running
in the same direction?
c) How long will the Coromandel Express take to cross the Howrah Mail if they are running
in the same direction?
d) At 10 a.m., the Coromandel Express is 100 m behind the Howrah Mail and running in the
same direction. At what time will it have completely crossed the latter?

26. A boat travels at the speed of 18 kmph in still water. It travels between points A and B, 80
km apart.
a) If the boat goes from A to B, downstream in 4 hours, how long will it take to return?
b) What is the normal speed of the current in the river?
c) If the speed of the river triples one day due to a rainstorm, how long will the round trip
from A to B and back take on that day?
d) If the boat develops engine trouble and can consequently go at only one-third of its normal
rate in still water, how long will the round trip take on a normal day?
e) At what speed must the current flow so that the round trip would take 22.5 hours with
the boat in normal condition?

27) * If a train 200 m long crosses a bridge 350 m long in 11 seconds, what is the speed of
the train?

28) * Find the time taken by a 240-foot roller-coaster moving at 56 ft/sec to pass through a
40-foot tunnel.

182
Chapter QA
TIME, SPEED AND DISTANCE 3.7

Challengers

THEORY
1. Wilson starts for office at the same time every day. If he walks at 5 kmph he is 2 min late.
If he walks at 10 kmph he is 4 min early. Find his correct speed to reach on time, and the

PRACTICE EXERCISE CLASS EXERCISE


distance to his office.
1) 6 kmph, 1 km 2) 6.33 kmph, 0.8 km
3) 6 kmph, 0.8 km 4) 7.5 kmph, 1 km

2. Prashant can row 7 1 kmph in still water. It takes him twice as long to row upstream as to
2
row downstream. Find the speed of the stream.
1) 2.5 kmph 2) 5 kmph 3) 3 kmph 4) 1.5 kmph

3. A train had engine trouble and hence its speed got reduced to 2/3rd of normal. It reached
the destination at 3:12 p.m. Had the engine trouble occurred 48 km further on, the train
would have reached at exactly 3 p.m. What is the normal speed of the train?
1) 60 kmph 2) 80 kmph 3) 100 kmph 4) 120 kmph

4. Two trains A and B, whose speeds are in the ratio 4 : 3, cross each other in 18 seconds
when they travel in opposite directions. Train A crosses a pole in 24 seconds. What is the
time taken (in seconds) by train B to cross a pole?

183
CATapult
ARITHMETIC

PRACTICE EXERCISE-1

Directions for questions 1 to 3: Solve as directed.

1. Rohan travels at 40 kmph for 2 hrs, 60 kmph for the next 2 hrs and 80 kmph for the last
2 hrs. Find the average speed for the entire trip?
2. P covers a certain distance in 4.8 hrs. Q covers twice the distance in 8 hours. How much
time will P take to cover the distance that Q covers in 4.5 hours?
3. Raman covers 3/7th of the distance by train, 5/14th of the distance by car and the remaining
9 km on foot. What is the total distance that he covered?

Directions for questions 4 to 17: Choose the correct alternative.

4. A train travelling at 42 km/hr. passes a cyclist going in the same direction in 9 secs.; if the
cyclist had been going in the opposite direction, the train would have passed him in 5 secs.
Find the length of the train.
1) 75 metres 2) 60 metres 3) 90 metres 4) 80 metres

5. A train overtakes 2 persons walking at 2 km/hr and 4 km/hr respectively in the same direc-
tion and completely passes them in 9 seconds and 10 seconds. Find the length of the train
and its speed in km/hr.
1) 75 m, 18 km/hr 2) 80 m, 21 km/hr
3) 60 m, 20 km/hr 4) 50 m, 22 km/hr

6. A policeman sees a thief 300m away and starts chasing him at a speed of 16 kmph. The
thief is running at a speed of 12 kmph but after 4 minutes, the thief gets exhausted and
falls down. What is the distance between the two now?
1) 10 m 2) 25 m 3) 33.33 m 4) 66.67 m

7. A man rows upstream 13 km. and downstream 28 km. taking 5 hrs. each time. Find the
speed of the current.
1) 2 km/hr. 2) 1.5 km/hr. 3) 3 km/hr. 4) 3.5 km/hr.

8. A place B lies exactly midway on the path from A to C such that all the places are connected
by roadway, waterway and railway. The average speeds by road, water and rail are 10 kmph,
5 kmph and 15 kmph respectively. Which of the following routes will take the maximum time
to reach C from A?
1) A to B by road and B to C by road
2) A to B by water and B to C by rail
3) A to B by rail and B to C by road
4) A to B by road and B to C by water

184
Chapter QA
TIME, SPEED AND DISTANCE 3.7

9. Two trains, both 120 metres in length, move in the same direction. The faster train completely

THEORY
overtakes the slower one in 15 seconds. If the slower train were to move at half its speed,
the overtaking would take 10 seconds. At what speeds are the two trains moving (in mps)?
1) 48 and 16 2) 32 and 16 3) 48 and 32
4) 64 and 48 5] 64 and 32

CLASS EXERCISE
10. Two trains A and B start from two points P1 and P2 respectively at the same time and travel
towards one another. The difference between their speeds is 10 kmph and train A takes one
hour more to cover the distance between P1 and P2 as compared to train B. Also by the time
200

PRACTICE EXERCISE
they meet, train B has covered 9 km more as compared to train A. What is the distance
between P1 and P2?
1) 150 km 2) 200 km 3) 250 km 4) Data insufficient

11. A train travelling at 10 m/sec from A to B at 7 a.m. meets a train leaving B at 7:20 a.m.
1
and coming to A at a speed 3 times faster than the first train. If the distance from A to B
is 68 km. then, at what distance from A will the two trains meet?
1) 72 km 2) 36 km 3) 60 km 4) 50 km

12. A car after travelling 18 km from a point A developed some problem in the engine and its
speed became 4/5 of its original speed. As a result, the car reached point B 45 minutes
late. If the engine had developed the same problem after travelling 30 km from A, then it
would have reached B only 36 minutes late. The original speed of the car (in km per hour)
and the distance between the points A and B (in km) is _____.
1) 25, 130 2) 30, 150 3) 20, 90 4) None of these
(Past CAT question)

13. Two trains are travelling in opposite direction at uniform speed 60 and 50 km/hr respec-
tively. They take 5 seconds to cross each other. If the two trains had travelled in the same
direction, then a passenger sitting in the faster moving train would have overtaken the other
train in 18 seconds. What are the lengths of trains (in metres)?
1) 112.78, 45 2) 97.78, 55 3) 102.78, 50 4) 102.78, 55
(Past CAT question)

14. A man travels from A to B at a speed of x km/hr. He then rests at B for x hours. He then
travels from B to C at a speed of 2x km/hr and rests for 2x hours. He moves further to D
at a speed equal to twice of that taken to travel between B and C. He thus reaches D in 16
hours. If the distances A-B, B-C, C-D are all equal to 12 km, the time for which he rested
at B could be _____.
1) 3 hours 2) 6 hours 3) 2 hours 4) 4 hours
(Past CAT question)

185
CATapult
ARITHMETIC

15. The distance between A and B is 72 km. Two men started walking from A and B at the same
time towards each other. The person who started from A travelled uniformly with an average
speed of 4 km/hr. The other man travelled with varying speeds as follows: In the first hour
his speed was 2 km/hr, in the second hour it was 2.5 km/hr, in the third hour it was 3 km/
hr, and so on. When will they meet each other?
1) 7 hours 2) 10 hours
3) 35 km from A 4) Midway between A and B
(Past CAT question)

16. A train travelled a certain distance at a uniform speed. Had the speed been 8 kmph more, the
journey would have taken 3 hours less and had the speed been 10 kmph less, the journey
would have taken 6 hours more. Find the distance travelled by the train.
1) 480 km 2) 720 km 3) 800 km 4) 640 km

17. A boat sails 24 km downstream of a river stretch in 3 hours. How long will it take to cover
the same distance upstream, if the speed of the current is one-third the speed of the boat
in still water?
1) 4 hours 2) 6 hours 3) 8 hours 4) 12 hours

Directions for questions 18 to 20: Solve as directed.

18. A motorboat travels 23 kmph in still waters. If it goes downstream from X to Y, a distance
of 120 km, in four hours, how long will it take to return?

19. The life guards at Aksa beach spotted an unconscious man flowing downstream along the
water towards the beach at 1m/s. They immediately set sail on their power boat upstream at
a speed of 25m/s. As soon as they reached the man, they stopped the boat. It took them 14
seconds to bring him to consciousness inside their boat while the engine of the boat stopped
and the boat itself was flowing downstream towards the shore at the speed of the water
current. Immediately after the man regained his conciousness, they sailed at their original
speed downstream and were able to return to the shore in 41 seconds. At what distance
had the lifeguards initially spotted the man?

20. At 7:00 AM, two brothers Ajay and Vijay started from their home to their school to appear
for an examination. Ajay walked at the speed of 6 km/hr while Vijay rode a bicycle at the
speed of 18 km/hr. On reaching school, Vijay realized that he forgot his hall-ticket. So he
immediately headed back home riding at the same speed. On his way back, at 7:15 AM, he
met Ajay, who was on his way to school. What is the distance between the home and the
school (in metres)?

186
Chapter QA
TIME, SPEED AND DISTANCE 3.7

PRACTICE EXERCISE-2

Directions for questions 1 and 2: Solve as directed.

1. A boy travels for 40 minutes at the speed of 4 km per hour and the next 40 minutes at the
speed of 6 kmph. What is his average speed?

2. The ratio of speeds of two brothers Jenish and Jinesh is 3: 2. Jinesh takes 20 min more
than Jenish to walk from home to school. If Jinesh walks at twice his usual speed, find the
time he will take to walk from home to school.

Directions for questions 3 and 4: Refer to the data below and answer the questions that follow.

It takes Kim 4 hours to drive from her house to her office at her normal speed. If she reduces
her speed by 3kmph, she takes 40 minutes extra.

3. Find the distance between Kim’s house and her office.


1) 24 km 2) 84 km 3) 93 km 4) None of these

4. If Kim’s car breaks down after 3 hours, while driving at her normal speed, and then she
decides to walk to the office, what should be her walking speed so that she is just 2 hours
late to the office than the anticipated time?
1) 3.5 kmph 2) 5 kmph 3) 3 kmph 4) 7 kmph

Directions for questions 5 to 17: Choose the correct alternative.

5. The speeds, of a sports car in four different practice runs on the same track, are 100, 120,
80 and 120 miles/hour. Find its average speed (approximately) in miles/hour.
1) 100 2) 102 3) 96 4) 110

6. Two trains A and B of length 88 yds. and 132 yds. respectively, are approaching each other
from opposite directions at 60 and 45 mph, respectively. How long do they take to pass each
other?

b1 yard = 3 feet, 15 sec. l


miles = 22 feet
hr

1 2 3
1) 5 secs. 2) 3 4 sec. 3) 4 7 secs. 4) 6 8 secs.

187
CATapult
ARITHMETIC

7. A train 150m long travels at a speed of 100 kmph. A man standing on top of the train runs
from one end of the train to the other at a speed of 20 kmph in the same direction as the
train. How many seconds will he take to reach the other end?
1) 27 2) 22.5 3) 2.25 4) None of these

8. A policeman, who is 80m away from a thief, starts chasing him. The thief is running at a
speed of 10m/s. After exactly 8 second, the distance between them is 40m. Find the po-
liceman’s speed.
1) 10.5 m/s. 2) 16.67 m/s. 3) 15 m/s. 4) None of these

9. If the current flows at 2 mph and it takes me 3 hrs. to row 9 miles upstream, how long will
it take to return?
2 5 2 5
1) 1 7 hrs. 2) 2 7 hrs. 3) 2 3 hrs. 4) 1 7 hrs.

10. Train A started from Mumbai to Calcutta. At the same time, train B started from Calcutta
to Mumbai. The two trains are sent for a testing wherein they travelled at constant speed,
reached the destination and returned to the source without halting. The trains meet each
other for the first time in 18 hours after starting. The two trains reached their respective
sources i.e., A to Mumbai and B to Calcutta in 17 hours 4 minutes and 19 hours 16 minutes
respectively after they cross each other for the second time. After how many hours from the
starting point, did they cross each other second time?
1) 36 hours 2) 45 hours 3) 54 hours 4) 72 hours

11. A hunter sees a deer 200m away and immediately shoots an arrow which can travel at a
speed of 30 m/sec for 300 metres after which it drops down (immediately). At what speed
should the deer run so as to miss the arrow?
1) more than 10 m/sec 2) more than 30 m/sec
3) more than 5 m/sec 4) None of these

12. I take 4 hours less to row down a 12 mile stream than I take to row upstream. For this 24
mile roundtrip, if I double my rowing speed, I would take half an hour less to row down-
stream than to row upstream. Find the speed of the stream in miles/hr.
1) 6 2) 8 3) 0 4) 10 5) None of these

13. Two trains A and B start from stations X and Y towards each other. B leaves station Y half an
hour after train A leaves station X. Two hours after train A has started, the distance between
19
trains A and B is 30 th of the distance between stations X and Y. How much time would it
take each train (A and B) to cover the distance X to Y, if train A reaches half an hour later
to its destination as compared to B?
1) 8 hrs, 7 hrs 2) 5 hrs, 4 hrs 3) 10 hrs, 9 hrs 4) 9 hrs, 8 hrs

14. A man starts cycling at 12.00 a.m. at a speed of 5 kmph. A train, A, starts running on a
track with a speed of 100 kmph at 5.00 a.m. Another train, B, starts running on the adjacent
parallel track with a speed of 120 kmph at 7.00 am from the same station and in the same
direction. Find the distance travelled by the man when the two trains meet.
1) 68 km 2) 85 km 3) 90 km 4) 92 km

188
Chapter QA
TIME, SPEED AND DISTANCE 3.7

15. Every day Neera’s husband meets her at the city railway station at 6.00 p.m. and drives her

THEORY
to their residence. One day she left early from the office and reached the railway station at
5.00 p.m. She started walking towards her home, met her husband coming from their resi-
dence on the way and they reached home 10 minutes earlier than the usual time. For how
long did she walk?

CLASS EXERCISE
1
1) 1 hour 2) 50 minutes 3) 2 an hour 4) 55 minutes
(Past CAT question)

16. I started climbing up the hill at 6 a.m. and reached the temple at the top at 6 p.m. The

PRACTICE EXERCISE
next day I started coming down at 6 a.m. and reached the foothill at 6 p.m. I walked on the
same road. The road is so short that only one person can walk on it. Although I varied my
pace along the way, I never stopped on my way. On the basis of this, which of the following
must be true?
1) My average speed downhill was greater than that uphill.
2) At noon, I was at the same spot on both the days.
3) There must be a point which I reached at the same time on both the days.
4) There cannot be a spot which I reached at the same time on both the days.
(Past CAT question)

17. A bus started from the bus stand at 8.00 a.m. After staying for 30 minutes at its destination,
it returned back to the bus stand. The destination is 27 miles from the bus stand while the
speed of the bus was 18 mph. During the return journey, the bus traveled 50% faster than
the original journey. At what time did it return to the bus stand?
1) 11:30 a.m. 2) 11:00 a.m. 3) 12:30 p.m. 4) 12:00 noon

Directions for question 18: Solve as directed.

18. If the speed of a car is increased by 20 km/h, it reaches its destination 2 hours earlier. If
the speed is further increased by 20 km/h, the car reaches 3 hours earlier than its usual
time. What will be the speed (in km/h) of the car if it reaches 2 hours late?

Directions for questions 19 and 20: Choose the correct alternative.

19. A hare and a tortoise decided to race each other. They began running at the same time.
After the hare gained a lead of 35 m, he stopped to rest while waiting for the tortoise to at
least reach the point where he had reached. But he lost track of time and when he woke up
2
after 4 3 minutes, he saw that the tortoise had gained a lead of 35 m. So he immediately
started running and lost the race by 7 m. What is the ratio of the time that the hare ran
before going to sleep and the time he ran after waking from sleep?
1) 4 : 5 2) 5 : 4 3) 1 : 1 4) Data insufficient

189
CATapult
ARITHMETIC

20. A cruise carries passengers from the port to a certain point upstream and brings them back
to the port. It travels upstream for 1.5 hours and downstream for half an hour. However,
on one occasion, after travelling 1.5 hours upstream, the ship developed a technical error,
because of which its speed reduced. At the same time, the speed of the stream suddenly
increased due to a thunderstorm. The ship managed to return to the port in time. What was
the ratio of the % increase in the speed of the stream to the % decrease in the speed of
the ship?
1) 1 : 2 2) 2 : 1 3) 3 : 2 4) 3 : 4

190
CATapult
ARITHMETIC

QA-3.8 WORK, PIPES AND CISTERN THEORY

Work
When we say, A has worked for an hour, we actually refer to the amount of job that is done in
one hour. The same job can probably be done by B in less time, if the speed of doing the job is
more than that of A. Therefore, we can say B is more efficient than A.
This chapter introduces you to the different concepts in Work and problems based on the application
of the concepts.

Definition
Work is defined as the amount of job assigned or the amount of job actually done.
Work is always considered as a whole or 1. There exists an analogy between the time-speed-
distance problems and work.
Work  1  Distance.
Rate at which the work is done = Speed
Number of days required to do the work = Time

Rules and Formulae for Work related problems


1
1. If A can do a piece of work in ‘a’ number of days, then in one day th of the work is done.
a
1
Conversely, if a man does th of a work in 1 day, then he can complete the work in 1 ÷
a
1
= a days.
a
1
2. If A is ‘x’ times as good a workman as B, then he will take th of the time taken by B to
x
do the same work.

3. If A and B can do a piece of work in ‘x’ and ‘y’ days respectively, then working together,
xy x+y
they will take days to finish the work and in one day, they will finish th part of
+
x y xy
the work.

4. To compare the work done by different people, first find the amount of work each can do in
the same time.

5. If the number of men to do a job is changed in the ratio a : b, then the time required to
do the work will be in the ratio b : a, assuming the amount of work done by each of them
in the given time is the same, or they are identical.

6. If two men A and B together can finish a job in ‘x’ days and if A working alone takes ‘a’
days more than A and B working together and B working alone takes ‘b’ days more than A
and B working together then x = ab .

7. To do a piece of work, the number of men employed and the number of days required to do
the work are in inverse proportion, also, the number of men employed and the hours worked
per day are in inverse proportion.

191
CATapult
ARITHMETIC

SOLVED EXAMPLES

Q : A group of labourers do a piece of work in 10 days, but five of them are absent and so the
rest do the work in 12 days. Find the original number of labourers.
A : More men, less days

x 12
=  12x – 60 = 10x  x = 30 men
x–5 10

Q : If 3 men or 5 women take 26 days to do a work, how long will 7 men and 10 women take?
A : 10 women are equivalent to 6 men.
More men, fewer days

3
x = 26 × = 6 days
13

Q : If 8 men can reap 80 hectares in 24 days, how many hectares can 36 men reap in 30 days?
A : More men, more hectares; more days, more hectares.

x 36 30
= ×
80 8 24
36 30
 Number of hectares = 80 × × = 450
8 24

Q : If 30 men working 7 hours per day can do a work in 18 days in how many days will 21 men
working 8 hours a day do the same work?
A : Fewer men, more days; more hrs., fewer days.

x 30 7
= ×
18 21 8
30 7
Number of days = 18 × × = 22.5
21 8

192
Chapter QA
WORK, PIPES AND CISTERN 3.8

Q : Two men and 7 boys can do a piece of work in 14 days. 3 men and 8 boys can do it in 11

THEORY
days. In how many days can 8 men and 6 boys do a work 3 times as big as the first?
A : 2 men + 7 boys in 14 days  28 men + 98 boys in 1 day
3 men + 8 boys in 11 days  33 men + 88 boys in 1 day

PRACTICE EXERCISE CLASS EXERCISE


 28 men + 98 boys = 33 men + 88 boys  2 boys  1 man
Now, 2 men + 7 boys = 11 boys; 8 men + 6 boys = 22 boys
More boys, fewer days; more work, more days

x 11 3
 = ×  Number of days = 21 days.
14 22 1

Q : A, B and C can do a work in 6, 8 and 12 days respectively. B and C work together for 2 days,
then A takes C’s place. How long will it take to finish the work?

A : Work done by B and C in 2 days = 2 b


8 + 12 l 20 5
– 2 × = th part.
96 96 12
7
Remaining work = th part.
12
6+8 14 7
Work done by A and B in 1 day = = =
48 48 24
7 7
 Number of days required = ÷ = 2 days.
12 24

Q : 2 women, A and B can mow a field in 8 and 12 hours respectively. They work for an hour
alternately, A beginning at 9 a.m. When will the work be completed?

A : In the first 2 hours, A and B will mow b l = 5 th of the field.


1 + 1
8 12 24
10 15 20 4
In 4 hrs, th, in 6 hrs th, in 8 hrs th and now the work left is th.
24 24 24 24
1 1 1
Now, A works for 1 hour and does th work; Work left = th which B can do in hr.
8 24 2
1 1
 Total time taken = 8 + 1 + = 9 hrs.
2 2
So the work will be over at 6:30 p.m.

Q : To do a piece of work, B takes 3 times as long as A and C together, and C twice as long as
A and B together. If the three together can complete the work in 10 days, how long would
each take by himself?
A : 3 times B’s daily work = (A + C)’s daily work.
1
 4 times B’s daily work = (A + B + C)’s daily work = (adding B’s daily work to both
10
sides)

193
CATapult
ARITHMETIC

1
 B’s daily work = th  B takes 40 days.
40
Similarly, 2 times C’s daily work = (A + B)’s daily work
1
 3 times C’s daily work = (A + B + C)’s daily work =
10
1
 C’s daily work = th work.  C takes 30 days.
30

– b l = 1 – 7 = 5
1 1 + 1
 A’s daily work =
10 40 30 10 120 120
120
 A takes = 24 days.
5

Concept Builder 1
1. P does the job in 11 days while Q takes 12 days and R takes 22 days for the same job.
P and Q started working together. But, after two days they left and R had to complete
the job. Find the number of days required to complete the job?
2. A and B can do a piece of work in 10 days, B and C in 12 days, and C and A in 15 days.
In how many days will they finish it together?
3. A can do a piece of work in 10 days. If B is 25% more efficient than A, then find the
number of days required by B to do the same piece of work.
4. A and B can do a piece of work in 12 days. B and C can do it in 15 days; A and C can
do it 20 days. Who among these will take the least time if put to do it alone?
5. A can do a piece of work in 7 days working 9 hours each and B can do it in 6 days work-
2
ing 7 hours each. How long will they take to do the work together, working 8 hours a
5
day?
6. A can do a piece of work in 15 days which B can do in 18 days. In how many days will
they finish the work, both working together?
7. A and B together complete a piece of work in 40 days while B alone can complete the
same work in 60 days. A alone will be able to complete the same work in:
8. X is half as good a workman as Y and together they finish a piece of work in 24 days.
The number of days taken by X alone to finish the work is:
9. P can do a piece of work in 10 days, which Q can finish in 15 days. If they work at it on
alternate days with P begining in how many days the work will be finished.

Answer Key
9. 12 days.
11
8. 72 days. 7. 120 days. days. 6. 8 5. 3 days.
2
3
4. B 3. 8 days. 2. 8 days. days. 1. 16
1

194
Chapter QA
WORK, PIPES AND CISTERN 3.8

Pipes & Cisterns

THEORY
In the previous section, you have learnt about the concept of work. Pipes and Cisterns is a special
application of the same. Filling or emptying a cistern can be considered as work done.
This section introduces you to problems based on work with relation to pipes and cisterns.

PRACTICE EXERCISE CLASS EXERCISE


A pipe connected with a cistern is called an inlet, if it fills the cistern.
A pipe connected with a cistern is called an outlet, if it empties the cistern.

Important formulae for pipe & cisterns related problems

1
1. If an inlet pipe fills a cistern in ‘a’ hours, then th part is filled in 1 hour.
a
The concept of negative work implies work that is destructive in nature. For example, if an
1
outlet pipe empties a cistern in ‘a’ hours, then th part is emptied in 1 hour. In this case,
a
the outlet pipe is doing negative work.

1
2. If pipe A is ‘x’ times bigger than pipe B, then pipe A will take th of the time taken by
x
pipe B to fill the cistern.

mn
3. If A and B fill a cistern in ‘m’ and ‘n’ hours, respectively then together they will take
m+n
m+n
hours to fill the cistern and in one hour th part of the cistern will be filled.
mn

Similarly, ‘A’ and ‘B’ empty a cistern in ‘m’ and ‘n’ hours, respectively, then, together they
m+n m+n
will take hours to empty the cistern and in one hour th part of the cistern will
mn mn
be empty.

4. If an outlet pipe empties the cistern in ‘n’ hours and an inlet pipe fills a cistern in ‘m’ hours

then the net part filled in 1 hour when both the pipes are opened is b l i.e., n – m
1 1
m – n mn
and the cistern will get filled in b l hours.
mn
n–m
For the cistern to get filled, it is necessary that m < n. If m > n, the cistern will never get
filled.
In general, Net part filled of a cistern = (Sum of work done by inlets) – (Sum of work done
by outlets)

5. If an inlet pipe fills a cistern in ‘a’ minutes, takes ‘x’ minutes longer to fill the cistern due
to a leak in the cistern, then the time in which the leak will empty the cistern is given by a

× b1 + l .
a
x
6. If two pipes A and B can fill a cistern in ‘x’ minutes and if A alone can fill it in ‘a’ minutes
more than ‘x’ minutes and B alone can fill it in ‘b’ minutes more than ‘x’ minutes then x =
ab .

195
CATapult
ARITHMETIC

SOLVED EXAMPLES

Q : A tank 9 ft. by 5 ft. by 2 ft. has a supply pipe pouring in 576 in3 of water in a minute and
an exhaust pipe emptying it in 3 hours. If the tank is full, and both pipes are open, how many
hours will it take to empty it?
A : Volume of the tank = 9 × 5 × 2 × 12 × 12 × 12 in3

90 × 12 × 12 × 12
Volume of water exhausted in 1 min = = 864 in3
3 × 60
 The combined effect of the two pipes in 1 min is (864 – 576)
i.e. 288 in3 of water is removed in 1 min.

90 × 12 × 12 × 12
 Time required to empty the tank = = 9 hrs.
288 × 60

Q : Pipes A and B can fill a cistern in 20 and 30 minutes and C can empty it in 15 minutes. If
the three are opened and closed one after the other successively for 1 min each in that order,
how soon will the cistern be filled?

1 1 1 1
A : Part filled in 3 min. = + – = th
20 30 15 60

55
Part filled in 55 × 3 min. = th
60

55 1 58
Part filled in 165 + 1 min. = + = th
60 20 60

58 1 58 + 2 60
Part filled in 166 + 1 min. = + = = = full
60 30 60 60
It takes 167 minutes to fill the cistern.

Q : A bath can be filled by the cold water pipe in 10 minutes and by the hot water pipe in 15
minutes. A person leaves the bathroom after turning on both. He returns just when the bath
should have been full. Finding however, the waste pipe has been open, he closes it. In 4 mins.
more, the bath is full. In what time will the waste water pipe empty it?
ab 150
A : Time taken by the two pipes to fill it = = = 6 minutes.
a+b 25
 For six minutes, the three taps worked, and for the last 4 minutes, the first two taps worked
and the bath was full. If x is the time taken to empty the bath, then,
6 6 6 4 4 10 6
+ – + + = 1  =  x = 9 mins.
10 15 x 10 15 15 x

196
Chapter QA
WORK, PIPES AND CISTERN 3.8

Q : 4 pipes can fill a reservoir in 15, 20, 30 and 60 hrs. respectively. The first was opened at 6

THEORY
a.m., second at 7 a.m., third at 8 a.m. and fourth at 9 a.m. When will the reservoir be full?
A : Let the time be t hours after 6 a.m.

PRACTICE EXERCISE CLASS EXERCISE


t (t – 1) (t – 2) (t – 3)
 + + + = 1
15 20 30 60

 4t + 3(t – 1) + 2(t – 2) + (t – 3) = 60

 t = 7 hrs.

 It is filled at 1 p.m.

Q : A barrel contains 36 gallons of beer at 12 noon. One tap draws a pint in every 4 minutes
and another draws a quart every 6 minutes. How much beer will be left at 12 minutes past
8 p.m.? (4 quarts = 8 pints = 1 gallon)

A : Capacity of the barrel = 288 pints.

8 × 60 + 12
Beer removed by the first tap = = 123 pints
4
2
Beer removed by the second tap in 492 minutes = × 492 = 164 pints
6
Total beer removed = 287 pints
 Quantity of beer remaining = 1 pint.

197
CATapult
ARITHMETIC

Concept Builder 2

1. A cistern has a leak which would empty it in 10 hours. A tap is turned on, which admits
4 litres a minute into the cistern, and it is now emptied in 12 hours. How many litres
does the cistern hold?
2. A barrel contains 56 litres of kerosene. It has two taps. One tap draws 500 ml in every 6
minutes. After first 5 litres are drawn from barrel, the second tap starts. It draws 1 litre
in every 5 minutes. How many hours will be taken by both taps to empty the tank?
3. A man fills up a vessel with a pipe in 10 minutes. Incidentally, there is a leak in the vessel
which would empty the vessel in 20 minutes. After how many minutes can the vessel be
filled up, if the pipe and the leak function simultaneously?
4. To fill a cistern, pipes A, B and C takes 2 hrs, 3 hrs and 6 hrs respectively. The time in
hours that the three pipes together will take to fill the cistern is:
5. Two pipes A and B can fill a tank in 6 hrs and 9 hrs respectively. While a third pipe ‘C’
empties the full tank in 3 hours. If all the three pipes operate simultaneously, in how
much time will the tank be emptied?
6. Two pipes A and B can fill a tank in 3 hours and 6 hours respectively. If both the pipes
are opened simultaneously, how much time will be taken to fill the tank?
7. A tap can fill a tank in 45 minutes and anothers tap can empty it in 3 hours. If both taps
are opened at 11.30 am. Then the tank will be filled by:
8. Two pipes A and B can fill a cistern 15 min and 20 min. respectively. Both pipes are
opened the cistern will be filled in just 12 min, if the pipe B is turned off after:
9. A water tank is three-fifth full. Pipe A can fill a tank in 8 minutes and pipe B can empty
it in 5 minutes. If both the pipes are open, how long will it take to empty/fill the tank
completely?

Answer Key
9. 8 mins to empty.
8. 4 min. 7. 12.30 p.m 6. 2 hours. 5. 18 hrs.
4. 1 hr 3. 20 minutes. 2. 4 hours 1. 14400 litres.

198
Chapter QA
WORK, PIPES AND CISTERN 3.8

CLASS EXERCISE

Teaser

A sage stays atop a mountain in the Himalayas. Every morning he starts at 5:00 and reaches
the foothills at 9:00. He travels non-uniformly, walking leisurely at times and at times breaking
into a trot. He also takes rest at some points, but never digresses from his path. In the evening
he starts at 5:00 to climb up in the same manner on the exact same path and reaches the top
exactly at 9:00.

On any given day, will the sage be at any point on his route at the same time in the morning
and evening?
1] Certainly 2] Certainly Not 3] Cannot Say

199
CATapult
ARITHMETIC

A piece of work

1. 5 men complete a piece of work in 25 days. In how much time will 15 men complete the
work?

2. If a man takes 15 days to complete a piece of work, what part of the work will 2 men
complete in 5 days?

3. Some workers complete a piece of work in 60 days. If 8 more workers are employed, they
take 10 days less. What is the original number of workers?

4. Twelve men complete a piece of work in 8 days. Three days after they start, another 3 men
join them. How many days later will they complete the work?

5. Three men can complete a piece of work in 10, 12 and 15 days respectively. They worked
on it together and received a total amount of Rs.4500 for the job. How should they distribute
the amount amongst themselves?

More work

6. Calvin’s father asks Calvin to clear the snow from the sidewalk. Calvin says, “Dad it’ll take
me 8 hours to complete the work! Why don’t you ask Hobbes instead? He is twice as fast;
he’ll finish it in just 4 hours.” “I have a better idea” says Calvin’s father, “Both of you work
at it together.”

a) How long will it take Calvin and Hobbes to complete the work together?
b) Calvin and Hobbes start on the job together at 9 am but after 2 hours Calvin gets bored
and leaves. At what time will Hobbes complete the work?

7. Miss Wormwood asks Calvin to complete an 8-page exercise set in Math. Susie blurts out,
“M’am, shall I do that too? I can complete it in just 2 hours!” Miss Wormwood replies, “Well,
if you help him out, the two of you will finish it in just 1hr 40 min.” Calvin thinks to himself,
“If I take Hobbes’ help, I will finish the task in 4 hours”.

a) If the three of them work at it together, how long will it take them to finish the task?
b) Calvin glances at the exercise and realises he has no clue how to solve it. So he dumps
the task completely on Hobbes. How long will Hobbes take to finish it alone?

8. Calvin and Hobbes together take 20 minutes to build a snowman. Hobbes and Susie together
take 30 minutes for the same while Calvin and Susie together take 40 minutes.

a) How long will Susie take to build a snowman alone?


b) If Calvin and Hobbes together make 24 snowmen, how many snowmen will Susie alone
make in the same time?

200
Chapter QA
WORK, PIPES AND CISTERN 3.8

Work on pipes

THEORY
For questions 9 and 10: Refer to the following data and answer the questions. Do not carry
additional data provided in a question to the next questions.

PRACTICE EXERCISE CLASS EXERCISE


A 500-litre water storage tank is situated at the top of a building at a height of 300 feet. The
tank, which is completely full, has two outlet pipes and one inlet pipe. Veeru climbs up to the
tank and opens an outlet which can empty a full tank in 10 minutes. At the same time Jay opens
an inlet pipe which can fill an empty tank in 12 minutes.

9. In how much time will the tank be empty?

10. When the tank becomes half empty, Veeru opens the other outlet pipe which can empty a
full tank in 15 minutes. In how much time (from the start) will the tank be empty?

11. There are 3 pipes P, Q and R, which could fill a certain container in 38, 44 and 28 minutes
respectively. The container is initially empty. Pipe P is opened at 9:41 a.m. 8 minutes later,
pipe Q is opened. How many minutes after this should pipe R be opened so that the con-
tainer will get filled at exactly 10 a.m.?

12. A cistern has 3 pipes P, Q and R. P and Q can fill it in 6 and 5 hours respectively and R
can empty it in 2 hours. If these pipes are opened in order (i.e. P, Q and R) at 6 a.m., 7
a.m. and 8 a.m. respectively, then when will the cistern be empty?

Still more work

13. 32 workers working 10 hours a day can complete one-third of the work in 8 days. After the
1st day, 32 additional workers were employed and all of them (i.e. the original set of workers
plus the additional workers employeed) worked 9 hours per day. In how many days will the
remaining work be completed?

14. * 20 workers were employed to do a task in 40 days. But some of the workers had to leave
after 16 days. The remaining workers completed the work in 46 days from the day the work
was started initially. How many workers left after 16 days?

15. A man, a woman and a boy can complete a piece of work in 20, 30 and 60 days respectively.
How many boys should join 2 men and 8 women to complete the work in 2 days?

16. 8 women or 6 boys do the work of 4 men. 15 boys and 5 men are employed together to
complete a job in 12 days. How long will 3 men, 4 women and 6 boys take to complete the
job together?

17. * In 10 days, 6 horses and 2 mules complete a task which takes 4 horses and 4 mules 12
days to complete. What is the ratio of rate of work of a horse to that of a mule?

201
CATapult
ARITHMETIC

Challengers

1. Ten men started on a job. After some days four men left. So, instead of 40 days the job
took 50 days in all. After how many days did the men leave?

2. Two taps fill a tank in 6 hours and 8 hours respectively. If they are opened turn by turn,
each for an hour starting with the 1st tap, then after how many hours will the tank be full?

3. A king is planning to build a tomb that would take ten masons ten days. Unfortunately, on
the first day only 1 mason is available. On the second day two more join. This continues,
with 2 more masons joining in the work every day till the work is complete. How many days
will it take to build the tomb?

4. There is an empty tank to be filled with water using the three pipes ‘x’, ‘y’ & ‘z’. Pipe ‘x’ &
‘y’ alone can fill the tank completely in 30 hours and 45 hours respectively. If the pipes ‘x’
& ‘y’ are opened in alternate hours along with ‘z’ (i.e. in the first hour pipes ‘x’ & ‘z’ fill the
tank, in the second hour pipes ‘y’ & ‘z’ fill the tank & so on), then the tank is 70% filled in
15 hours. If the supply of through pipe ‘z’ is 75% of the original supply when it is opened
along with pipe ‘x’ or pipe ‘y’, then what is the time taken by pipe ‘z’ alone to fill the tank
completely?
1) 45 hours 2) 40.5 hours 3) 56.25 hours 4) 50 hours

5. 11 men and 8 women can complete a piece of work in 5 days, whereas 11 boys and 3 women
can complete the same work in 6 days. In how many days will 1 man, working along-with
1 woman and 1 boy complete the work?
1) 11 days 2) 30 days 3) 26 days 4) 21 days

202
Chapter QA
WORK, PIPES AND CISTERN 3.8

PRACTICE EXERCISE-1

PRACTICE EXERCISE CLASS EXERCISE THEORY


DIRECTIONS for questions 1 to 8: Choose the correct alternative.

1. A is thrice as good a workman as B and so takes 60 days less than B for doing a job. Find
the time in which they can do it together.

1 1 3
1) 20 days 2) 21 days 3) 22 days 4) 23 days
2 2 4

2. Two pipes can fill a cistern in 10 and 12 hours respectively. A third pipe can empty the
cistern in 15 hours. If all of the three pipes are opened simultaneously how much time will
they take to completely fill the cistern?

4 3 4 7
1) 8 hours 2) 4 hours 3) 7 hours 4) 4 hours
7 7 8 8

3. 3 pipes can fill a reservoir in 10, 15 and 20 hours respectively. If the three taps are opened
one after another in the given order, with a certain fixed time gap between them, the reservoir
fills in 5 hours. Find the time gap.

1 1 3
1) hr 2) hr 3) hr 4) 1 hr
2 4 4

4. A alone completes some work in 18 days and B alone can do it in 16 days. They work to-
gether for 5 days, then B quits and rejoins after 3 days, when A quits the work completely.
Find the number of days required to finish the job.
1) 9 days 2) 10 days 3) 13 days 4) None of these

5. A can paint a wall in 11 days. How long would it take (approximately) for B and C to paint
the same wall together if it is known that to paint a smaller wall, individually, A took 7 days,
B took 10 days and C took 12 days?
1) 9 days 2) 15 days
3) 60 days 4) Cannot be determined

6. 8 men and 3 women finished a piece of work in 8 days. 2 men and 12 women can also
finish the work in 8 days. How many days will 6 men and 6 women take to finish the same
work?
1) 10 days 2) 8 days 3) 6 days 4) None of these

203
CATapult
ARITHMETIC

7. There are 16 workers in a coal mine factory. All of them start working for a project. After
1 1
working for rd of the estimated time, they realise that only th of the work is done. If
3 4
2
they finish the entire work in rd of the original time, how many extra workers they have
3
employed?
1) 40 workers 2) 20 workers 3) 32 workers 4) 24 workers

8. An amount of Rs.148.39 is to be distributed as wages amongst a man, a woman and a boy.


The man worked for 26 days, the woman for 22 days and the boy for 21 days. If in a fixed
time 5 times the work done by the boy is equivalent to 3 times the work done by the woman
and 7 times the work done by the woman is equivalent to 5 times the work done by the
man, how much would the man earn?
1) Rs.76.076 2) Rs.81.33 3) Rs.73.99 4) Rs.57.42

DIRECTIONS for questions 9 and 10: Refer to the data below and answer the questions that
follow.

Sita can complete a project in 4 days and if the same project is done by Gita she needs 5 days.
Their sister Rita can do a project in 4 days while for the same project Gita requires 3 days. All
of them working together can finish a particular project together in 25 days.

9. How many days are required by Rita to complete the project alone?
1) 200 days 2) 145 days 3) 150 days 4) 100 days

10. How many days are required by Gita if she works alone to complete the project?
1) 175 days 2) 75 days 3) 100 days 4) 150 days

DIRECTIONS for questions 11 to 18: Choose the correct alternative.

11. Three pipes can fill a pond in 4, 6 and 8 hours respectively. In addition to these three pipes,
a fourth pipe was used and all these 4 pipes used together filled the pond in 2 hours. It is
also known that the first two pipes were opened one hour prior to opening the other two
pipes. Find the number of hours required for the fourth pipe to fill the pond individually.
1) 24 2) 8 3) 4 4) None of these

12. Two pipes A and B can separately fill a cistern in 15 and 20 mins respectively and the waste
pipe C can carry off 10 litres per minute. If all the pipes are opened when the cistern is full,
it is emptied in 2 hours. How many litres does the cistern hold?
1) 80 litres 2) 90 litres 3) 56 litres 4) 40 litres

204
Chapter QA
WORK, PIPES AND CISTERN 3.8

13. Mr.X has to build a wall 1000 metres long in 50 days. He employs 56 men but at the end

PRACTICE EXERCISE CLASS EXERCISE THEORY


of 27 days finds that only 448 metres are built. How many more men must be employed so
that the work may be finished in time?
1) 81 2) 25 3) 38 4) 58

14. A supply of water lasts for 150 days, if 7.5 gallons leak out every day, but only for 100 days
if 15 gallons leak out daily. What is the total quantity of water in the supply?
1) 2250 gallons 2) 1125 gallons 3) 3350 gallons 4) 1250 gallons
(Past CAT question)

15. If 200 soldiers eat 10 tonnes of food in 200 days, how much will 20 soldiers eat in 20 days?
1) 1 ton 2) 10 kg 3) 100 kg 4) 50 kg
(Past CAT question)

16. A group of workers was put on a job. From the second day onwards, one worker was with-
drawn each day. The job was finished when the last worker was withdrawn. Had no worker
been withdrawn at any stage, the group would have finished the job in two thirds the time.
How many workers were there in the group?
1) 2 2) 3 3) 5 4) 10
(Past CAT question)

17. P, Q and R are the three workers assigned to a particular work. They can finish the work
individually in 8, 12 and 16 days respectively. On any given day, all the three do not work
together. Also no worker can work on more than two consecutive days. What is the minimum
number of days in which the work will be completed?
4 3 5 3
1) 4 2) 5 3) 5 4) 5
7 5 9 7

18. Three groups—A, B and C—are required to complete the same job individually. Group A con-
sists of 1 man and 3 women, group B consists of 2 men and 3 women and group C consists
of both men and women. The number of days required by groups A, B and C to individually
complete the job are in the ratio 6 : 4 : 3. If, instead of three groups, all the men work
together and all the women work together to do the same job, which of the following cannot
be the ratio of the number of days taken by the group of men and women?
1) 1 : 2 2) 4 : 5 3) 2 : 1 4) 5 : 4

205
CATapult
ARITHMETIC

DIRECTIONS for questions 19 to 21: Solve as directed.

19. Hitler and Mussolini are working separately on two assignments that are identical in all
respects. Hitler takes 8 days to complete the assignment while Mussolini takes 12 days to
complete the assignment while working alone. If the both of them start working on their
assignments at the same time, after how many days will the remaining work to be done by
Mussolini be exactly double the remaining work to be done by Hitler?

20. Person A was given a certain job to be completed in 20 days. But after working for 5 days,
he was asked to finish the job in total 15 days. For this, he included person B to join him
from the 6th day onwards such that they would be able to finish the job exactly at the end
of 15 days. But after the 10th day, they were asked to finish the job in total 13 days. So, A
included another person C who would join them from the 11th day onwards such that the
work would be completed exactly at the end of 13 days. Had all three been working together
from the beginning, how many days would they have taken to finish the job?

21. Three friends - A, B and C - were assigned to do a certain job together. A alone can do it
in 15 days, B alone can do it 10 days and C alone can do it in 12 days. Initially, all three
started working together on the job. After some time, C left the job and A and B worked
together to finish the job. They received a total of Rs.12000 for finishing the job and A got
Rs. 1300 more than C. How much did B earn (in Rs.)?

206
Chapter QA
WORK, PIPES AND CISTERN 3.8

PRACTICE EXERCISE-2
DIRECTIONS for questions 1 to 12: Choose the correct alternative.

1
1. 3 women and 4 boys together can finish a piece of work in 2 days. 4 men and 2 boys
2
together can finish the same work in 2 days. Also, 2 men and 3 women together can finish it
1
in 2 days. How long would it take 1 man, 1 woman and 1 boy working together at double
2
their efficiencies to complete the work?
4 4 8 8
1) 1 days 2) 2 days 3) 3 days 4) 4 days
13 13 13 13

2. An amount of Rs.121.55 is to be distributed as wages amongst a man, a woman and a boy.


If the man worked for 16 days, the woman for 14 days and the boy for 10 days with the
1 1 1
ratio of the work done by them per day being : : , how much would the man earn?
3 4 5

1) Rs.59.74 2) Rs.48.82 3) Rs.59.84 4) Rs.59.27

3. A, B and C can finish a project in 9, 12 and 15 days respectively. They decide to take turns
and complete the work with A alone working on Monday, B alone on Tuesday, followed by
C alone on Wednesday and so on. What fraction of the work is done by C?
2 4 1 16 4
1) 2) 3) 4) 5)
9 15 3 45 45

4. In a shoe manufacturing factory, 18 men manufacture 20 shoes in 10 days working for 7.5
hours a day. How long will it take for 36 men to manufacture 40 shoes working 6 hours a
day? It is also known that 4 men in the latter case do as much work as 6 men in the former.

20 25 30
1) days 2) days 3) days
3 3 3
35
4) days 5) Cannot be determined
3

5. Taps A and C can fill a tank in 5 and 7 hrs respectively. Taps B and D can drain a full tank in
6 and 8 hrs. respectively. Taps A and B are opened at 6 a.m. and 6.30 a.m. respectively, till
65% of the tank is filled. Then, C and D are also opened. At what time will the tank get filled?

1025 1134 1249 1312


1) hrs 2) hrs 3) hrs 4) hrs
43 43 43 43

207
CATapult
ARITHMETIC

6. A tank has four taps A, B, C, D. A and C are inlets and can fill the tank in 4 and 6 hours
respectively. B and D are outlets and can drain the completely filled tank in 5 and 7 hours
respectively. A is opened at 12:00 noon and B, C, D are opened after a gap of one hour
each, in that order. In how many hours, will the tank be full?

269 296 267 324


1) 2) 3) 4)
31 31 31 31

7. There are ten water tanks T1, T2,..., T10 and another tank U each of capacity 6000 litres. A
common inlet pipe giving 100 litres/min fills the tanks according to the following scheme.
While T1 gets filled, half the inlet water goes into it, half of the remaining inlet water goes
into T2, half of the remaining goes into T3 and so on till T10. Whatever is left out of the inlet
water goes into U. After T1 gets filled, no more water goes into it and half the inlet water
goes into T2, half of the remaining goes into T3, and so on till T10 and then all the remaining
inlet water goes into U. This procedure goes on as each tank gets filled. How long does it
take to fill U?

1) 5 hours 2) 10 hours 3) 11 hours 4) None of these

8. There are three pipes in a tank which can work as both inlet and outlet pipes. Their rate of
filling or emptying a tank are in the ratio 2 : 3 : 6. The largest pipe can fill the tank alone
in 3 hrs. Initially the tank is empty. The first pipe is used as an outlet and second and third
pipe are used as inlets for the first hour. In the second hour, the second pipe works as an
outlet pipe and the first and the third are used as inlet pipes. In the third hour the third
pipe is used as an outlet and the first two as inlet and so on. The process is continued till
the tank is full. The total time required to fill the tank is:
1) 5 hrs 2) 4 hrs 3) 3.5 hrs 4) None of these

9. The work done by a man, a woman and a child is in the ratio of 3 : 2 : 1. There are 20
men, 30 women and 36 children in a factory. Their weekly wages amount to Rs.780, which is
divided in the ratio of work done by the men, women and children. What will be the wages
of 15 men, 21 women and 30 children for 2 weeks?

1) Rs.585 2) Rs.292.5 3) Rs.1170 4) Rs.900

10. A can do a piece of work in 8 days which B and C can do in 12 and 16 days respectively.
If they work on alternate days, then which of the following is true?
I. If A started the work on the first day followed by B and C, then the work will be
finished on the 10th day.
II. If B started the work on the first day followed by C and A, then the work will be
finished on the 11th day.
III. If C started the work on the first day followed by A and B, then the work will be
finished on the 11th day.
1) I 2) I & II 3) II 4) III

208
Chapter QA
WORK, PIPES AND CISTERN 3.8

11. Pipes P, Q and R are attached to a tank and each can act as either an inlet or outlet pipe.

PRACTICE EXERCISE CLASS EXERCISE THEORY


Pipes P, Q and R respectively take 8, 10 and 12 hours to fill the empty tank or empty the full
tank. In the first hour, pipes P and R work as inlet and Q work as outlet. In the second hour,
pipes P and Q work as inlet and pipe R as outlet. In the third hour pipes Q and R work as
inlet and pipe P as outlet and the process goes on like this. When will the cistern be filled?
1) In the 8th hour. 2) In the 9th hour.
3) In the 10th hour. 4) In the 11th hour.

DIRECTIONS for questions 12 and 13: Refer to the data below and answer the questions that
follow.

The boiler tank in a chemical factory holds 105 litres. 5 tanks working together each having one-fifth
the capacity of the boiler tank can fill in ‘hard water’ at same rates in the boiler tank in 2 hours.
Let us suppose the outlet of the two of the small tanks work as inlet pipes, two other small tanks
work as outlet pipes and the fifth small tank fills in the main ‘boiler’ at half its efficiency.

12. What is the number of hours taken to fill in the main boiler when all the pipes are opened
at once?
1) 1 hour 2) 5 hours 3) 20 hours 4) 25 hours

13. Only three pipes are working, two at their full efficiency and the third one at half its efficiency
and all three are acting as inlet pipes. If all the three pipes are opened alternately (with only
one pipe operating at a time) to fill the boiler such that each small tank fills ‘hard water’ in
the main boiler for equal time, then the number of hours taken to fill the boiler to half its
capacity is:
1) 2 hours 2) 4 hours 3) 6 hours 4) 8 hours

DIRECTIONS for questions 14 to 19: Choose the correct alternative.

14. A water tank has three taps A, B and C. A fills 4 buckets in 24 minutes, B fills 8 buckets in
1 hour and C fills 2 buckets in 20 minutes. If all the taps are opened together, a full tank is
emptied in 2 hours. If a bucket can hold 5 litres of water, what is the capacity of the tank?
1) 120 litres 2) 240 litres 3) 180 litres 4) 60 litres
(Past CAT question)

15. There is a leak in the bottom of the tank. This leak can empty a full tank in 8 hours. When
the tank is full, a tap is opened into the tank which admits 6 litres per hour and the tank
is now emptied in 12 hours. What is the capacity of the tank?
1) 28.8 litres 2) 36 litres 3) 144 litres 4) Indeterminate
(Past CAT question)

209
CATapult
ARITHMETIC

16. One man can do as much work in one day as a woman can do in 2 days. A child does one-
third the work done by a woman in a day. If an estate-owner hires 39 pairs of hands - men,
women and children in the ratio 6 : 5 : 2 and pays them a total of Rs. 1,113 at the end of
the day’s work. What must the daily wages of a child be, if the wages are proportional to
the amount of work done?
1) Rs. 14 2) Rs. 5 3) Rs. 20 4) Rs. 7
(Past CAT question)

17. 3 men and 4 women together finish a certain job in 11 days. However, all of them did not
work for all the 11 days. The ratio of the number of days that the 3 men and 4 women did
not work is same as the ratio of the efficiency of 1 man and 1 woman. What is the ratio of
the number of days that the 3 men and 4 women worked?
Note: Either all three men worked together or didn’t work together. Similarly, either all four
women worked together or didn’t work together. It is also known that 5 men working to-
gether can complete the entire job in 3 days and 2 women working together can complete
the entire job in 12.5 days.
1) 5 : 3 2) 1 : 5 3) 3 : 5 4) 5 : 1

18. B is 20% less efficient than C and A is 20% more efficient than C. The number of days tak-
en by A and B together to complete a particular work is one less than the number of days
taken by B and C together to complete the same work. If C works alone for as many days
as the number of days required for A and B to complete the work together, what percentage
of work will C complete in those many days?
1) 40% 2) 50% 3) 60% 4) 75%

DIRECTIONS for questions 19 and 20: Solve as directed.

19. 5 men can mow 500 m2 of a lawn in 10 days whereas 15 women can mow 1500 m2 of the
lawn in 20 days. In how many days can 6 men and 2 women mow 2100 m2 of the lawn?

20. Jericho started working at a certain rate and finished half of the work in 16 days. Then he
increased his efficiency by 100% each day as compared to the previous day. On which day
will he complete the work, counting from the day he began?

210
CATapult
ARITHMETIC

QA-3.9 TSD APPLICATIONS - I THEORY

Races
Basic terminology in races
Races is an application of Time, Speed and Distance.
A contest of speed between participants is called a race.
The point from where a race begins is called the starting point and the point where the race
finishes is called the winning post or finishing point or a goal.
If all the persons contesting a race reach the winning post at exactly the same time, then the
race is said to be a dead-heat race.
Important Concepts and Formulae related to Races
1. Start distance: ‘A gives B a start of x metres’, implies that, if the distance between the
starting point and finishing point is L metres, A covers L metres while B covers L – x metres.
e.g., In a 100 metre race, A gives B a start of 10 metres means, while A runs 100 metres,
B runs 100 – 10 = 90 metres.
2. Beat distance: ‘A beats B by x metres’, implies that, if the distance between the starting
point and finishing point is L metres, A wins the race by covering L metres, while B covers
L – x metres only.
3. Start time: ‘A gives B a start of t seconds’, implies that, A starts the race t seconds after B
starts from the starting point.
4. Beat time: ‘A beats B by t seconds’, implies that, A and B start together from the starting
point, but A reaches the finishing point t seconds before B finishes.
Note: (3) & (4) both imply that B takes t seconds more than A to finish the distance.
5 A beats B by ‘x’ metres or ‘t’ seconds means, B runs ‘x’ metres in ‘t’ seconds.
6. Winner’s distance = Length of the race.
7. Distance covered by loser = Winner’s distance – (Beat distance + Start distance)
8. Time taken by winner = Time taken by loser – (Beat time + Start time)
Winner's Time Loser's Time Beat Time + Start time
9. = =
Loser's Distance Winner's Distance Beat Distance + Start distance
10. If a race ends in a dead heat, beat time = 0 and beat distance = 0.
11. Two persons starting at the same time and from the same point along a circular path will be
together again for the first time, when the faster gains one complete round over the other.
Length of race course
Time taken by faster person to complete one round over the other =
Relative Speed
12. Two persons, starting at the same time from the same point along a circular path, will be
together again for the first time at the same starting point, at a time which is the LCM of
the time taken by each to complete a round.

211
CATapult
ARITHMETIC

13. Three persons, starting at the same time and from the same point along a circular path, will
be together for the first time after the start at a time which is equal to the LCM of the time
taken by the fastest to gain a complete round over each of the other two.

th of xth round means, when A has completed b x – l rounds, B has


1 1
14. A overtakes B
n n
completed :(x – 1) – D round.
1
n
A's Speed Rounds completed by A in a given time
Also, =
B's Speed Rounds completed by B in the same time
Example
A overtakes B in the middle of the 4th round implies, when A has completed 3½ rounds, B
1
A's Speed 32
has completed 2½ rounds. =
B's Speed 1
22

SOLVED EXAMPLES

Q : A can give B a 40 metres start and C a 70 metres start, in a one km race. How many metres
start can B give C in a 1 km race?
A : A runs 1000 m, while B runs (1000 – 40) or 960 m.
A runs 1000 m, while C runs (1000 – 70) or 930 m.
 B runs 960 m, while C runs 930 m.
1000 × 930 3
 B runs 1000 m, while C runs = 968 m.
960 4
 B can give C b 1000 – 968 l or 31
3 1
m. start.
4 4

Q : In a km race, A beats B by 40 metres or 7 seconds. Find A’s time over the course.
A : Here B runs 40 metres in 7 seconds.
 B runs 1000 m in 1000 × 7 = 175 seconds.
40
Hence, A’s time over the course = (175 – 7) = 168 seconds.
Alternatively,
Winner's Time Beat Time + Start time
By formula, =
Loser's Distance Beat Distance + Start distance
A's time 7+0 7
=  A’s Time = × 960 = 168 seconds.
1000 – 40 40 + 0 40

212
Chapter QA
TSD APPLICATIONS-I 3.9

Q : In a km race, if A gives B a 40 m start, A wins by 19 seconds, but if A gives B 30 seconds

THEORY
start, B wins by 40 m. Find the time that each takes to run a km?
A : Suppose A takes x seconds and B takes y seconds to run 1000 m.
960 960x
Then x + 19 = and + 30 = y

PRACTICE EXERCISE CLASS EXERCISE


1000 y 1000
Solving for x and y: x = 125 seconds and y = 150 seconds.
A takes 125 seconds and B takes 150 seconds.

Q : Two men, A and B, walk around a circle 1200 m in circumference. A walks at the rate of 150
m/min. and B at the rate of 80 m/min. If they both start at the same time from the same
point, and walk in the same direction,
(a) When will they first be together again at the starting point?
(b) When will they be together again?
1200
A : (a) A makes one complete round of the circle in = 8 minutes,
150
1200
B in = 15 minutes
80
That is, after every 8 minutes, A is at the starting point and after every 15 minutes B
is at the starting point.
Now, A and B will be together again at the starting point at the end of the time during
which each can make an exact number of rounds. Hence, the required time is the LCM
of 8 and 15 minutes, i.e., 120 minutes or 2 hours.
(b) A and B will be together again for the first time when A has gained one complete
round over B.
Now, A gains (150 – 80) or 70 metres on B in 1 minute.
1200 1
 A will gain 1200 metres in or 17 minutes
70 7
1
 A and B will be together in 17 minutes.
7

Q : Three men A, B and C walk around a circle, 1760 metres in circumference, at the rate of 160,
120 and 105 m/minute, respectively. If they all start together and walk in the same direction,
when will they first be together again?

A : A, the quickest man gains one complete round on C, the slowest man, in b l or 32
1760
160 – 105

min. A gains one complete round on B, the next slowest man, in b l or 44 mins.
1760
160 – 120
Thus, A and C are together after every 32 minutes and A and B are together after every 44
minutes. Hence, A, B and C will be together in the time which is the LCM of 32 and 44, i.e.,
352 minutes or 5 hours 52 minutes.

213
CATapult
ARITHMETIC

Concept Builder 1

1. In a 100 m race, A can give B a start of 10 m and C 19 m. In the same race, B can give
C a start of:
2. A and B start from the same point to run in opposite directions round a circular path 550
yards in length, A giving B a start of 100 yards. They pass each other when A has run 250
yards. Who will come first to the starting point and at what distance will they be apart?
3
3. A runs 1 times faster than B. A gives B a start of 120 metres. How far must the winning
8
post be so that it may be a dead heat?
4. How much time does a racer X take running on a circular track of 440 metres in anticlock-
wise direction to meet racer Y who runs at double the speed of X in the anticlockwise
direction. They start running in the same race from the diametrically opposite ends given
that the speed of X is 19.8 kmph?
5. Red and black ants are running on a rectangular frame of length 7 cm and breadth 3 cm.
Red ant is running with a speed of 60 cm/min and black ant is running with a speed of
40 cm/min. How much time (in seconds) will it take for both of them to meet at starting
point if both of them start running from the same point?
6. Two boys running in opposite directions meet each other after 10 minutes on travelling a
distance of 2.4 km. Their speeds are in the ratio 3 : 5. Find the time required for both of
them to meet each other on a circular track of 2.4 km, for the first time if both of them
start at the same time from the same point and move in the same direction.
7. The speeds of A and B are 100 m/s and 75 m/s, respectively. If A and B runs a race of
500 metres, then A beats B by what distance?

Answer Key

125 metres 7.
40 minutes. 6.
60 sec. 5.
40 sec. 4.
440 metres 3.
A, 10 yards 2.
10 m 1.

214
Chapter QA
TSD APPLICATIONS-I 3.9

Clocks

THEORY
This section introduces you to the application of the concepts of Time, Speed, Distance & races
to the problems on clocks.

PRACTICE EXERCISE CLASS EXERCISE


Basic Terminolgy
The face or dial of a clock is divided along the circumference into 60 equal spaces called minute
spaces. The minute hand moves around the whole circumference of the clock once in one hour.
The hour hand moves around the whole circumference of the clock once in 12 hours. Thus, the
minute hand is twelve times faster than the hour hand.
The minute hand passes over the 60 minute spaces, while the hour hand goes over the 5 minute
spaces.
55
That is, in 60 minutes, the minute hand gains 55 minutes over the hour hand or minute
60
spaces in 1 minute.

Important formulae for clock related problems

360°
1. 1 minute space = . (As 360° of the circle is divided into 60 minutes)
60
2. In one minute, the minute hand moves 6°.

360 360 1°
3. In one minute, the hour hand moves = =
12 × 60 720 2
(As there are 12 hours of 60 minutes each) Thus, in one minute the minute hand gains

5 over the hour hand.
2
4. In every hour,
(a) the hands coincide once (0° apart).
(b) the hands are twice at right angles (90° apart) and in these positions the hands are 15
minute spaces apart.
(c) the hands point in opposite directions (180° apart) once and in this position, the hands
are 30 minute spaces apart.

5. The hands are in the same straight line when they are coincident or opposite to each other.

6. The hands coincide 11 times in every 12 hours (between 11 and 1 o’clock there is a common
position at 12 o’clock). Hence, the hands coincide 22 times in a day.

7. If both the hands start moving together from the same position, both the hands will coincide
360 × 2 5
after = 65 minutes.
11 11
8. The hands of a clock are at right angles twice in every hour, but in 12 hours they are at
right angles 22 times since there are two common positions in every 12 hours.

9. Interchangeable positions of minute hand and hour hand occur when the original interval
60
between the two hands is minute spaces or a multiple of this.
13

215
CATapult
ARITHMETIC

Incorrect clocks
A clock which gains or loses time is called an incorrect clock. In incorrect clocks both hands
5
coincide at an interval which is not equal to 65 minutes.
11
In a slow clock, i.e., a clock that loses time:
5
Total time lost in T hours = (T × 60) f
x – 65 11 p
minutes,
x
where x is the time in which the hands of the incorrect clock coincide.
Also, for a fast clock, i.e., a clock that gains time:
5
Total time gained in T hours = (T × 60) f
65 11 – x p
minutes,
x
where x is the time in which the hands of the incorrect clock coincide.

SOLVED EXAMPLES

Q : At what time between 4 and 5 will the hands of a watch coincide?


A : At 4 o’clock, the hands are 20 minutes apart. Clearly, the minute hand must gain 20 minutes
before the hands can be coincident. But the minute hand gains 55 minutes in 60 minutes.
Hence, it will gain 20 minutes in:
20 × 60 240 9
= = 21 minutes.
55 11 11
9
 The hands will be coincident at 21 minutes past 4.
11

Q : At what time between 4 and 5 will the hands of a watch be at right angles?
A : At 4 o’clock the hands are 20 minutes apart. They will be at right angles when there is a space
of 15 minutes between them. This will happen twice (i) when the minute hand has gained (20
– 15) or 5 minutes; (ii) when the minute hand has gained (20 + 15) or 35 minutes.
5 × 60 5
The minute hand gains 5 minutes in = 5 minutes.
55 11
5
i.e., 5 minutes past 4.
11
and
35 × 60 2
The minute hand gains 35 minutes in = 38 minutes.
55 11
2
i.e., 38 minutes past 4.
11

216
Chapter QA
TSD APPLICATIONS-I 3.9

Q : At what time between 4 and 5 will the hands of a watch point in opposite directions?

THEORY
A : They will be opposite to each other when there is a space of 30 minutes between them.
This will happen when the minute hand gains (30 + 20) minutes = 50 minutes.

PRACTICE EXERCISE CLASS EXERCISE


50 × 60 6
The minute hand gains 50 minutes in = 54 minutes,
55 11
6
i.e., they will point in opposite directions at 54 minutes past 4.
11

Q : My watch, which gains uniformly, is 2 minutes slow at noon on Sunday, and is 4 minutes 48
seconds fast at 2 p.m. on the following Sunday. When was it correct?
A : From Sunday noon to the following Sunday 2 p.m. = 7 days 2 hours = 170 hours.
48 48 4
The watch gains 2 + 4 + = 6 + minutes or 6 minutes in 170 hours.
60 60 5
The watch will show the correct time when it has gained 2 minutes.
2
The watch gains 2 minutes in × 170 or 50 hours.
4
65
Now, 50 hours = 2 days and 2 hours. The watch will show the correct time after 2 days and
2 hours from Sunday noon, i.e., at 2 p.m. on Tuesday.

Concept Builder 2

1. From noon, by how many degrees has the minute hand moved to 2 : 40 p.m.?
2. In 36 hours, how many times will the hands of the clock coincide?
3. How many minutes does a watch gain per day, if its hands coincide after every 60 min-
utes?
4. How many degrees does an hour hand move in 20 minutes?
5. What is the angle between the minute hand and the hour hand of a clock when the time
is 7:30?

Answer Key
45° 5.
10° 4.

11
mins 130 3.
10

33 times 2.
960° 1.

217
CATapult
ARITHMETIC

Calendar
The Solar Year consists of 365 days, 5 hours, 48 minutes. In the calendar known as Julian
1
Calendar, arranged in 47 BC by Julius Caesar, the year was taken as being of 365 days and in
4
order to get rid of the odd quarter of a day, an extra or intercalary day was added once in every
fourth year and this was called Bissextile or Leap Year. But as the Solar Year is 11 minutes 12
seconds less than a quarter of a day, it followed in a course of years that the Julian Calendar
became inaccurate by several days and in 1582 AD, this difference amounted to 10 days. Pope
Gregory XIII determined to rectify this, and devised the calendar now known as the Gregorian
Calendar. He dropped or cancelled these 10 days – October 5th being called October 15th and
made centurial years leap years only once in 4 centuries – so that whilst 1700, 1800 and 1900
were to be ordinary years, 2000 was a leap year. This modification brought the Gregorian System
into such close exactitude with the Solar Year that there is only a difference of 26 seconds which
amounts to a day in 3323 years. This is the new style. It was ordered by an Act of Parliament
to be adopted in England in 1752, 170 years after its formation and is now used throughout the
civilized world.

The following simple facts should be remembered:

1. An ordinary year contains 365 days, i.e. 52 weeks and 1 odd day.

2. A leap year contains 366 days, i.e. 52 weeks and 2 odd days.

3. 100 years (a century) contain 76 ordinary years and 24 leap years


= (76 × 52) weeks + 76 odd days + (24 × 52) weeks + 48 odd days
= [(76 × 52) + (24 × 52)] weeks + 124 odd days
= [(76 × 52) + (24 × 52) + 17] weeks + 5 odd days,
i.e. 100 years contain 5 odd days.
200 years contain 10 and therefore 3 odd days. Similarly, 300 years contain 1 odd day, 400
years will have (20 + 1) odd days i.e. 0 odd days. Similarly, the years 800, 1200, 1600, 2000
each contain no odd days.

4. First January 1 AD was Monday. Therefore, we must count days from Sunday, i.e. Sunday for
0 odd days, Monday for 1 odd day, Tuesday for 2 odd days and so on.

5. February in an ordinary year gives no odd day, but in a leap year gives one odd day.

218
Chapter QA
TSD APPLICATIONS-I 3.9

SOLVED EXAMPLES

THEORY
Q : What day of the week was 20th June 1837 ?

PRACTICE EXERCISE CLASS EXERCISE


A : 20th June 1837 means 1836 complete years + first 5 months of the year 1837 + 20 days of
June.
1600 years give no odd days.
200 years give 3 odd days.
36 years give (36 + 9) or 3 odd days.
1836 years give 6 odd days.
From 1st January to 20th June there are 3 odd days.
Odd days :
January : 3
February : 0
March : 3
April : 2
May : 3
June : 6
17
Therefore, the total number of odd days = (6 + 3) or 2 odd days.
This means that the 20th of June fell on the 2nd day commencing from Monday. Therefore, the
required day was Tuesday.

Q : How many times does the 29th day of the month occur in 400 consecutive years?
A : In 400 consecutive years there are 97 leap years. Hence, in 400 consecutive years February
has the 29th day 97 times and the remaining eleven months have the 29th day 400 × 11 or
4400 times.
Therefore, the 29th day of the month occurs (4400 + 97) or 4497 times.

Q : Today is 3rd November. The day of the week is Monday. This is a leap year. What will be the
day of the week on this date after 3 years?
A : This is a leap year. So, none of the next 3 years will be leap years. Each year will give one
odd day so the day of the week will be 3 odd days beyond Monday i.e. it will be Thursday.

219
CATapult
ARITHMETIC

Q : What will be the day of the week on 19th April, 2020?


1] Sunday 2] Saturday 3] Friday 4] Monday
A : 2000 years has 0 odd day.
19 years = (4 leap years + 15 ordinary years) = [(4 × 2) + (15 × 1)] odd days = 23 odd days
 2 odd days
Days of months (Jan, Feb, Mar, Apr) 31 + 29 + 31 + 19 = 110 days = 15 weeks + 5 days =
5 odd days
Total number of odd days = (0 + 2 + 5) odd days = 7 odd days  0 odd days
 19th April 2020 is Sunday. Hence, [1].

220
Chapter QA
TSD APPLICATIONS-I 3.9

CLASS EXERCISE

Teaser

You have two wooden sticks each of which burns completely in an hour. You have a match box
with many matches to light the sticks. The sticks are of uneven density so they don't burn evenly
(for instance, in half an hour not necessarily half a stick is burnt).
How can you use these sticks to measure exactly 45 minutes?

221
CATapult
ARITHMETIC

Races

Data for questions 1 – 4:

In a 100 m race, Harry the hare beats Terry the tortoise by 20 m.

1. When Harry runs half a kilometre, how far will Terry have run, if both start together?

2. If both start together, how much distance will Harry have to run to gain a lead of 150 m?

3. If Terry starts 250 metres ahead, how much further will he reach before Harry catches him?

4. If Harry beats Terry by 10 seconds in a 200 m race, find their speeds.

5. A gives B a start of 100 m in a 600 m race. If the ratio of their speed is 5:4 who will win
and by how much?

6. A beats B by 20 m in a 100 m race. B beats C by 50 m in a 200 m race. C beats D by 150


m in a 500m race. By how much meters A beats D in a km race?

7. * In a race, if Shahid gives Saif a start of 100 metres, Shahid wins by 30 sec. If he gives
Saif a start of 150 metres, he still wins but by 15 sec. Find Saif’s speed (in m/min).

8. * In a 100m race, if Raj gives Rohit a start of 10 seconds, Raj wins by 5m. If he gives Rohit
a start of 10m, he wins by 5 seconds. What is Rohit’s speed?

Data for questions 9 – 15:

Every morning A, B and C go for a jog on a circular racing track 300 m long.

9. If A runs clockwise at 60 m/min and B runs clockwise at 100 m/min, starting from the same
point, when and where will they meet for the first time?

10. If A runs clockwise at 60 m/min and B runs clockwise at 100 m/min, starting from the same
point, when will they meet at the starting point for the first time?

11. If A runs clockwise at 60 m/min and C runs anti-clockwise at 40 m/min, starting from the
same point, when and where will they meet for the first time?

12. If A runs clockwise at 60 m/min and C runs anti-clockwise at 40 m/min, starting from the
same point, where will A be when they meet for the first time?

13. If A runs clockwise at 60 m/min and C runs anticlockwise at 40 m/min, starting from dia-
metrically opposite points, when and where will they meet for the second time?

14. If A runs clockwise at 60 m/min and B runs clockwise at 100 m/min, starting from the same
point and if they keep running infinitely, at how many points will they keep meeting?

222
Chapter QA
TSD APPLICATIONS-I 3.9

15. If A runs clockwise at 60 m/min and C runs anti-clockwise at 40 m/min, starting from the

THEORY
same point and if they keep running infinitely, at how many points will they keep meeting?

16. A, B and C start from the same point at the same time in the same direction and move
along a circular track of length 120 m at speeds of 5 m/s, 3 m/s and 2 m/s respectively.

PRACTICE EXERCISE CLASS EXERCISE


a. After how much time will they meet for the first time?
b. After how much time will they meet for the first time at the starting point?
c. If they keep running infinitely, at how many points will they keep meeting.

17. A, B and C start from the same point at the same time and move along a circular track of
length 112 m at speeds of 5 m/s, 3 m/s and 2 m/s respectively. B and C move in the same
direction and A moves in the opposite direction.
a. After how much time will they meet for the first time?
b. After how much time will they meet for the first time at the starting point?
c. If they keep running infinitely, at how many points will they keep meeting.

18. * A, B and C start driving at speeds of 15, 24 and 42 kmph, from the same point and in
the same direction, around a circular track of length 360 km. When and where will all three
meet again?

Clocks

Answer questions 19 – 27 assuming a standard clock with 12 hours marked on its face:

19. What are the speeds (in degrees/minute) of the hour and the minute hand of a clock?

20. When will the hour and the minute hand of a clock first be together after 4 o’clock?

21. When will the hour and the minute hand of a clock first be at right angles after 8 o’clock?

22. When will the hour and the minute hand of a clock first point in opposite directions after
noon?

23. When will the hour and the minute hand of a clock first be together after 11 o’clock?

24. What is the smaller angle between the hour and the minute hand at 9.20?

25. *What is the smaller angle between the hour and the minute hand at 3.40?

26. *A watch was running 7 minutes behind time on Monday at noon. It was running 10 minutes
ahead of time next Monday at 2.00 pm. If it showed the correct time exactly once during
the week, on which day and at what time did that happen?

27. * When will the hour and minute hands of a clock be at right angles for the second time
after 2 pm?

223
Calendar

28. In 2010, Valentine’s Day was a Sunday.


a) In which two years after 2010 will Valentine’s Day (14th Feb) again be a Sunday?
b) Independence Day (15th Aug) would have fallen on which day in the same year (2010)?
c) What was the day on Bal Din (14th Nov) in the year 2000?

29. * In a certain year, January had exactly 4 Mondays and 4 Fridays. What was the day on
Gandhi Jayanti (2nd Oct) the previous year?

Challengers

1. In a 2 km cross-country race, if Rachit gives Suraj a start of 120 m, Rachit wins by 70


seconds. But if Rachit gives Suraj a start of 120 seconds in the same race, Suraj wins by
100 m. What is the time taken by Rachit to complete the race?
1) 4 min 10 s 2) 5 min 3) 6 min 40 s 4) 7 min 30 s

2. Hritik, Abhishek and John run a race starting from the same point. They run at the speeds
of 200, 300 and 400 m/min respectively. Abhishek being faster than Hritik, starts 10 minutes
after Hritik. John being the fastest starts even later. Abhishek and John overtake Hritik at
the same time, how many minutes after Abhishek does John start?

3. The rim of a clock is divided into 60 equal divisions. At a certain point in time, the second
hand is exactly 1 division ahead of the minute hand. After how long will they next meet?
1) 58 sec 2) 59 sec 3) 60 sec 4) 61 sec

4. A and B start jogging at the same moment, from the same point (but in opposite directions)
along a circular track of length 500 metres. Initially, A jogs at 90 m/min while B jogs at 135
m/min. Every time they meet each other, they exchange speeds and reverse directions. When
they meet at the starting point for the first time, how much distance will A have jogged in
total?
1) 1200 m 2) 1500 m 3) 1 km 4) Cannot be determined

224
Chapter QA
TSD APPLICATIONS-I 3.9

PRACTICE EXERCISE
DIRECTIONS for questions 1 to 14: Choose the correct alternative.

1. Ram and Shyam can run at the speeds of 45 m/min and 54 m/min respectively. They start
from a point on a circular track of circumference 1980 m in the opposite direction. On meet-
ing, they turn around and run in opposite directions. Which of the following is a possible
distance on the track between their two meeting points?
1) 1080 m 2) 980 m 3) 1100 m 4) 1030 m

2. A and B run a km race and A wins by 60 seconds. A and C run a km race and A wins by
375 metres. B and C run a km race and B wins by 30 seconds. Find the time each takes to
run a km.
1 1 1
1) 2 min., 3 min., 4 min. 2) 3 min., 4 min., 4 min.
2 2 2
1 1 1 1
3) 3 min., 4 min., 5 min. 4) 4 min., 5 min., 6 min.
2 2 2 2

1
3. Two men, A and B, run a 4 km race on a circular course of km. If their speeds are as
4
5:4, how often does the winner pass the other?
1) Once 2) Twice 3) Thrice 4) Four times

4. Two cyclists start a race at 3 p.m. around a circular track. The first goes round once in 3
minutes 12 seconds and the second in 3 minutes 30 seconds. Find at what time they will
again meet at the starting point.
1) 4:52 p.m. 2) 3:06 p.m. 3) 9:12 p.m. 4) 3:50 p.m.

5. I want my watch which gains 1 minute every 15 hours to show the correct time after exactly 24
hours. How many minutes slow should I set my watch at the beginning of the 24 hours?
1) 1.6 2) 2 3) 1.06 4) 1.55

6. A man, who went out between five and six o’clock and returned between six and seven
o’clock, found that the hands of the watch had exchanged places. When did he go out?
4 2
1) 32 minutes past 5 o’clock. 2) 27 minutes past 5 o’clock.
13 13
3 5
3) 37 minutes past 5 o’clock. 4) 31 minutes past 5 o’clock.
13 13

7. Determine the time between 7 a.m. and 8 a.m. when the hands of a clock will be in the
same straight line but not coincide.
5 5
1) 7 hours 5 min 2) 7 hours 3 min
11 11
3 7
3) 7 hours 7 min 4) 7 hours 7 min
11 11

225
CATapult
ARITHMETIC

8. Find the time between 5 and 6 o’clock when the hands of a clock are 3 minutes apart.
6
1) 24 minutes past 5 2) 30 minutes past 5
11
6
3) Both (1) and (2) 4) 2 minutes past 5
11

9. A clock loses 5 seconds an hour and is set right on Sunday at noon. What time will it indicate
on the following Monday at noon?
1) 11.56 a.m. 2) 11.58 a.m. 3) 12.02 p.m. 4) 12.04 p.m.

10. Ram was born on the first Monday of March 1952. Then on which date was he born?
1) 1st 2) 3rd 3) 5th 4) 2nd

11. Which of the following is true?


1) 21st February 2001 is a Saturday. 2) 27th February 1999 is a Saturday.
3) 23rd February 2002 is a Sunday. 4) 22nd February 1998 is a Saturday.

12. A race is designed such that the person who completes twenty rounds of the circular field track of
50 meter radius first wins the race. Irrespective of the time taken everybody has completed the race.
A, B and C were among the participants. A has overtaken B (for the first time) after A completed
4
th of his 10th round and B has overtaken C (for the first time) in exactly the middle of
5
B's 12th round. If C has taken 30 minutes to complete the race then what was the speed of
A? It is assumed that the running speed of participants remains constant throughout the race.
[m/m = meter/minute]
1) 236 m/m 2) 246 m/m 3) 256 m/m 4) 266 m/m

9
13. If the hands of the clock coincide after every 66 minutes, then how many minutes do
11
the clock gain or lose in an entire day?
1) 30 minutes, gains 2) 30 minutes, loses
8 8
3) 32 minutes, gains 4) 32 minutes, loses
11 11

1
14. In a 2 km race on a circular course of of a km, A overtakes B in the middle of his 6th
4
round. By what distance will A win at the same rate of running?
2 18 4 11
1) km 2) km 3) km 4) km
9 11 11 9

226
Chapter QA
TSD APPLICATIONS-I 3.9

Directions for questions 15 to 18: Answer the following questions based on the information given.

PRACTICE EXERCISE CLASS EXERCISE THEORY


A and B are running along a circular course of radius 7 km in opposite directions such that when
they meet, they reverse their directions and when they meet, A will run at the speed of B and
vice versa. Initially, the speed of A is thrice the speed of B. Assume that they start from M0 and
they first meet at M1, then at M2, next at M3, and finally at M4.

15. What is the point that coincides with M0 along the course?
1) M1 2) M2 3) M3 4) M4
(Past CAT question)

16. What is the shortest distance between M1 and M2 along the course?
1) 11 km 2) 7 2 3) 7 km 4) 14 km
(Past CAT question)

17. What is the shortest distance between M1 and M3 along the course?
1) 22 km 2) 14 2 3) 22 2 4) 14 km
(Past CAT question)

18. What is the total distance travelled by A when they meet at M3?
1) 77 km 2) 66 km 3) 99 km 4) 88 km

(Past CAT question)

DIRECTIONS for questions 19 to 23: Choose the correct alternative.

19. In a watch, the minute hand crosses the hour hand for the third time exactly after every 3
hours, 18 minutes, 15 seconds of watch time. What is the time gained or lost by this watch
in one day?
1) 14 min 10 sec, lost 2) 13 min 50 sec, lost
3) 13 min 20 sec, gained 4) 14 min 40 sec, gained
(Past CAT question)

20. In a mile race, Akshay can be given a start of 128 metres by Bhairav. If Bhairav can give
Chinmay a start of 4 metres in a 100-metre dash, then who out of Akshay and Chinmay will
win a race of one and half miles, and what will the final lead given by the winner to the
loser be (in terms of miles)? (One mile is 1600 metres).
1 1 1 1
1) Akshay, 2) Chinmay, 3) Akshay, 4) Chinmay,
12 32 24 16
(Past CAT question)

227
CATapult
ARITHMETIC

21. If Raghu and Ram are running along a circular path after starting from the same point and
running in the same direction with speeds 9 m/s and 11 m/s respectively, how many times
will Ram overtake Raghu before they meet at the starting point for the first time?
1) 1
2) 2
3) 9
4) Depends on the circumference of the circular track

22. In a 100 metres walking race, Abbas gives Mustaan a headstart of 4 metres and beats him
by 19 seconds. If Abbas gives Mustaan a headstart of 30 seconds, by how much will the
winner win over the loser, if it is known that the ratio of Abbas’ speed to that of Mustaan
is 6 : 5?
1) Abbas will win by 5 seconds. 2) Mustaan will win by 5 seconds.
3) Abbas will win by 10 seconds. 4) Mustaan will win by 10 seconds.

23. Three participants—A, B and C—ran a race of total length 192 m. A won the race ahead of
B by 48 m and ahead of C by 72 m. By what distance (in metres) did B win the race ahead
of C?
1) 30 2) 32 3) 36 4) 40

DIRECTIONS for question 24: Solve as directed.

24. Karan and Arjun are running a race between points A and B. If both start from point A,
Karan beats Arjun by 20 m. If Karan starts from 10 m behind A and Arjun starts from point
A, Karan beats Arjun by 12 m. What is the distance between points A and B (in metres)?

228
CATapult
ARITHMETIC

QA-3.10 TSD APPLICATIONS - II CLASS EXERCISE

Teaser

Tony plans to go on a tour of South India in his car. All the 4 tyres in his car have been recently
replaced and in addition he has a new spare tyre. From experience he knows that a tyre wears
out after 2500 km of travel. However, the route he plans to take is 3000 km long. Will he be
able to manage the tour without needing to buy fresh tyres?

229
CATapult
ARITHMETIC

Answer questions 1 to 3 based on the following information:

A jogging track in a park is in the shape of a regular hexagon inscribed in an equilateral triangle
as shown in the adjoining figure. Two people M and N start jogging
along this track along different paths. The speeds of M and N are in
the ratio 1 : 2 and M can complete the distance PR in 9 minutes.

1. If both of them start jogging from Y and M follows the clock-


wise path round the hexagon while N follows the clockwise
path around the equilateral triangle, where will they meet for
the first time? After how long?

2. If after the meeting in the previous question, M reverses his


direction of jogging, where and when will they meet for the
second time?

3. If both of them start together at Y as in the first question, by what percent will M have to
change his speed so that both of them complete one round and returns to Y at the same
time?

4. Passenger trains ply between Kochi and Kollam throughout the day at 30 minutes interval. The
first train of the day starts at 7 AM while the last train starts at 11.30 PM from both Kochi
and Kollam. The distance between the two cities can be covered in four hours. Muralidharan
boarded a train at Kochi at 9 AM. How many trains travelling in the opposite direction will
he see (including the trains at either station) till he reaches Kollam?

5. Two trains A and B start simultaneously at 10 a.m from cities P and Q towards each other
at speeds of 60 and 80 kmph respectively. They meet at point R at 10 pm. After how long
will they reach their respective destinations Q and P?

6. Two people W and X start from two points Y and Z walking towards each other’s starting
points. They meet along the way and then W takes 18 minutes more while X takes 50 min-
utes more to reach their destinations. How much time after starting did they meet? What is
the ratio of their speeds?

7. * At 9 a.m., A starts driving from city P to city Q at a constant speed. At 10 a.m., B starts
driving from city Q to city P, at a different constant speed. The two of them meet along the
way at 10:48 a.m., and eventually reach their destinations simultaneously. At what time did
they reach?

230
Chapter QA
TSD APPLICATIONS-II 3.10

8.

THEORY
PRACTICE EXERCISE CLASS EXERCISE
* A robotics design competition requires remote-controlled cars to move along the paths in
the adjacent figure (where ABCD is a rectangle circumscribing two circles touching external-
ly). One car moves along the circular path JEKHGFKIJ while a second moves along the path
JAEFBGCHIDJ. If both start from J at the same time, and also finish simultaneously, what
is the ratio of their speeds?

9. At 9:00 AM, Ajay and Vijay started walking together at the same speed for Udaipur railway
station in order to reach just in time to catch a train there at 11:00 AM. When they were
exactly midway, Vijay realized that he forgot his ticket. Therefore he immediately rushed back
home in a horse-cart, while Ajay continued walking towards the railway station. Vijay collected
his ticket from home and took the same horse-cart to the railway station and reached the
railway station just in time to catch the train. What is the ratio of the speeds of the horse
cart to the walking speed of Ajay?

10. Donald was supposed to reach Washington railway station in a train from New York at 10
AM and his brother, Mike was suppose to leave home at 8 AM in a car to reach Washington
railway station exactly at 10 AM to receive Donald. However, Donald got ticket on an earlier
train and reached Washington at 7 AM. On reaching Washington, Donald immediately called
Mike and Mike left home right away. In the meantime, Donald took a horse-cart and started
travelling towards home. Mike met Donald and both Mike and Donald drove back home. If
they reached home at 10.30 AM, what is the ratio of the distance travelled by Donald in a
horse cart to the distance travelled by him in a car?

11. Mr. Slow drives at a speed of 25 kmph while Mr. Fast drives at a speed of 50 kmph. Both
drivers drove from point P to point Q and then immediately return to point P. Mr. Slow started
at 8 AM while Mr. Fast started at 10 AM. Mr. Fast reached point Q and on his way back to
point P, he met Mr. Slow, who was still on his way to point Q at 12:40 PM. Calculate the
distance between points P and Q.

12. Two friends named Ajay and Vijay simultaneously start swimming from points A and B re-
spectively towards points B and A respectively with constant speed. On reaching points B
and A respectively, they immediately turn back and run towards their starting points with
same speed. They continue swimming this way. On their way, they meet each other at a
point 500 m away from A during their first meeting and at a point 300 m from B in their
second meeting.
Calculate:
1. The distance of their third meeting point from point A.
2. The distance of their third meeting point from point B.

231
CATapult
ARITHMETIC

13. Two cats named Amy and Pamy simultaneously start running in opposite directions from
vertex A of a running track ABC in the shape of an equilateral triangle of side 500 m. They
keep running indefinitely. If their first meeting point is along side BC at a distance 200 m
from vertex B, what would be the location of their 20th meeting?
1] Between point A and point B 2] Point B
3] Between point B and point C 4] Between point C and point A

14. Mr. One and Mr. Two simultaneously started driving towards each other at the speed of 25
kmph from two points separated by 1000 km. At the same time, Mr. Three, who was exactly
at the midpoint of the segment joining Mr. One and Mr. Two, started driving towards Mr. One
at 100 kmph. On meeting Mr. One, Mr. Three immediately reversed his direction and drove
towards Mr. Two at the same speed of 100 kmph. On meeting Mr. Two, Mr. Three immediately
reversed his direction and drove towards Mr. One at the same speed. Mr. Three continued
driving this way at the same speed till Mr. One and Mr. Two met. Calculate:
1. Total distance driven by Mr. Three
2. Distance driven by Mr. Three in the direction towards Mr. One
3. Distance driven by Mr. Three in the direction towards Mr. Two

15. Everyday I go for a morning walk to a hill at a distance of 3 km from my home. Today, my
dog, Rocky also accompanied me. He left home with me and ran to the hill at a speed three
times that of my walking speed. On reaching the hill, he immediately reversed his direction
and ran towards me at the same speed. On meeting me, he again ran towards the hill at
the same speed. This process continued till I reached the hill.
Calculate:
1. The total distance run by Rocky
2. The distance run by Rocky towards the hill
3. The distance run by Rocky towards me

16. Two trains, named A and B are at a distance of 200 km and they simultaneously start to-
wards each other at equal speed of 100 km. A bird, who is perched on the engine of train
A starts flying towards train B at a speed 200 kmph. The bird keeps flying between the two
trains at the same speed without stopping till two trains meet.
Calculate:
1. Total distance travelled by the bird
2. The distance travelled by the bird in the direction from train A to train B
3. Total distance travelled by the bird in the direction from train B to train A

17. Two local trains simultaneously start from Bandra and Andheri towards each other with speeds
30 km/hr and 42 km/hr respectively. A crow is perched on the engine of the train starting
from Bandra while a parrot is perched on the engine of the train starting from Andheri. Both
crow and the parrot start flying towards each other with speeds 48 km/hr and 60 km/hr
respectively from the moment the two trains start. The crow and the parrot meet each oth-
er and then immediately return to the trains that they started from. They again fly towards
each other and on meeting, immediately fly back towards the train they started from. This
process continues till the two trains meet. If the distance between Bandra and Andheri is
24 km, what is the total distance flown by the crow?
1] 20 km 2] 16 km 3] 32 km 4] 40 km

232
Chapter QA
TSD APPLICATIONS-II 3.10

18. Rajesh started walking towards his home from a point 6 km away. At the same instant, his dog

THEORY
started running from home towards him at twice the speed of Rajesh. The dog met Rajesh,
then immediately reversed his direction and ran towards the home at the same speed. The
dog continued running between home and Rajesh till the time Rajesh reached home.

PRACTICE EXERCISE CLASS EXERCISE


Calculate:
1. Total distance run by the dog
2. The distance run by the dog towards the home
3. The distance run by the dog towards Rajesh

19. Tushar takes 80 seconds to climb down an escalator that is moving downwards but he takes
100 seconds to climb down an escalator that is moving upwards. If the escalator is stationary,
how many seconds would he take to climb down the escalator?

20. Tushar is climbing down an escalator that is moving downwards from the first floor to the
ground floor. He takes 100 seconds to reach the ground floor and traverses 60 steps if he
decides to walk. On the other hand, he takes 60 seconds and traverses 90 steps if he de-
cides to run. How many steps are there on the escalator?

21. Tushar and Vishal are climbing on an escalator that is moving up. Tushar takes 25 seconds
while Vishal takes 20 seconds to reach the top because Vishal is faster than Tushar. Vishal
takes 2 steps per second, while Tushar takes 1 step per second. What is the total number
of steps on the escalator?

22. Tushar is climbing up an escalator that is going up, while Vishal is climbing down the same
escalator. For every one step Tushar takes, Vishal takes two steps. Both start together and
reach their destinations together. If Tushar walks 48 steps up the escalator, what is the total
number of steps of the escalator?

23. Raghav is walking at a constant speed beside a busy highway that connects Ahmedabad and
Vadodara. Buses ply between the two cities at equal intervals in both directions. He encoun-
ters a bus going from Ahmedabad to Vadodara after every 12 minutes and a bus going from
Vadodara to Ahmedabad after every 10 minutes. What is the time interval between the two
consecutive buses going from Ahmedabad to Vadodara if it is known that the buses in both
directions run at the same speed?

233
CATapult
ARITHMETIC

Challengers

For questions 1 to 3 refer to the data below:

There is a railway line in East-West direction. Three railway stations—Rampur, Laxmanpur and
Sitapur are on the railway line, with Rampur being towards West and Sitapur being towards East
and Laxmanpur between Rampur and Sitapur. Ajay is standing exactly to the south of Laxmanpur
railway station at a distance of 2.4 km. He is 3 km away from Rampur station and 4 km away
from Sitapur station. A train travelling eastward is approaching Rampur station. The speed of train
and Ajay are such that if Ajay had walked at his normal speed, he would have just caught the train
either at Rampur or Sitapur. However Ajay decides to walk up north towards Laxmanpur station
and board the train there. On reaching Laxmanpur station, he realizes that the train does not halt
there. Therefore he decides to walk towards Sitapur station. As Ajay is on his way to Sitapur, he
receives a call from his brother saying that the train has just left Rampur and he should better
run lest he would miss the train.
(Note: Halting time of the train at Rampur, time spent by Ajay at Laxmanpur and time spent for
the phone call may be considered negligible)

1. How much faster should Ajay run as a percentage of his normal walking speed in order to
just catch train at Sitapur?
1) He should run 200% faster as compared to his normal walking speed
2) He should run 260% faster as compared to his normal walking speed
3) He should run 160% faster as compared to his normal walking speed
4) Even if he walks at his normal walking speed, he would be catch the train

2. If Ajay continues to walk from Laxmanpur to Sitapur at his normal walking speed, where
would he be when the train reaches Sitapur?
1) 1.2 km short of Sitapur 2) 1.6 km short of Sitapur
3) 1.8 km short of Sitapur 4) At Sitapur station

3. How much faster does the train run as compared to Ajay’s normal walking speed?
1) 500% faster 2) 250% faster
3) 300% faster 4) 400% faster

4. An escalator in a mall has 30 steps. Mithun steps on it and climbs along with it and climbs
10 steps by the time he reaches the top. Rajni climbs at twice the speed Mithun does. How
many steps will he need to reach the top?
1) 12 2) 15 3) 18 4) 20

5. Shanti’s school normally finishes at 4 pm. Her mom drives from home to pick her up, reach-
ing the school exactly at 4 pm. One day, a half-holiday is announced and the school finishes
for the day at 1 pm. Rather than sitting and waiting, Shanti decides to start walking towards
home. Her mother meets her along the way and as a result they reach home an hour earlier
than normal. What is the ratio of Shanti’s walking speed to her mother’s driving speed?
1) 1 : 4 2) 1 : 2 3) 1 : 5 4) 1 : 8

234
Chapter QA
TSD APPLICATIONS-II 3.10

PRACTICE EXERCISE

DIRECTIONS for questions 1 and 2: Answer the questions on the basis of the information
given below.

Cities A and B are in different time zones. A is located 3000 km east of B. The table below
describes the schedule of an airline operating non-stop flights between A and B. All the times
indicated are local and on the same day.

Departure Arrival
City Time City Time
B 8:00 AM A 3:00 PM
A 4:00 PM B 8:00 PM

Assume that planes cruise at the same speed in both directions. However, the effective speed is
influenced by a steady wind blowing from east to west at 50 km per hour.
1. What is the time difference between A and B?
1) 2 hours and 30 minutes 2) 1 hour 3) 1 hour and 30 minutes
4) 2 hours 5) Cannot be determined
(Past CAT question)

2. What is the plane’s cruising speed in km per hour?


1) 600 2) 500 3) 700
4) 550 5) Cannot be determined.
(Past CAT question)

DIRECTIONS for questions 3 and 4: Choose the correct alternative.

3. A jogging park has two identical circular tracks touching each other and a rectangular track
enclosing the two circles. The edges of the rectangles are tangential to the circles. Two
friends, A and B, start jogging simultaneously from the point where one of the circular tracks
touches the smaller side of the rectangular track. A jogs along the rectangular track, while B
jogs along the two circular tracks in a figure of eight. Approximately, how much faster than
A does B have to run, so that they take the same time to return to their starting point?
1) 3.88% 2) 4.22% 3) 4.44% 4) 4.72%
(Past CAT question)

4. On a rectangular track, points A and B are diagonally opposite. If X starts running from point
A towards point B along rectangular track, he takes 12 minutes more than he would have
taken had he run along the straight path. This difference is 4 minutes for Y. Z travels half
the distance with X’s speed and remaining half with Y’s speed taking 10 minutes to travel
9 km. Find the speed of Y.
1) 10 m/s 2) 15 m/s 3) 12 m/s
4) 9 m/s 5) None of these

235
CATapult
ARITHMETIC

DIRECTIONS for questions 5 and 6: Refer to the data below and answer the questions that
follow.

Kedar and Vishnu are moving along a rectangular track ABCD. Kedar starts cycling at 6 kmph from
point A at 8:00 am on the route A-D-C-B-A. Vishnu starts walking at 8:50 am from point C at the
speed of 1 kmph and takes the route C-B-A-D-C.
AB = 6 km and BC = 4 km.

5. At what time does Kedar overtake Vishnu?


1) 10:15 am 2) 9:00 am 3) 9:50 am
4) 10:00 am 5) None of these

6. How far away from each other are the two at 10:50 am?
1) 5 km 2) 4 km 3) 10 km 4) 17 km 5) 6 km

DIRECTIONS for questions 7 and 8: Refer to the data below and answer the questions that follow.

K is travelling from A to E through four different roads of equal length i.e., AB, BC, CD and DE.
He covers this distance in 10 minutes at a speed of 1.2 kmph.

7. Find the length AD, if AC = BD = CD.


1) 50 m 2) 50 3 m 3) 100 m
4) 100 3 m 5) 25 6 m

8. If there is a direct road joining A and D, find the approximate time taken by K to reach E,
if he travels directly via D. (use data from the previous question)
1) 5 minutes 2) 7 minutes 3) 8 minutes
4) 9 minutes 5) Cannot be determined

DIRECTIONS for questions 9 to 12: Choose the correct alternative.

9. A wolf spots a rabbit 100 m away and starts chasing it. The rabbit realizes this after two
minutes and starts running away from the wolf towards a burrow which is 40 m from where
the rabbit is right now. The rabbit covers 1 m in 3 leaps whereas the wolf covers 3 m in 4
leaps. At what speed should the rabbit run so that it just manages to reach the burrow if the
wolf is running at 20 m per minute? (The wolf, rabbit and burrow are in a straight line.)
180
1) 24 leaps/min 2) 18 leaps/min 3) 16 leaps/min 4) leaps/min
11

236
Chapter QA
TSD APPLICATIONS-II 3.10

10. Two cats named Amy and Pamy are standing at points A and B respectively. They simultane-

PRACTICE EXERCISE CLASS EXERCISE THEORY


ously start running towards each other and meet after ‘t’ seconds. After meeting each other,
Amy takes 6.4 seconds to reach point B whereas Pamy takes 10 seconds to reach point A.
What is the value of ‘t’ (in seconds)?
1) 8
2) 6
3) 7.2
4) More information is needed to answer the question

11. A and B are standing at the opposite ends of a 200-metre long stretch of road. They start
driving towards each other. The one who reaches the half way first wins the race, however
they continue driving towards the opposite point even after the race is over. It is known the
A won 1.33 seconds before meeting B along the way and B takes 6.66 seconds more to
travel the entire distance than A. What is the speed of A?
1) 10 m/s 2) 12 m/s 3) 15 m/s 4) 18 m/s

12. Basanti is walking beside a railway track between Pune and Baramati at a constant speed
towards Baramati. Local trains ply between the two cities at equal intervals in both direc-
tions. She encounters a train going from Pune to Baramati after every 8 minutes and a train
going from Baramati to Pune after every 6 minutes. What is the time interval between the
two consecutive trains going from Pune to Baramati?
1) 7 minutes
6
2) 6 minutes
7
1
3) 6 minutes
2
4) More information is needed to answer this question

DIRECTIONS for questions 13 and 14: Solve as directed.

13. Karan and Arjun are standing at points A and B respectively, separated by 500 meters. Karan
starts running towards Arjun while Arjun starts running away from Karan, both along the same
line. Since Karan runs faster than Arjun, Karan catches up with Arjun at a point beyond B. On
meeting Arjun, Karan immediately reverses his direction and starts running towards point B
and then continues towards point A. If Arjun has run 1200 meters by the time Karan reaches
point A, how much distance has Karan run (in meters)?

14. Gaurav starts running along a straight line at uniform speed from point A to point B, which
is located at a distance of 3000 m from point A. At the same instant, his pet dog starts
running from point B to point A at a speed twice that of Gaurav. After the dog meets Gaurav
at a point between A and B, the dog reverses his direction and starts running towards point
B. On reaching point B, the dog again reverses his direction and meets Gaurav. On meeting
Gaurav, the dog again reverses his direction and starts running towards point B. This process
continues till Gaurav reaches point B. What is the distance run by the dog in the direction
opposite to Gaurav (in meters)?

237
CATapult
ARITHMETIC

DIRECTIONS for questions 15 to 18: Choose the correct alternative.

15. Every morning, Rajesh goes for morning walk from his house to a temple located at a dis-
tance of 6 km from his house at a uniform speed of 6 km/hr. Today his pet dog, who was
exactly midway on the way to temple started running towards the temple the same instant
when Rajesh started from his house. The dog reached the temple, immediately turned back
and started running towards Rajesh (who was on his way to temple). On meeting Rajesh,
his dog again reversed his direction and started running towards the temple. This process
continued till Rajesh reached the temple. What was the distance run by the dog against the
direction of Rajesh (i.e. in the direction from the temple to the house) if the speed of the
dog was 11 km/hr?
1) 7 km 2) 4 km 3) 11 km 4) 8 km

16. There is an escalator connecting the foot of a hill and the hilltop that goes up from the foot
to the top. While going up, Sachin took the escalator and reached the top in 20 seconds
after taking 120 steps. While returning, he took the same escalator but walked down on the
escalator that is moving up. He reached the foot of the hill in 40 seconds after having taken
240 steps. If the escalator is stationary, how many steps would he have to take to climb
from the foot to the top?
1) 200 2) 180 3) 160 4) 144

17. Rahim plans to drive from city A to station C, at the speed of 70 km per hour, to catch a
train arriving there from B. He must reach C at least 15 minutes before the arrival of the
train. The train leaves B, located 500 km south of A, at 8:00 am and travels at a speed of
50 km per hour. It is known that C is located between west and northwest of B, with BC at
60° to AB. Also, C is located between south and southwest of A with AC at 30° to AB. The
latest time by which Rahim must leave A and still catch the train is closest to
1) 6:15 am 2) 6:30 am 3) 6:45 am 4) 7:00 am 5) 7:15 am
(Past CAT question)

18. There is a race track ABCDEFGH in the shape of a regular octagon. Two runners start running
from point A along the race track in opposite directions at speeds which are in the ratio 9
: 7. While running along the track, they meet each other multiple times. At which of the
following meetings will the two be the farthest from point A?
1) 12th 2) 15th 3) 20th 4) 24th

DIRECTIONS for questions 19 and 20: Solve as directed.

19. There is a ground ABC in the shape of an equilateral triangle with A, B and C as its verti-
ces. Amy, a cat, starts running from vertex A in the direction towards C along side AC. At
the same instant, Pamy, another cat, starts running from vertex C in the direction towards
A along side CA. The two cats meet for the first time after 12 seconds. After meeting, they
continue running in their respective directions along the sides of the triangle. After how many
seconds will they meet for the third time?

20. Rohit climbs up an escalator which is going up. If he climbs 35 steps, he will reach the top
in 28 seconds. If he climbs 44 steps, he will be able to reach the top in only 16 seconds.
How many steps are visible on a stationary escalator?

238
CATapult
GEOMETRY

QA-3.11 TRIGONOMETRY THEORY

Definition of Trigonometrical ratios

Trigonometry is the study of the relationship between the sides and angles of a right-angled triangle.

Angles are measured in degrees or radians.

1 degree = 1o = 60 minutes

°
One radian = 1c = b
180 l
and 1o = b l
r c
r 180
r
 Radian Measure = × Degree Measure
180°
180°
Degree Measure = × Radian Measure
r
Degree 30° 45° 60° 90° 180° 270° 360°
Radian r r r r  3r 2
6 4 3 2 2

In a right angled triangle ABC, if mABC = 90°, then AC is the hypotenuse.


With respect to ACB, AB is the opposite side and BC is the adjacent side.
With respect to BAC, BC is the opposite side and AB is the adjacent side.

In a right angled triangle for a given acute angle, BAC there are six possible ratios of sides.
The values of all these ratios are constant. These ratios are called trigonometric ratios.

Opposite side BC
Sine  i.e., sin = =
Hypotenuse AC

Adjacent side AB
Cosine  i.e., cos = =
Hypotenuse AC

Opposite side BC
Tangent  i.e. tan = =
Adjacent side AB
Hypotenuse AC
Cosecant  i.e., cosec = =
Opposite side BC
Hypotenuse AC
Secant  i.e., sec = =
Adjacent side AB

Adjacent side AB
Cotangent  i.e., cot = =
Opposite side BC

239
CATapult
GEOMETRY

Relation between the trigonometrical ratios of complementary and supplementary angles


Opposite side BC
1. sin  = =
Hypotenuse AC
Adjacent side BC
cos (90 – ) = =
Hypotenuse AC
 sin  = cos(90 – ) for 0o <  < 90o
2. cos  = sin(90 – )
3. tan  = cot(90 – )
4. cot  = tan(90 – )
5. cosec  = sec(90 – )
6. sec  = cosec(90 – )
7. tan  × tan(90 – ) = 1
8. cot  × cot(90 – ) = 1
9. cos(90 + ) = –sin 
10. sin(90 + ) = cos 
11. sin(180 – ) = sin
12 cos(180 – ) = –cos

Example
sin25
(i) Find the value of .
cos65
sin25 cos65
sin 25 = cos (90 – 25) = cos 65;  = = 1
cos65 cos65

(ii) If cos 50 = 0.6428. Find sin 40.

sin 40 = cos (90 – 40) = cos 50 = 0.6428

Trigonometrical Identities

BC AC
1. sin  = , cosec  =
AC BC
 sin  × cosec  = 1

2. cos  × sec  = 1

3. tan  × cot  = 1

Note:
cosecant, secant and cotangent ratios are the reciprocal of sine, cosine and tangent ratios
respectively.

240
Chapter QA
TRIGONOMETRY 3.11

sini
4. tan  =

THEORY
cosi
cosi
5. cot  =
sini

PRACTICE EXERCISE CLASS EXERCISE


6. sin2  + cos2  = 1

7. sec2  = 1 + tan2 

8. cosec2  = 1 + cot2 

9. sin(–) = –sin

10. cos(–) = cos

Trigonometrical ratios of some angles

241
CATapult
GEOMETRY

Graphs of trigonometric functions (sinusoidal curves)

Signs of Trigonometrical ratios in the Cartesian Plane

This can be easily remembered as:

Students All
Take Coffee

Note:
The sine, cosine and tangent ratios of 0°, 30°, 45°, 60° and 90° are most important and need
to be learned by heart.
0 1 2 3 4
The values of sine ratios are the square root of the fraction , , , and .
4 4 4 4 4
The values of cosine ratio are written in the reverse order of sine ratios and the values of
tangent ratios are obtained by dividing the respective sine ratio by the cosine ratio.

242
Chapter QA
TRIGONOMETRY 3.11

Example

THEORY
Find the value of cos 30 . cos 60 + sin 30 . sin 60
cos 30 . cos 60 + sin 30 . sin 60

PRACTICE EXERCISE CLASS EXERCISE


3 1 1 3 3 3 3
= × + × = + =
2 2 2 2 4 4 2

Appplications of Trigonometry in Geometry


1
1. Area of a triangle = × Product of 2 sides × sine of included angle.
2
2. Area of a parallelogram = Product of 2 sides × sine of included angle.

1
3. Area of a quadrilateral = × product of diagonals × sine of angle between them.
2
4. In a ABC, where a, b and c are the lengths of the opposite sides of BAC, ABC
and ACB.
a b c
= =
sinA sinB sinC
This is called the sine rule.

5. In ABC,
a2 = b2 + c2 – 2bc cos A
b2 = c2 + a2 – 2ca cos B
c2 = a2 + b2 – 2ab cos C.
This is called cosine rule.
In case of a right angled triangle, if C = 90°

c2 = a2 + b2 – 2ab cos90°

 c2 = a2 + b2 [Pythagoras theorem]

Example

(i) In ABC, BC = 13 cm, AC = 8 cm and AB = 7 cm. Find BAC.


2 2 2 2 2 2
b +c –a 8 + 7 – 13 64 + 49 – 169 1
cos A = = = = – ;
2bc 2×8×7 2×8×7 2
 BAC = 120°

(ii) Find the area of ABC, if AB = 10 cm, BC = 8 cm and ABC = 30°.


1
A(ABC) = × product of two sides × sine of included angle
2
1 1 1
= × 10 × 8 × sin 30 = × 10 × 8 × = 20 sq. cm.
2 2 2

243
CATapult
GEOMETRY

Angles of Elevation & Depression

If a person on a lower level looks up at an object at a higher level,


the line of sight makes an angle with the horizontal called the angle of
elevation.

If a person standing at a higher level observes an object at a lower level,


the line of sight makes an angle with the horizontal called the angle of
depression.

Note:
Numerically angle of elevation is equal to the angle of depression.
The angle of elevation and angle of depression are measured with the horizontal.

Example
A boy is standing on the road at a distance of 50m from the bottom of a tower, and his angle of
elevation of the tower is 30°. Find the height of the tower.
Let AB be the height of the tower and let the boy be standing at position C.
Now, AB = height of the tower and BC = 50m
In ABC, C = 30°
AB
 tan 30° =
BC
1 AB
=
3 50
50
AB = m
3

244
Chapter QA
TRIGONOMETRY 3.11

Advanced Trigonometrical Formulae

THEORY
Some of these formulae are used in management entrance exams like XAT, IIFT etc.

1. Cos (A + B) = CosA CosB – SinA SinB

PRACTICE EXERCISE CLASS EXERCISE


2. Cos (A – B) = CosA CosB + SinA SinB

3. Sin (A + B) = SinA CosB + CosA SinB

4. Sin (A – B) = SinA CosB – CosA SinB

tanA + tanB
5. tan (A + B) =
1 – tanA tanB
tanA – tanB
6. tan (A – B) =
1 + tanA tanB
7. sin2x = 2sinx.cosx

8. cos2x = cos2x – sin2x

Concept Builder 1

Convert b l into degrees


r c
1.
4
2. Convert 60° into radians
1
3. If sin = . find a) cos b) tan
3
4. The bottom of a tree is at a horizontal distance of 60m from an object lying on the ground.
From the top of the tree, the angle of depression with respect to the object is 60°. Find
a) The angle of elevation of the object with respect to the top of the tree
b) The height of the tree
5. In the fig. Find BC.

6. Find Sin 15° [Use Sin (A – B) formula]

Answer Key
2 2
6. 4 3 5. a) 60° b) 60 3 4.
3 –1
2 3 3
, b) tan = 3. a) cos = 2. 45° 1.
1 2 r

245
CATapult
GEOMETRY

SOLVED EXAMPLES

Q : In the figure, EF = 100 cm, mHEF = 30° and mHFG = 60°. Find HG.
A : mHFE = 180 – 60 = 120°.
 mFHE = 30°  FH = EF = 100 cm …[Isosceles tri-
angle]
HFG is a 30° – 60° – 90° triangle
3 3
HG = side opp. 60° = × FH = × 100 = 50 3
2 2
cm.

Q : From the top of a cliff the angle of depression of a point on the ground 150 feet away from
the bottom of the cliff is 30°. Find the height of the cliff.
A : Let CA be the cliff.
CA
tan 30 =
AP
1 CA 150
= ;  CA = = 50 3 feet
3 150 3

Q : From the top of a hill the angle of depression of the top


and the bottom of a tower are observed to be 45° and
60° respectively. If the height of the hill is 100 3 m,
find the height of the tower.

HL 100 3
A : tan 60 = ; 3 =
LR LR
 LR = 100
LR = HC = 100 m
HCT is a 45° - 45° - 90° triangle.  HC = CT =
100 m
 TR = 100 3 – 100 = 100(1.732 – 1) = 73.2 m

246
Chapter QA
TRIGONOMETRY 3.11

CLASS EXERCISE

Teaser

How many equilateral triangles having length of side equal to one unit can be formed by using 6
matchsticks of length one unit each?

247
CATapult
GEOMETRY

Trigonometry

Consider the right-angled triangle shown in the adjoining figure.

Trigonometric ratios for the angle θ may be defined as follows:

Opposite b Hypotenuse a
sin  = = , cosec  = =
Hypotenuse a Opposite b

Adjacent c Hypotenuse a
cos  = = , sec  = =
Hypotenuse a Adjacent c

Opposite b Adjacent c
tan  = = , cot θ = =
Adjacent c Opposite b

Some useful rules: • sin2 + cos2 = 1 • sin = cos (90 – ) • sin (2) = 2 sin cos

248
Chapter QA
TRIGONOMETRY 3.11

1. In the adjoining triangle, calculate the following ratios

THEORY
sin  =
cos  =

PRACTICE EXERCISE CLASS EXERCISE


tan  =

sin  =
cos  =
tan  =

cosec  =
sec  =
cot  =

cosec  =
sec  =
cot  =

2. *Consider a right angled triangle with sides 9 cm, 40 cm and 41 cm. Calculate—
a) sin of the angle opposite to side of length 9 cm
b) cos of the angle opposite to side of length 40 cm
c) tan of the angle opposite to side of length 9 cm.

3. Calculate sin, cos and tan of 30, 45 and 60

Now fill in the following table:

30 45 60
Sin
Cos
Tan

249
CATapult
GEOMETRY

Graphs of trigonometric functions (sinusoidal curves)

4. In triangle BAC, BD is an altitude on side AC, which meets side AC in D such that A-D-C.
Suppose side AB = 6, side BC = 10 and angle C = 30 degrees. Calculate lengths of AD, DC
and BD.

sin1.sin2.sin3.............sin89
5. Calculate the value of
cos1.cos2.cos3...........cos89

Advanced trigonometric formulae

1. sin (A + B) = sin A cos B + cos A sin B


2. cos (A + B) = cos A cos B " sin A sin B
tanA ! tanB
3. tan (A + B) =
1 " tanA tanB
4. sin 2x = 2 sin x cos x
5. cos 2x = cos2x – sin2x

1
6. If each of a, b and c is a positive acute angle such that sin (a + b – c) = , cosec (b +
2
2 1
c – a) = and tan (c + a – b) = , what are the values of a, b and c?
3 3
1) 37.5, 52.5, 45 2) 37,53,4 3) 45,37.5,52.5 4) 34.5,55.5,45

7. If cot2 – (1 + 3 ) cot + 3 = 0, then what is the value of  ?


1) 90, 60 2) 45, 60 3) 45, 30 4) 90, 180

250
Chapter QA
TRIGONOMETRY 3.11

8. Given that 2 sin x cos x = sin 2x and 1 – 2 sin2x = cos 2x, which for the following is always

THEORY
true about f(x) = sin2x tan x – cos2x cotx, for 45 < x < 90 ?
1) f(x) is always positive
2) f(x) is always negative

PRACTICE EXERCISE CLASS EXERCISE


3) f(x) intersects the X-axis at only one point
4) None of the above

9. If x = sin215° + sin230° + sin245° + sin260°+ sin275°+ sin290°, what can be said about x?
1) 1 < x < 2 2) 2 < x < 3 3) 3 < x < 4 4) 4 < x < 5

10. Find the approximate value of (sin 34° + cos 56°)(sin 56° – cos 34°)

11. *If sin (3x) = cos (7x), where x is an angle between 0° and 90°, what will be the value of
tan (5x)?

Applications of trigonometric ratios


1
1. Area of a triangle: ab sinC
2

2. Area of a parallelogram: bc sin d

3. Suppose a regular polygon of n sides is divided into n congruent triangles by joining the
2
nr
vertices of the polygon with the center of the polygon, area of the polygon: sinθ,
2
360
where r is radius of the circumcircle and  =
n

12. Suppose in triangle ABC, side AB = 20 cm and side BC = 10 cm. If angle B = 30°, what is
area of triangle ABC?

13. Calculate area of parallelogram PQRS if SR = 20 cm, PS = 12 cm and angle Q = 45.

14. Calculate area of a regular octagon inscribed in a circle of radius 10 cm.

15. *Calculate area of a regular polygon with 20 sides inscribed in a circle of radius 10 cm.,
given that sin18 = 0.3090

16. *Calculate value of (sin 10 + cos 10)2 (1 – sin 20) – cos220

251
CATapult
GEOMETRY

Angle of elevation and depression

17. Sagar is standing in front of a building at a distance of 400 feet. The angle of elevation for
the top of the building is 30°. There is one electric pole between Sagar and the building.
Sagar can see the top of the building just beyond the top of the pole. If Sagar is standing
at a distance of 20 feet from the pole, what is the height of the pole?

18. There are two buildings, one on each bank of a river, opposite each other. From the top of
one building, which is 60 m high, the angles of depression of the top and the foot of the
other buildings are 30 and 60 respectively. What is the height of the other building?
1) 30 m 2) 18 m 3) 40 m 4) 20 m

19. A car is being driven, in a straight line and at a uniform speed towards the base of a vertical
tower. The top of the tower is observed from the car and in the process, it takes 10 minutes
for the angle of elevation to change from 45 to 60 degrees. After how much more time(in
minutes) will this car reach the base of the tower?
1) 5 ^ 3 + 1 h 2) 6 ^ 3 + 2h 3) 7 ^ 3 – 1 h 4) 8 ^ 3 – 2 h

20. Raju is a naughty boy. He climbs a tree of height 50 m and reaches the top. There is a pillar
in front of the tree. The angle of depression for the top of the pillar is 30 degrees. Raju can
also see that just beyond the top of the pillar, a dog is sleeping on the ground at a distance
of 10√3 m from the bottom of the pillar and 50√3 m from the bottom of the tree. An eagle
is flying exactly above the pillar such that Raju’s angle of elevation for the eagle from the
top of the tree is 60 degrees. What is the distance between the eagle and the top of the
pillar?
1) 120 meters 2) 120 3 meters 3) 160 meters 4) 130 meters

21. The angles of depression from a 150m tower to the top and bottom of a chimney are 30°
and 60° respectively. Find the height of the chimney.
1) 50√3 m 2) 75 m 3) 75 3 m 4) 100 m

22. *A soldier standing at the top of a tower of height 10 sees three enemies approaching along
a straight road. The angles of depression made by the three are 35°, 45° and 55°, while
their distances from the foot of the tower are a, b and c respectively. What is the value of
a × b × c?

252
Chapter QA
TRIGONOMETRY 3.11

Challengers:

THEORY
For all real numbers x, except x = 0 and x = 1, function F is defined by F b l = 1 . If
x
1.
x – 1 x
0 <  < 90, what is the value of F(cosec2)?

PRACTICE EXERCISE CLASS EXERCISE


1) sin2  2) cos2  3) tan2  4) cot2  5] sec2 

2. In triangle ABC, angle A = 60 degrees. AD is an angle bisector of angle A. Side AB = 8 and


side AC = 10 cm. Calculate length of AD.
40 3 50 3
1) 4 3 2) 3) 5 3 4)
9 9

3. A tower is at corner B of a rectangular field ABCD. From corner D, the top E of the tower
is at an angle of elevation of 30°, while from the corner C, it is at an angle of elevation of
45°.
Find the ratio of the lengths of the sides of the field.
1) 3 : 1 2) 2 : 1 3) 2 : 1 4) 1 : 1

4. A warship and a submarine (completely submerged in water) are moving horizontally in a


straight line. The captain of the warship observes that the submarine makes an angle of
depression of 30 degrees and the distance between them from the point of observation is
50 km. After 30 minutes, the angle of depression becomes 60 degrees. Find the distance(in
kms) between them after 30 minutes from the initial point of reference.
50 25
1) 2) 25 3) 4) 25 3
3 3

If sin + sin = a, cos + cos = b, tan b l * tan b l = c and a  b  c  0, c  1,


a b 1–c
5.
2 2 1+c
is equal to
b 2a 2b a
1) 2 2 2) 2 2 3) 2 2 4) 2 2
a +b a +b a +b +
a b

253
CATapult
GEOMETRY

PRACTICE EXERCISE
DIRECTIONS for questions 1 to 6: Choose the correct alternative.

1. If cot + tan = p then which of the following is true?

1) p sin i – sin i = 1 2) 1 = ptancos2


2 4

3) Both of these 4) None of these

2. If  &  are supplementary angles then tan2 – sec2 will be equal to:
1) sin2 + cos2 2) sin2 – cos2
3) cos2 + sin2 4) –sin2 – cos2

3. Find the simplified form of:


[sincos + sin(90 – )cos( + 90)]2 –[sincos– sin(90 + ) cos( – 90)].
1) 2 sin cos 2) (sin + cos)2 – 3 sin cos
3) 1 4) 0

4. If cosec - sin = a, sec – cos = b & tan + cot = c then which of the following is not
true?
1) abc = 1
2) (a2b) + (ab2) = 1
3) ab = sin cos
1 + sini + cosi
4) – (sin + cos) = a + b + c
ab

5. If cosec – sin = p and sec – cos = q then which of the following is true?
1) p2q2 + pq2 = 1 2) pq = 1
2 2/3 2 2/3
3) (p q) + (pq ) = 1 4) p/q = 1

6. Which of the following cannot be the value of cosec2x + sec2x?

1) 2 3 2) 3 3 3) 3 2 4) 2 5

DIRECTIONS for question 7: Solve as directed.

7. What is the maximum value that 4sinA + 3cos A can take?

254
Chapter QA
TRIGONOMETRY 3.11

DIRECTIONS for questions 8 to 20: Choose the correct alternative.

PRACTICE EXERCISE CLASS EXERCISE THEORY


8. If 50 ≤ x0 ≤ 150, then the value of sin 300 + cos x0 – sin x0 will be:
1) Between -1 and - 0.5 inclusive 2) Between - 0.5 and 0 inclusive
3) between 0 and 0.5 exclusive 4) between 0.5 and 1 inclusive
5) None of the above
(Past XAT question)

9. In the given figure, what is the value of sin B – sin A?

1) 0.5 2) 1 3) 0.33 4) 0.25

10. A dog moving towards a wall, observes a lizard moving up a wall. At the particular instant
the angle of elevation is 30°. When the lizard reaches the top of the wall, the dog is 15
metres away from the wall.
What is the distance travelled by the dog in this period if the lizard has travelled 5 3 m
(during the same period) and the angle of elevation of the dog’s eye with the lizard becomes
60o?
1) 16.5 m 2) 15 m 3) 16.5 3 m 4) 15 3 m

11. A crow, sitting on the ground, sees the top of a building at an angle of elevation of 30°
and starts flying towards it along a straight line. At the same time, a pigeon, who is sitting
at the bottom of the building, flies straight up towards the top of the building. They both
fly at the same speed. The pigeon reaches 20 seconds earlier than the crow. If they were
moving along the ground from their initial positions towards each other at the same speed,
after how much time (in seconds) will they meet?
1) 5 2) 5 3 3) 10 4) 10 3

12. A tower of height 100 m is installed at a point in an open ground. A cat named Amy is standing
at a point from where the angle of elevation of the top of the tower is 30°. Another cat named
Pamy is standing at a point on the line joining Amy’s position and the foot of the tower (on the
same side as Amy) such that the angle of elevation of the top of the tower from Pamy’s posi-
tion is 60°. If Amy and Pamy simultaneously start running towards each other, they meet at a
point from where the angle of elevation of the top of the tower is 45°. If Amy and Pamy both
simultaneously start running towards the tower from their original positions, how far from the
foot of the tower would Amy be when Pamy reaches the foot of the tower (in m)?
1) 100( 2 – 1) 2) 100 ( 3 + 1) 3) 100 4) 100 ( 3 – 1)

255
CATapult
GEOMETRY

13. There are two towers AB and CD (A and C being their tops and B and D being their feet).
The angle of elevation of C from B is 30° and the angle of elevation of A from D is 60°.
An eagle and a kite start flying at the same moment from the tops of towers AB and CD
respectively towards the foot of the other tower along a straight line and at uniform speeds.
If they meet at one point along their flight, find the ratio of their speeds.
1) 3 : 1 2) 3 3 : 2 3) 3 : 1 4) 3 3 : 1

14. On a regular octagon ABCDEFGH, two towers of heights h1 and h2 are set on the points D and
F respectively. The angle of elevation of the top of the tower at F from A is 45° while the angle
of depression of the point A from the top of the tower at D is 30°. Find the ratio h1 : h2.

2 1 3
1) 3 2) 3) 4)
3 3 2

15.

The upper half of a 20 metre high pole got bent away from the sun by 90° at its midpoint
such that the upper half and lower half of the pole are now at right angles to each other, as
shown in the figure. The sunrays fall at an angle of 60° with respect to the horizontal. Find
the distance between the shadows cast by the topmost point of the pole before and after it
got bent.

1) 10 e om 4) 10 ^ 3 – 1 h m
3 –1 10
2) m 3) 10 3 m
3 3

256
Chapter QA
TRIGONOMETRY 3.11

16. The angle of elevation of the top of a building from point A on the ground is 30°. The angle

PRACTICE EXERCISE CLASS EXERCISE THEORY


of elevation of the top of a tower from the top of the building is 60°. Point A and the tower
are on either side of the building. If the angle of elevation of the top of the tower from point
A is 45°, what is the ratio of the distances of the base of the building from point A and the
base of the tower respectively?

1) 3 : 1 2) 1 : 3 3) 1 : 2 4) 2 : 1

17. The angle of elevation of the top of a tower from point P on the ground is . The angle of
elevation of a certain point on the tower above the ground from point P is . If the distance
of the point P from the foot of the tower is 15, find the height of the tower. Given: tan  =
1
;  = 2.
3
1) 11.25 2) 13.75 3) 15 4) 16.25

18. A lizard is lying stationary at point A on the floor. It observes an insect on a vertical wall at
point B which is at an angle of elevation of 30° from A. As soon as the lizard starts moving
along the floor towards the vertical wall in the direction of the insect, the insect starts mov-
ing upwards along the vertical wall. Once the lizard reaches the edge of the wall, it starts
moving upward along the vertical wall towards the insect. The lizard is able to catch the
insect when it reaches point C on the vertical wall, which is at an angle of elevation of 60°
with respect to A. What is the ratio of the speeds of the insect and the lizard?

1 2 1 2
1) 2) 3) 4)
3 1+ 3 3+ 3 3+ 3

19. Buildings A and B are parallel to each other. The height of B is less than that of A. I am
standing in such a way that building B is between building A and me. I am standing at a

distance of 200 3 from building B and can see only 20 floors of building A. After travelling
400
a distance of × 3 m towards the buildings, I am able to see only the top of building
3
A. Building B has 20 floors.
(Height of floors in buildings A and B are the same. Also assume my height to be of negli-
gible value).
What is the distance between the buildings A and B?

1) 100 3 m 2) 200 3 m 3) 100 m 4) 200 m

257
CATapult
GEOMETRY

20. A person is standing at a distance of 1800 meters facing a giant clock at the top of a tower.
At 5.00 p.m., he can see the tip of the minute hand of the clock at 30 degree elevation from
his eye-level. Immediately, the person starts walking towards the tower. At 5.10 pm., the
person noticed that the tip of the minute hand made an angle of 60 degrees with respect to
his eye-level. Using three dimensional vision, find the speed at which the person is walking.

The length of the minute hand is 200 3 meters ( 3 = 1.732).


1) 7.2 km/hour 2) 7.5 km/hour
3) 7.8 km/hour 4) 8.4 km/hour
5) None of the above

(Past XAT question)

258

You might also like